1st series [1] [2] [3] [4] [5] [6] [7] [8] [9] [10] [11] [12] [13] [14] [15] [16] [17] [18] [19] [20] [21] [22] [23] [24]  2nd series [1] [2] [3] [4] [5] [6] [7] [8] [9] [10] [11] [12] [13] [14] [15] [16] [17] [18] [19] [20] [21] [22] [23] [24] [25] [26] [27] [28] [29] [30] [31] [32] [33] [34] [35] [36] [37] [38] [39] [40] [41] [42] [43] [44] [45] [46] [47] [48] [49]

  View the latest questions and answers at askaphilosopher.org

Ask a Philosopher: Questions and Answers 49 (2nd series)

When referring to an answer on this page, please quote the page number followed by the answer number. The first answer on this page is 49/1.

(1) Dave asked:

I have not studied philosophy but I seem to keep gravitating to it unwittingly.

I spend a lot of time thinking and I recently discovered Solipsism and thought it fit almost perfectly with my beliefs.

However, upon trying to find out more about it I just found people using it as a device to make ironic jokes. I just seek clarification on what I am and whether I am a type of Solipsist.

People seem to think that Solipsists would not want to congregate because by definition they are denying everyone but themselves and see little value in others.

I believe in quite the opposite. If all other people are fabrications of my mind, I would find great value in meeting with them especially those with similar ideas. This is because I am not consciously creating them and the fact that they are aspects of my mind's creation means that they are aspects of myself and I have created them for a reason.

Have I got Solipsism right or wrong? Or am I specific type of Solipsist?

---

I don't want to get into a boring taxonomy of philosophical positions. And who cares about names and labels anyway? However, it is necessary to make some preliminary 'cuts' in order to address Dave's question.

The first cut — the first option which I want to put on one side — is scepticism about other minds. Scepticism about other minds, or more specifically the hypothesis that I am the only conscious being in the universe, could be contingently true if either of the following circumstances obtained:

(a) I live in a world populated by robots disguised as human beings, each controlled by a pre-programmed tape. (This is to rule out the possibility, which a materialist might argue for, that if the 'robots' are genuine examples of AI, then they have consciousness just as I do.) The super-intelligent alien scientist who created the robots and tapes has died.

(b) Mind-body dualism, of the epiphenomenalist variety, is true, but I am the only person with a mind as well as a body. Everyone else that I meet is a zombie, which behaves in every respect just like a human being except that it lacks a mind or consciousness. (I'm not asserting that this is necessarily a coherent possibility, merely that it is initially plausible. I actually think that it is incoherent, but I won't try to show that here.)

The second cut I want to make relates to another contingent possibility, related to the Matrix scenario. Imagine that the machine world is devastated by a massive power cut, leaving only myself alive and my dreams of living in Sheffield in 2010 and answering questions for the Ask a Philosopher web site. Apart from my personal life-support system, all the machines have ground to a halt. It is possible that I am the only consciousness in the universe (assuming the absence of any alien life forms).

Well, actually, I am answering those questions, and not just dreaming that I am answering them, because we are assuming, by hypothesis, that I am in full possession of my intellectual faculties. However, the questions originate, not from named or anonymous surfers on the internet, but in the computer program my brain is interacting with. Dave and his question are the invention of the original Architect of the Matrix.

Why am I confident that none of these scenarios fits Dave's description? He states that he 'recently discovered Solipsism and thought it fit almost perfectly with my beliefs'. No plausible process of scientific investigation or inference to the best explanation could lead to the belief that I exist in a world populated by robots, or the other bizarre possibilities outlined above. You don't believe something just because it's possible, unless you are suffering from serious mental problems.

So now we need to supply an argument — which Dave does not give — which might plausibly have lead him to the conclusion that he is a 'solipsist'. By understanding how that argument works, we can diagnose exactly what kind of solipsist Dave is.

I suggest that the missing argument is along the lines given by Descartes at the beginning of the Meditations. All I know for certain is my own existence, and the fact that I have experiences. Descartes never actually goes this far: he proposes, as sceptical hypothesis, the idea that an evil demon is deliberately deceiving me into thinking that a material world and other people exist, when in reality all there is, is me and the evil demon. (Note, that this goes way beyond the Matrix scenario which assumes the existence of material objects in space.)

If all that exists is me and the evil demon, then my experience of looking at my computer monitor has two sides. It is my experience, but it is also produced by something external to my conscious mind. Dave would say at this point, 'Exactly! I am not consciously creating the computer monitor. But my mind is still the source of my experience.' But there is a problem here. What makes this 'unconscious' source of my conscious experience mine or part of me? My experience would be just as it is now if it was the evil demon who was responsible for it. Or, rather, 'my unconscious mind' is the evil demon for all intents and purposes.

This is still unsatisfactory, because one could argue that Dave is assuming something he has no right to assume: that when experience happens, it comes from somewhere, something is 'producing' it. Why?

A large part of the answer lies in our adherence to a certain model of causal explanation. You don't have an effect without a case. You can't have experience without something producing the experience. But isn't this a merely contingent matter? Based purely on my experience, I cannot say for certain whether it has an external cause or not.

I'm going to take a leap at this point — I don't know whether Dave is willing to join me — and assume that my experiences have no external cause. All that exists in the universe, all that I have any certain knowledge of, consists of my actual experiences. This isn't some crazy lunatic fantasy but a powerful philosophical position. This is all I know, and all that could ever be. Nothing that is not this could possibly have any impact on me, or have any meaning for me.

At this point, there are some subtle arguments that the solipsist can deploy, along the lines of Kantian and Husserlian phenomenology, to the effect that, in some sense, it is necessary that my existence takes the form of perception of 'objects in space', and that I identify myself as a 'person' in relation to other 'persons'. The details aren't important. What is important is that they allow, or indeed justify, my concept of 'other persons' as an essential part of my experience, characters in the story of my world. If my experience was not like this, if it didn't take this logical form, there wouldn't be anything describable as 'me' or 'I'.

A suitable name for this position (if you are into naming philosophical positions) is transcendental solipsism. The kind of solipsist that Dave is, is a transcendental solipsist.

One very curious feature of transcendental solipsism is that, prima facie, no practical consequences follow from this theory. It's not as if you look at people in a funny way. You deal with them exactly as you would do if you didn't believe in solipsism. You can attend solipsist philosophical conventions, and argue the toss with solipsists and anti-solipsists.

I said 'prima facie', because there is a problem here. You can deal with other persons in just the same way as you would if you weren't a solipsist (or Dave's kind of solipsist). But you don't have to. After all, they are just characters in the story of 'my world'. You can choose to behave ethically, if this helps to keep up the illusion that you are enjoying their 'company', but that's just your choice. On the other hand, it might be more fun if you played games with some of these characters. After all, they are just your barbie dolls and action men. Whatever you do can't be 'wrong'.

Personally, I wouldn't like to be stuck with this view of ethics, which is why I think it is important to find an argument which would be sufficient to refute solipsism. But that's another story.

Geoffrey Klempner

back

(2) Dave asked:

I want to ask you a quick but really weird question. If it makes me happy is being eaten by my cousin Louisa whole and alive ok. I am just asking if something makes people happy is it right even if its wrong such as being eaten whole and alive by my first cousin Louisa. Actually I have a weird fantasy of being eaten by my cousin Louisa so I can be in her belly.

---

Some people are made happy by killing other people. We call these people psychopaths and what they do can never be right. If you have trouble understanding this then think about whether you would like to be murdered without your permission.

There is nothing wrong about fantasy as long as you don't confuse fantasy with reality. I'm sure your cousin doesn't want to eat you alive or dead, so keep your fantasies to yourself. You might also like to think about the meaning of this fantasy, since all fantasies have a meaning.

Fantasising being eaten alive doesn't hurt. In reality it would really, really hurt.

Shaun Williamson

back

(3) Don asked:

Can God and evil coexist?

---

This is a question which, in various forms, arises in philosophy at regular intervals. Probably the first thing that needs to be said about this question is that the questioner (Don), seems to suggest that while there may be a God, that evil, whilst it exists, may not have been part of His/ Her/ Its original design.

Should this be the case, the second thing that needs to be said is that proof of the existence of God has been, and ( since it has more to do with faith that reason.) still remains one with which philosophy has some difficulty.

Let us take, for example, the most commonly posited proofs of God's existence: the cosmological argument, the ontological argument, and the teleological argument, and the arguments these 'proofs'.

The cosmological argument for God's existence is also known as the causal or 'first cause' argument. This takes the view that from experience we know that behind everything that happens we know that there is a cause. If this chain of effects is traced back far enough, the argument goes, it must come to the ultimate or first cause, which is God. If there was no first cause, then there would be no succession of events, therefore, there must be a first cause, and this first cause is God. The argument against this is that there does not necessarily have to be a first cause. That is, can it not be that things do go back indefinitely.

Another version of the cosmological argument is that the universe only exists 'contingently', not necessarily, and therefore must depend on something else, which is God. Since in this argument God is also the ultimate cause, the argument against this is the same as that against the cosmological argument.

Ontology is concerned with the nature of being. The ontological argument is an attempt to prove rationally that God exists: that God must necessarily Be. This argument stems from St Anselm (1033-1109) and takes the view that proof of God's existence can be shown by reason alone. That is, it does not depend on any knowledge about the world (it is a purely a priori demonstration). Anselm asks us to imagine the greatest, most perfect being possible. If the being you imagine possesses everything but existence, it is not the greatest or most perfect being, because, clearly, a being that exists is greater and more perfect than one that does not. Therefore the greatest, most perfect being must exist, and this greatest and most perfect being must be God. That is, God must exist both in reality and as an idea. God, says Anselm, is greater than that which non greater can be conceived.

One of the first and most powerful critics of the ontological argument was a monk called Gaunilo, who was a contemporary of Anselm. Anselm had written that if you could conceive of a perfect being, but could not accept that such a being exists, then you are a fool. In defence of the 'fool' Gaunilo pointed out that if Anselm's argument was correct then it would be possible that all sorts of unreal and imaginary things must exist. If one could imagine a perfect island, said Gaunilo, far beyond anywhere any explorer might venture, that would mean that such an island must exist, for if it did not that would mean that it was not perfect. Anselm's defence that he only claimed that his argument applied to God, since no other concept could be that of a perfect object, is not sustainable.

Teleology means that there is a design or purpose to nature. Also known as the argument from design, the teleological argument falls into two categories: the first holds that the universe is designed by divine intelligence, and that this design is moving towards an ultimate goal; the second teleological argument places emphasis on uniformity and argues that the universe displays design in the sense of order. It also takes the view that the universe contains order which is independent of human minds. If human beings ceased to exist, the cosmos would its merry, ordered way, without us. The teleological argument is that this ordered universe is the work of a supernatural intelligent being, and that this supernatural being is God. One of the most forceful arguments against the teleological argument is Immanuel Kant. According to Kant, God cannot be demonstrated conclusively in a rationalistic way, by merely extending the use of abstract categories which introduce order and design into our experience. There are as many instances of chaos, waste and of useless repetition and multiplication, says Kant as there are for beauty, symmetry and order. The appearance of external design, he asserts, is not conclusive proof of Providence.

Thus we see that, philosophically speaking, the existence of God cannot be taken as a given. And since it cannot be taken as given, we cannot say that God and evil coexist.

However, let us put aside these arguments and see if we can reconcile the view of those of a religious persuasion, that God, who is all knowing, all powerful and all good, can coexist with evil. Whilst people of such conviction take it as a given that such a God exists, they will argue that whereas God created the world, men (I mean, of course, humankind) created evil. From a philosophical point of view this argument is absurd, for what kind of omniscient, omnipotent, and benign entity would create a world in which abuse of children, rape, plunder, thievery, murder, fraud and other such 'evil' acts are perpetrated against innocent victims on a daily basis.

Just as the cosmological argument, the ontological argument, and the teleological argument each have their shortcomings, so too does the Christian argument, which over the millennia has given us a God who has created not only the world, but heaven and hell (with the now defunct purgatory and limbo somewhere in between), is also philosophically unsustainable, and Descartes' argument, which is really a variation of Anselm's ontological argument, is equally rationally invalid. However, whilst the argument for coexistence of God and evil cannot be sustained, what can be said is that Good and Evil , since they are both concepts determined by man, can be seen to coexist. Moreover, since they are ways of describing man's treatment of his neighbour, it must also be said that it is within man's power to ensure that evil does not prevail.

Tony Fahey

back

(4) Sonia asked:

Is phenomenology part of philosophy or something different?

---

The term 'phenomenon' derives from the Greek term 'phainomenon' meaning 'appearance'. The related term 'phenomenal', while it primarily means cognisable by or evidenced only by the senses, is also used, in everyday language in particular, to suggest that a 'thing', 'happening' or 'event' is remarkable. In philosophy a phenomenon is a thing, a quality, a relation, a state of affairs, an event, and so on, as it appears to us — as it is perceived (see The Penguin Dictionary of Philosophy, p.421). Although the term 'phenomena' plays a significant role in Immanuel Kant's philosophy where he contrasts it with 'noumena' (the former being things as they appear to us, whilst the latter is 'things-in-themselves' — see my response to Bernice in Ask a Philosopher 48). In relation to the term 'phenomenology' itself, it is accepted that it was introduced to describe a movement which began with Edmund Husserl his The Logical Investigations (1900) (for more on Phenomenology and Husserl see my response to Jerome in Ask a Philosopher 48).

Tony Fahey

back

(5) Dave asked:

I have not studied philosophy but I seem to keep gravitating to it unwittingly.

I spend a lot of time thinking and I recently discovered Solipsism and thought it fit almost perfectly with my beliefs.

However, upon trying to find out more about it I just found people using it as a device to make ironic jokes. I just seek clarification on what I am and whether I am a type of Solipsist.

People seem to think that Solipsists would not want to congregate because by definition they are denying everyone but themselves and see little value in others.

I believe in quite the opposite. If all other people are fabrications of my mind, I would find great value in meeting with them especially those with similar ideas. This is because I am not consciously creating them and the fact that they are aspects of my mind's creation means that they are aspects of myself and I have created them for a reason.

Have I got Solipsism right or wrong? Or am I specific type of Solipsist?

---

Well you are a particular type of solipsist. You are a gregarious Solipsist.

However you really don't understand Solipsism because in your real life you don't act like a solipsist. You continue to act as though the external world is real. Think about the implications of Solipsism and stop being a philosophical tourist.

Shaun Williamson

back

(6) Derek asked:

Dear Philosopher,

I do not want free will. Is free will forced on me without my choosing??

Is it possible to live a life using only random choices?

Could one live a to old age without using free will?

Did I have free will when I was a baby?

Was I born with free will.

At what age does free will come into play?

Do lunatics have free will. Please say, no!

Tomorrow the psychiatrist, by his own free will, is going to ask me if I'm well enough to leave the asylum. I want all the blame to fall on him.

---

Forget about free will! You are responsible for the things you do unless society judges otherwise.

Babies don't have free will or children below the age of criminal responsibility. However it is sometimes difficult to make clear judgements about where criminal responsibly starts, in individual cases.

Lunatics often have free will although their free will may be impaired in certain situations.

The psychiatrist will judge if you are well enough to leave the asylum and he will be responsible for his judgement.

You need to remember that even if you are insane, you can still be responsible for the things you do. You do not have to do evil, so avoid it and do something else instead. There is nothing wrong with being insane but don't make other people pay for your insanity, that is just selfishness.

Shaun Williamson

back

(7) Emmanuel asked:

Dr I am Emmanuel a student at a university pursuing a Bachelors in Business Administration. I need your assistance on some questions such as: Provide philosophical arguments to the ethical questions which arise when considering modern advertising techniques: 1. What responsibility, if any, does a company have for honestly educating the consumer about its product?

2. Should advertisers be allowed to suggest that a product will make a person more sexy/ interesting/ beautiful/ successful etc?

3. Is it ethical to use celebrities to sell products they probably don’t even use themselves?

4. Is it the buyer’s responsibility to be aware of these strategies and not allow adverts to manipulate their emotions?

---

This question was sent as a personal email rather than submitted to Ask a Philosopher. I'm guessing that Emmanuel found my article Ethics and Advertising. I don't give private advice because that's too close to helping students cheat with their homework. All answers to questions submitted to Ask a Philosopher are published on the internet.

These are very good questions, which you won't find the answers to in my article. I was more concerned to set limits to what ethics can reasonably demand from advertisers, rather than put forward specific principles governing the ethics of advertising. However, it seems to me that the questions Emmanuel raises don't require any special expertise in business ethics. They are a matter of plain common sense.

What responsibility, if any, does a company have for honestly educating the consumer about its product?

Let's imagine a case where you are marketing a very nice product, which has some features not found in any of the competing products in the marketplace. You go to an advertising agency, who discuss your 'unique selling point' (USP), and possible ways of presenting this in TV adverts, billboard advertising etc.

However, you know, and your advertising agency knows that there is a better product available from a rival company. You've done extensive secret testing and their product beats yours every time. Yes, your product has features the rival product doesn't have, but that is more than offset by the fact that these features are mostly eye candy and not very useful. Is it unethical to tell consumers that yours is the best available?

I am told that in Germany it is actually against the law to state in an advert that your product is the best unless you can prove that it is. Elsewhere, such as the UK where the rules are a bit more relaxed, saying that a product is 'the best' isn't considered as potentially misleading information. Whereas if you say that your toilet cleaner kills 99% of germs when it only kills 75% then you are breaking the Trade Descriptions Act.

'We think it's the best,' is a way of saying, 'We believe in our product, we stand behind it.' To me, that is a perfectly reasonable attitude.

Do you have an ethical obligation to tell your potential customers that the rival product is better, according to your own tests? Absolutely not. You are ethically (and in many cases legally) obliged to ensure that your product is fit for purpose, not dangerous to use, and not misleadingly described. On the other hand, a sufficiently resourceful and creative advertising agency can make the most of the fact that you are not the leading brand. 'We're Number Two But We Try Harder,' was the famous Avis advert which won them an increased slice of the car hire market against their leading rivals, Herz.

I would love to see an advert which said, 'Product X is Better But Ours Has More Eye Candy!'

Should advertisers be allowed to suggest that a product will make a person more sexy/ interesting/ beautiful/ successful etc?

My answer to this would be, Yes, if it's true. If the product in question really does make you more sexy, for example, then you have every right to tell consumers that it does.

But how could this be measured? 'In a survey of a randomly chosen sample of consumers, users of laptop A were considered more sexy than users of laptop B.' Well, an advertiser would never say this, just like that. But they would imply it. The finesse here (as I argue in Ethics and Advertising) is to realize that the advertising campaign in itself can give the product the power to make you more feel, or appear sexy. The money invested in the campaign adds to the value of the product, not by making it more useful, but by making the users of the product feel or appear more sexy, or cool, or whatever.

I suspect that behind this question is a puritanical attitude that hates the glitz and the glamour of today's marketplace. A car is just a useful machine from getting you from A to B. A laptop is just a useful device for sending emails and browsing the internet. As if!

I know that there will be some who are unsatisfied with my defence of the glitz and glamour. Do we really want to live in a tinsel world far removed from reality? — How close to reality do you want to be? I don't want my face rubbed in the dirt. Don't take away my dreams, the world can be a hard place. But I understand that there's a happy medium. Use value is an important consideration, of course it is. Just don't get puritanical on me.

Is it ethical to use celebrities to sell products they probably don’t even use themselves?

This is a sneaky question, because of the use of the qualifier 'probably'. We have to look at two different cases:

The first case is where a celebrity states that they use a product, and that they like it and they endorse it. If they are lying, if they don't use the product, then that is unethical, because it is unethical to lie. There's no argument here. However, in the real world things are not quite so black and white. Consider the immensely lucrative field of sports endorsements. A leading tennis player uses Wilson tennis rackets. But this isn't a Wilson that they purchased in a local store. The racket has been finely adjusted and tweaked. To buy something like that in a shop would cost you thousands. But surely you'd have to be an idiot to think that you could win Wimbledon with a racket you got from the local sports shop!

The second case is where celebrities appear in adverts but don't explicitly endorse the product. Rather, the product gains glamour through the association. Here, again, I think that most viewers of the advert are not taken in. Having said that, you have to consider things from the point of view of the celebrity. Would you, a famous film actor for example, appear in an advert for a product that you considered junk, which had the potential to harm your image? It is not unreasonable to infer some degree of endorsement, even if this isn't explicitly stated.

Is it the buyer’s responsibility to be aware of these strategies and not allow adverts to manipulate their emotions?

If you are able to prevent anyone ever manipulating your emotions then you are a better man than me. Of course our emotions get manipulated, and often we willingly allow this to happen. I don't like it when an advert makes me feel bad, yet if it is an advert, say, for the charity NSPCC which campaigns against child abuse then, then I know that I ought to feel bad about the things the adverts depict. On the other hand, if an advert makes me feel good that's a gift for free, and I haven't even bought the product! Before buying it I will consider the practicalities, of course, but in my eyes its value is already enhanced. That's how human emotions work.

Consumers are not puppets, we do succeed in resisting what we see as irresponsible or shameless manipulation of our emotions. It is in the advertiser's own interest not to go too far in this respect, but to remain within the bounds of good taste. Campaigns backfire badly when advertising executives get this wrong.

Yes, emphatically, the buyer has responsibilities. The responsibility doesn't all lie with the seller or advertiser. But there are different cases to consider. If your marketing campaign is aimed at younger persons, especially children, then different rules apply than if it is aimed at adults. It's a matter of common sense.

Geoffrey Klempner

back

(8) Kat asked:

what do we know about the concept of infinity is it beyond humans capacity for understanding?

---

The word literally means 'without end.' This is something we can understand intellectually (that is, abstractly) but cannot imagine (that is visualize concretely). And it is troublesome intellectually, also. It is defined these days as the number of a set in which a proper part is equal to the whole; for example, in the set of the natural numbers, the number of even numbers is equal to the number of the whole set, even though you would think it to be half the set. (The even numbers can be put into one-one correspondence with the natural numbers, and so both sets have the same number of members, namely infinity.) Leibniz was the first to discover this, and thought it a contradiction; Bertrand Russell denied Leibniz and said that it is only an oddity. I personally agree with Leibniz: I think that 'infinity' is a word empty of meaning that we use when we do not know the limit of something. My reason for this is that the concept of infinity seems to be irredeemably paradoxical. For example, Cantor's Paradox requires that the set of all sets has more members than it actually has.

Note that if you deny infinity then there must be a largest number. We have no idea how big it is, or how small, but we can give it a name. Suppose that we call it g. Then any number smaller than g is a genuine number, but any number written so as to be larger than g is not; thus g+1 or 2.g are merely names of non-existent numbers.

Helier Robinson

back

(9) Dave asked:

I have not studied philosophy but I seem to keep gravitating to it unwittingly.

I spend a lot of time thinking and I recently discovered Solipsism and thought it fit almost perfectly with my beliefs.

However, upon trying to find out more about it I just found people using it as a device to make ironic jokes. I just seek clarification on what I am and whether I am a type of Solipsist.

People seem to think that Solipsists would not want to congregate because by definition they are denying everyone but themselves and see little value in others.

I believe in quite the opposite. If all other people are fabrications of my mind, I would find great value in meeting with them especially those with similar ideas. This is because I am not consciously creating them and the fact that they are aspects of my mind's creation means that they are aspects of myself and I have created them for a reason.

Have I got Solipsism right or wrong? Or am I specific type of Solipsist?

---

The best way of understanding solipsism is by means of the concept of an imperceptible. I can define an imperceptible for me, now, as anything that I am not conscious of, now. 'I' and 'now' refer to whoever is considering this and the time they are doing so. Then solipsism is the doctrine that comes from denying the existence of all imperceptibles for me, now. This makes it simple to deduce the precise meaning of solipsism.

Thus I am not conscious of the future or of the past. I may be conscious of expectations and memories, but these are not the same as the future and the past because they can be false. So if there is no future or past there is no time, there is only an eternal now, in which all expectations and memories (all that I am conscious of, now) are false. I experience time passing, but since there is no time this must be an illusion. And all change (all that I am conscious of, now) is an illusion also, sine there can be no change without time.

All beliefs (all that I am conscious of, now) are false because beliefs are perception substitutes so what they refer to is imperceptible for me, now, and so non-existent.

All explanations (all that I am conscious of, now) are false because all explanations are by means of imperceptibles for me, now, and so non-existent. So any attempt at explaining why solipsism is true, or false, fails.

All that I perceive around me is unreal because the real is all that exists independently of perception and all that I perceive now exists only because I perceive it.

If I should be perceiving someone else now, then he/ she has no mind (because I cannot perceive it) and anything that s/ he tells me about themselves is false. If you happen to be alone now, on the other hand, then there are no other people.

So there you are. Do you still want to be a solipsist? It's an all or nothing deal.

On the other hand, if you want to prove solipsism false then there are two way to do it, in principle: one is to demonstrate that solipsism leads to a contradiction, and the other is to prove that at least one imperceptible, for me, now, exists. Neither is easy.

Helier Robinson

back

(10) Dave asked:

I have not studied philosophy but I seem to keep gravitating to it unwittingly.

I spend a lot of time thinking and I recently discovered Solipsism and thought it fit almost perfectly with my beliefs.

However, upon trying to find out more about it I just found people using it as a device to make ironic jokes. I just seek clarification on what I am and whether I am a type of Solipsist.

People seem to think that Solipsists would not want to congregate because by definition they are denying everyone but themselves and see little value in others.

I believe in quite the opposite. If all other people are fabrications of my mind, I would find great value in meeting with them especially those with similar ideas. This is because I am not consciously creating them and the fact that they are aspects of my mind's creation means that they are aspects of myself and I have created them for a reason.

Have I got Solipsism right or wrong? Or am I specific type of Solipsist?

---

I am tempted to say that you are the more interesting type of solipsist, in that some solipsists showing the influence of Kant merely maintain that other persons are physical mechanisms and they themselves are the singular autonomous consciousness, whereas you go further and lay it down that pretty much everything in the world is the fabrication of your mind, ie, no independent mechanism or reality whatsoever. And you can't call that uninteresting. It certainly grabs the attention, and there are unkind people who would say that this is often one of the main purposes of maintaining a philosophical 'belief'.

Actually all solipsists are rather interesting fellows. The usual interest in Solipsism is in:

1. what leads you to find your theory a satisfying explanation of the facts

then, pulling in a slightly different direction,

2. whether you really believe what you say you believe (whether you really find your theory a satisfying explanation of the facts)

and connected with this,

3. whether what you say is even intelligible.

There is a forth point of interest in your case, in that your response to one of the arguments offered on point 2 is rather persuasive. You point out that if you regard other people as figures of your fantasy life, this will lead to a certain kind of interest in them, given that human beings are usually rather interested in their own fantasy lives. Thus, one popular line of attack on the solipsist 'if you really believe that you wouldn't bother talking to me' seems to be refuted, and I feel sure you will appreciate my congratulating you on this refutation, fantasy creation though you say you think me.

However, point 2 still has some life in it because of the obvious connection with point 3. You can't 'really believe' things that are in fact unintelligible even to yourself. So the relevant question is: can solipsism even be made sense of?

The solipsist's general temptation is to respond along these lines:

'What's the difficulty? I claim that only I exist as a conscious being — that's the only test of intelligibility that can matter, given that I am the only intellect'

The problem with this is that you can claim all sorts of things that are in fact unintelligible nonsense, but which you can, with varying degrees of success over varying periods of time, imagine yourself to understand. Surely you have had this sort of experience yourself. In your case, if you are now imagining that your own intellect is God of the entire universe, this will go with the thought that any previous imagining that God was an intelligence omnipotent and omniscient and beyond you was not only in error (as the ordinary atheist might think), but actually the sort of claim which assigned the wrong meaning to terms. You now think that the idea of a power beyond you is not merely wrong but unintelligible, given that You yourself are (you now imagine) the power animating the universe entire. My point is not to agree with your analysis but to point out that even on your analysis it would be possible for you to think you understand claims which are in fact unintelligible. And there is good reason to suppose that this is just what is occurring in this case.

For what, after all, does the word 'fabrication' mean? Obviously, it means making, or something akin to that. There are 'Steel fabricators' meaning, people who make things out of steel. If I make a bed, that's one clear kind of making. Now, if I make up a story and just pretend that it's true, that's another kind of making, more like the sense you invest in 'fabrication'. But it seems to me that we don't know what it would be for me to 'make something up' outside of the ordinary human context in which we are one of a number of thinking persons amidst a reality which has a nature and a presence that is independent of of our imaginations. Really, I am a seven year old boy who will shortly be called to tea and not a cowboy riding out into the west, but in the garden, with my friends, I can fabricate that alternative world. Now, it seems to me that we know what the 'fabrications of my mind' are in that sort of case, that these are the cases by which we learn the meaning of 'fabrication', and that these are the sorts of case in which 'fabrication' has a meaning and is intelligible.

Your use of 'fabrications' is unlike such paradigm cases in every way, since on your use of 'fabrication' there is nothing in your experience that is not to count as your 'fabrication'. This underlines a doubt about whether what you say is at all meaningful. For in using the word precisely so as not to discriminate in your ascription you thereby lose any content to that ascription. If you claim that it is all fabrication, what can 'fabrication' possibly mean? Think, if it were always raining everywhere and throughout all eternity, how could the word 'rain' come to have a meaning?

Given that doubt about whether your claim is intelligible even to yourself, the doubt about whether you really believe what you say you believe returns.

And with that we return to the general question, appropriate for all varieties of solipsism, of why you should find your unintelligible theory a satisfying response to the facts.

I suppose that, apart from the slight satisfaction to be got from maintaining a philosophical position and defeating a range of less effective opponents (and naturally we can discount that!), the thought that I am, despite appearances, really in control of EVERYTHING is weirdly consoling. Whatever happens, I shall have intended it — perhaps without knowing, but all the same:

they are aspects of myself and I have created them for a reason

Well, if this seems to offer consolation, I have to say that any deep consideration of that appearance will return us to the old anxieties pretty quickly, and with knobs on. For now, in place of other persons whose intentions are unknown to me, but which I could in principle come to know something about if I accepted that that they are real, I am now presented with my own unconsciousness which is said to have 'a reason' for everything, but whose reasoning and intentions must be essentially unknowable to me, if the world is to retain the kind of apparent diversity and development that is the essence of life. Turning Freud into an absolute metaphysic of the fantastic, you speak as though you could inquire into your unconscious and consolingly fathom the reason for the creation of X. But again, what is the meaning and home of 'reason'?

I know perfectly well what a 'reason' is in the ordinary context in which I am one of a number of conscious beings facing a world which is independent of our imaginings. I have a reason to pay my taxes in that sort of world. In the world in which the unconscious is not merely an actor in the world but the creator of the world in which it acts, how can we possibly make sense of the idea that it has a 'reason' for creating X? Again, I can have fabricated reasons in my cowboy fabrication just because there is an ordinary world which is the backdrop to that that fabrication, but your totalisation of fabrication denies us the ordinary meaning of words and makes your claims unintelligible. That is, even the consolation you might hope to derive from your unintelligible theory is itself unintelligible. Your unconsciousness could not have reasons for it's creations, and your solipsism replaces the knowable and reassuring reality of other persons with the frighteningly unknowable and irrational unconsciousness. Your claim is unintelligible and your consolations unconsoling.

This would make your contribution inexplicable but for the joy to be derived, and rightly, from intellectual contact with one's independently real peers.

David Robjant

back

(11) Derek asked:

Dear Philosopher,

I do not want free will. Is free will forced on me without my choosing??

Is it possible to live a life using only random choices?

Could one live a to old age without using free will?

Did I have free will when I was a baby?

Was I born with free will.

At what age does free will come into play?

Do lunatics have free will. Please say, no!

Tomorrow the psychiatrist, by his own free will, is going to ask me if I'm well enough to leave the asylum. I want all the blame to fall on him.

---

You try to make free will sound like a faculty: 'using free will' etc. You then wonder whether this might be a faculty you could get along perfectly well without, given that the supposed benefits go along with the heavy weight of responsibility. This goes with an unhelpful Kantian literature that understands our moral responsibility and freedom chiefly in terms of choices (the faculty of choice not existing in the 'natural order'), and you raise some obvious sorts of difficulty with any absolute theory of autonomy along those lines. There are others.

In fact freedom is neither an absolute which we are all granted equally at birth nor a purely legal condition. I suggest you think about your freedom rather differently as something which you might retain in some degree, and which it would advantage you to nurture: you are responsible for that nurturing. This freedom need not simply be a matter of choices, random or otherwise, but of taking responsibility for your vision of the world. Simone Weil said that choices and actions are the pointer of the balance — the thing is to move the weights. The 'choices' will take care of themselves if we attend properly, and see the world as it really is.

Since you have been thinking about your personal situation in the light of philosophical thought on the matter (and if one takes seriously your last paragraph are thus already, one would suspect, a difficult case for the professionals involved with you) I recommend that the pair of you, your psychiatrist also most importantly, read Iris Murdoch's essay 'Vision and Choice in Morality' in the collection 'Existentialists and Mystics'. If at the end of your discussion (seminar?) about that you still want him to take 'the blame', best stay in the asylum. It is indeed rather a marvelous sort of place for anyone who really wishes someone else to take the blame. Shame they can't all fit in.

David Robjant

back

(12) Crystal asked:

Does God know what it's like to be a bat?

On Tues, Oct 5, 2010 at 18:53:14

Marl asked:

My question here is, Can God himself reverse history so I can change something in the past? Also I have some reasons to go against but also to go with the theory that maybe he can reverse history.

A. Reasons against the theory:

1. God makes it clear that we make mistakes and must learn from them.

2. God is not a genie in a magic lamp.

3. God cannot reverse history because he does not have the power.

4. God would not do this for one person and not the whole world.

5. God would not reverse history because it would wind back everything that had happened to everyone else and take their free will away.

6. God does not need to reverse history.

B. Responses to A:

1. We have the right to make heaven on Earth. If all time in history is the same, we can go back to make it as we want to correct our mistakes.

2. Nothing else we know can make us able to change history so God is the only thing that can. We do not need to ask for money because we can get it but [without God] we can't reverse history and change it.

3. If God has no control over reversing history, then he cannot do everything. If God made the universe, he can do what he wants with it, anyway, how big a thing is reversing history? why should reversing history be hard?

4. But if God has done it, nobody will ever know.

5. If you changed something in history, it would not take everybody else's free will away because they could react how they wanted to what had been done.

6. God can do what he likes but if you do need to reverse history, he's the only thing that can.

---

I love Crystal's question, but Marl has to be given credit for attempting an answer — something that we advise anyone submitting questions to Ask a Philosopher to try do do.

Before we get started, there's an excellent discussion of changing the past in Michael Dummett's article 'Bringing About the Past' reprinted in Richard Gale Ed. The Philosophy of Time Macmillan 1968 pp 252-274. Crystal's question is prompted by Thomas Nagel's famous piece, 'What Is It Like to Be a Bat?' Philosophical Review 1974 pp 435-450, reprinted in many anthologies.

I don't recall that Nagel brings up the question whether God can know what it is like to be a bat; he argues that we can't, and therefore a basic assumption of physicalism is brought under strong pressure. Dummett in his long and intricate discussion considers as one of his examples asking God to make something have happened in the past, e.g. praying 'that the announcer has made a mistake in not including my son's name on the list of survivors', something we seem to have no difficulty in imagining, and yet which seems to imply God's power to bring about something in the past.

What is the point of all this? Especially if your an atheist, why get so het up about what God can do or not do? As a principled non-believer, my justification would be that considering what God can know or do, or, better, what any god (with a small 'g') can know or do, we are using a kind of shorthand for considering fundamental questions of epistemology and ontology.

Let's start from that premise. There are very good reasons (in my view) for holding that no god exists. But if he (or she, or it) did, what knowledge or powers could conceivably be attributed to such an entity?

The crux of Nagel's argument in 'What Is It Like to Be a Bat?' is that we can't conceive of what bat awareness or consciousness is like because there is too wide a gulf between human perception (through sight, hearing etc.) and bat perception (through sonar). It is crucial to his argument that the issue we are considering is knowing what something is like. I know what eating pineapple ice cream is like, even though I have never tasted pineapple ice cream, because I have eaten ice cream many times and I have also tasted pineapple. It doesn't require a great mental feat to put the two together. But no amount of mental gymnastics will bring me to the point of appreciating how the world 'appears' from the point of view of a bat. You can make up anything you like (as a novelist writing a story whose main characters are bats would) but in this case it is pure fantasy.

But God (or 'any god') isn't confined to our forms of human perception. He (or she or it etc.) doesn't need eyes to see or ears to hear. Kant in the Critique of Pure Reason speculates that God is not confined to gaining knowledge by subsuming 'intuitions' under 'concepts', has a form of understanding and knowledge which is directly intuitive. He 'knows' each and every thing in its very existential essence. There is no perceptual gap, as is the case for created beings such as ourselves.

Consider this analogy: in order to determine the current state of my Apple Power Macintosh G3 (soon to be replaced, I hope, by a G5), I need to look at the screen. Whatever is going on inside the grey box, information written to RAM or onto disk, or the instructions being processed by the CPU, is something to which I can have no direct access. There is software (the MacsBug debugger, as my G3 crashes rather often) which can give some information about this, and display the information on the screen. But would it be possible (leaving aside the question of the immense amount of time needed, given that thousands of instructions are being executed every second) to display all the information?

There does seem a logical problem here, which one can illustrate with the somewhat simpler example of HTML code. One of my pages, Today's Note consists of a representation of a Unix screen, running the text editor Pico. This is in fact the method I use to update this page. I don't upload the page using an FTP program. I write directly onto it using my text editor. On your monitor you will see representations of the HTML code which has been used to create this effect. What you don't see is the extra code which is needed to represent the code. (You can check this for yourself by going to 'View → Page Source'.) In principle, it would be impossible to show everything using HTML. I could add even more extra code to represent the 'extra code' but then this new extra code would not appear on the screen.

All human knowledge is in a sense 'representative' knowledge. There is a space, a gap, between knower and known. Even when we have so-called 'direct' knowledge through action (the sense of touch, or awareness of making an effort, or the proprioceptive feedback which enables us to know how our limbs are moving) this knowledge is immediate and non-representative only when described from the point of view of the agent. I may know that I am moving my fingers as I type this, but what is going on beneath the skin, in my muscles and sinews, is out of my immediate ken. Even if I lacked all senses other than the sense of touch (as in the Helen Keller story) I still need language and concepts, to represent the knowledge gained through 'direct' action.

This leads to a rather simple (possibly too simple) characterization of the difference between human knowledge and God's knowledge. God doesn't know the world and His creatures by means of representations (bringing intuitions under concepts). He knows them directly, as they are, for the simple reason that He is their creator and sustainer for every moment that they continue in existence.

It follows that there isn't a synapse or a cell or a molecule in the body of a flying bat that that God doesn't 'know'. Nor is God confined to only knowing about the world on the very lowest level. He can abstract. He can consider the bat as a collection of molecules, or cells, or organs, or as a discrete entity in relation to its environment. But I don't think that this is enough for knowing what it is like to be a bat. God knows too much — not least, that he is God.

In Western philosophy, Spinoza is the philosopher who has come closest to solving this conundrum. The bat is just a 'mode' of God. It no independent existence from God, the only true 'substance'. God 'knows' himself as the bat, but the bat, from its point of view doesn't know this and necessarily cannot know this (otherwise it would be God). Spinoza, through the medium of his finely wrought philosophy, knows and yet doesn't know that he is God.

For making this claim, Spinoza incurred the charge of pantheism, and his insistence on arguing the case at every opportunity got him excommunicated from the Jewish community. But we're not concerned with the finer points of theology. Our question is what any god could know. The question of what kind of god would be worthy of adoration or worship is something which does not the least bit interest me.

I said that this was the best case for explaining the possibility of God's knowledge of what it is like to be a bat. But I don't think it is good enough. It depends, ultimately, on a fudge. If God 'knows' everything only in Spinoza's sense, because God just is everything, then I have lost my grasp of what 'know' means. Even if we assume that God's knowledge is 'intuitive' (in Kant's sense) and non-representational, we still need conceptual room for God as a self-conscious entity, which Spinoza does not seem to provide.

In other words, if God just is the universe, seen under the aspect of unity, then the term 'God' reduces to a mere 'something concerning which nothing can be said'. To talk about God is just to talk about the universe.

Let's now look at the case of changing the past. Dummett notes in his article that the possibility of praying to God in order to change the past is unacceptable to orthodox Jewish theology. I didn't know that. You learn something new every day. But are the orthodox Rabbis right?

I remember the day when I received my A-level examination results, which would determine whether or not I went to university (I'm keeping this simple for the sake of the example). Supposing I did believe in God. I have no difficulty in imagining that I might pray to God that I passed, and that God heard me and granted my request. Even though I had in fact failed! I'm sure many people have done this.

One easy get-out would be to say that God exists outside of time, and therefore knows my prayer 'before' I made it. So no funny backwards causation was involved. But this solves the problem of how God can change the past by effectively destroying the difference between past, present and future. I no longer know what 'cause' means in this scenario. So let's stick with a world in time, and a God who is also in time and not looking down on the universe sub specie aeternitatis.

What did God have to do, in the exam results example? The slip of paper inside the envelope with 'failed' written on it had to change to 'passed'. All the steps which led up to the typing of that word (and my marks or grades) had to alter also, right back to the point where some examiner was marking my script, and indeed before that to when I was actually sitting in the exam making a total mess my (say) Pure and Applied Mathematics paper. But how could I have written a good paper, if I hadn't revised? How could I have revised, when most of my time was spent going to parties and getting stoned?

I can totally see how Marl would object that this is a serious encroachment into my free will, not to mention the other difficulties.

But we're looking for a logical objection, if there is any. The question of whether or not you would like the kind of God who would do this, or consider them 'worthy of adoration or worship' is neither here nor there.

Dummett's case rests largely on a single observation: If I know that it was the case that P (e.g. that it was the case that I failed) then I can't, by any action including prayer intend to bring it about that not-P (e.g. to bring it about that I passed). It's a logical contradiction. In fact, it is more or less the same logical contradiction which vitiates time travel (as traditionally conceived — see my Afterword to David Gerrold's The Man Who Folded Himself).

Okay, but let's look at this from another angle. Where is the past? The past is gone, its water under the bridge. (John Donne: 'Tell me where all past yeares are, Or who cleft the divel's foote'.) But where we are, always, is on the bridge of the present. We can remember, infer from evidence, hypothesize more or less accurately about 'what happened', but always with the proviso that we could be wrong, that is to say, there is no logical contradiction in the idea that the most vivid memory of something that happened only ten seconds ago could be false.

As long as I don't know what's inside the envelope, the question whether I passed or failed is up for grabs, at least by me (as indeed is the question whether my maths teacher knows, whether the secretary at the examining board knows etc.). It's not that difficult to think in this way of the past as something malleable, or flexible, not fixed and final. From my perspective, at least, there is just me and God and the world which God partially reveals to me and partially conceals. That's all 'the world' is — from my perspective.

This sounds rather like anti-realism about the past (which Dummett defends in another article, 'On the Reality of the Past' in Truth and Other Enigmas Duckworth 1978 pp 358-374). But there is just one rather important difference: it is essential, in order to make sense of exam results example that one is only considering my perspective (or our perspective if a group of persons is praying together) not human knowledge generally, not 'what can be known' of the past, but only what I (or you and I) can know.

As Marl has seen, if God changes the past, as such, He changes it for everyone, not just for me. Whereas, if all God changes is 'my past' or 'our past' then we have left behind the very idea that there is such a thing as the past. This is just a little too close to Solipsism for comfort. (If you can be a quasi-solipsist about 'we'.)

Ultimately, as I have indicated, the question whether God can know what it is like to be a bat, or whether God can change the past isn't about theology. It is about the nature of the universe and our knowledge of it. But one thing it does show is that as a theologian you can't make assumptions about God's powers without at the same time making fairly hefty commitments to your epistemology and metaphysics.

Geoffrey Klempner

back

(13) Paula asked:

Why thought the social democrats that it was better with an evolutionary development than a revolution?

---

In his Evolutionary Socialism, [1895] Eduard Berstein of the Social Democratic Party of Germany, argued against Revolutionary change. His arguments against this were that economic crisis were not as apocalyptic as Karl Marx had argued they would become. Credit and state intervention would avert or at least, ameliorate crisis. Secondly, contrary to Marx's prediction about the pauperisation of the proletariat and the corresponding polarisation of impoverished proletarian masses against a minority capitalist class, there was no such occurrences. In fact, the petty bourgeoisie and medium sized capitalists were growing. Thirdly, the intervention of the State would pass into Socialism. Economic interests were being held to account and regulated by the state. Having a majority in parliament would allow the representatives of the working class to pass laws democratising the means of production. hence change in a socialistic direction would occur peacefully without any need for violent, revolutionary change. The grounds and justification for revolution were not present and were increasingly unlikely to be. Reform was the realistic option.

After the Bolshevik Revolution of 1917 the Third Communist International was set up. Initially at least, it advocated the Leninist concepts of 'vanguard' Communist Party organised around Democratic Centralism which would be dedicated to world proletarian revolution. On the other side, those socialists opposed to the Communist model variously organised separate Internationals. These accepted Labour, Social Democratic and Socialist Parties who nominally accepted democracy, pluralism and the Parliamentary road to Socialism.

From a Communist perspective, there is a distinction between themselves as Marxists intent on overthrowing Capitalism and 'Social Democrats'- whether of Labour or Socialist Parties. The latter are accused by the former of perpetuating Capitalism and in no way, working for its supersession by reformist measures. Some will even argue that social democrats have become the gatekeepers of neo-liberalist capitalism.

Philosophically, this raises the issues of the nature of Marx's and other economic 'laws'. can there be laws in social science as there [arguably] are in Natural science or, are there just tendencies? Is Marxism a 'science' as in the description of it as 'scientific socialism'? If the confirmation of Marx's theory occurred only once in 1917 as Trotskyist Marxists maintain [People's Democracies of Eastern Europe, China, Korea, Cuba, Vietnam all being Stalinist or Nationalist caricatures], the 'science' doesn't seem accurate in its analyses, predictive capacity and confirmation.

Martin Jenkins

back

(14) Andy asked:

Why teach philosophy to teenagers?

---

Teenagers minds expand as rapidly as their bodies and along with this goes an increased capacity to understand abstract ideas. Many philosophical problems about the nature of reality occur to teenagers naturally.

While I feel that it would be good if they were given the opportunity to study philosophy at school, I don't feel that it should be taught as a formal examination type subject. The peculiar nature of philosophy i.e. the fact that there are no agreed philosophical truths, makes it difficult to devise a set syllabus for a philosophy course at school level.

Of course given the competitive nature of our education system and the time pressure on school studies it is unlikely that time will be found for optional courses which might only appeal to a small group of students.

Shaun Williamson

back

(15) Kyle asked:

Hello,

I came across this joke presented in a book I was reading for my philosophy of literature class that goes as follows:

What is a goy?

A goy is a person who is a girl if examined at any time up to and including t, and a boy if examined any time after t.

I was wondering if you could explain its meaning, so I can then decide if I think it is funny.

---

This made me laugh, just a bit, although it could be argued that the 'joke' is on the borderline of being politically incorrect. It has an obvious, and very straightforward explanation (if you know enough about philosophy, and also are familiar with the term 'goy'). But that doesn't fully explain the humour, which incorporates an element of tragedy, like other jokes of this genre. It is also a joke about knowledge and ignorance, which explains why philosophers would find it particularly amusing.

It is a 'Jewish' joke, in the sense that it is about Jews, although I would hesitate to describe it as a 'Jewish joke', i.e. a joke which you need to be Jewish in order to fully appreciate its deeper resonances.

In common usage, the term 'goy' refers indiscriminately to non-Jews, although the Hebrew term originally referred to 'nation', including the nation of Israel. It is not a term that one would usually use in polite conversation — 'gentile' is better — although lumping everyone together who is not of the Jewish people is itself somewhat problematic.

'What became of your friend Sue?' 'She married a goy.' In this exchange, Sue's decision to 'marry out' (marry a non-Jew) is deprecated. In social mythology, one has lower expectations of the behaviour of goyim (plural of goy) than one has of one's fellow Jews. Goyim are more likely to get drunk at weddings and other social occasions. If you do business with goyim, then you are taking a greater risk than you would otherwise.

On the other hand, unlike 'n**ger', used of people of colour, 'goy' is not intended to demean or insult. You wouldn't say to someone, 'You goy!'. Or, if you did, you would expect to get a blank stare. It is a term a Jewish person would use only in conversation with other Jews. To my ear, there is a conceit (which is surely false) that non-Jews wouldn't understand what one was talking about.

As a Jew, it is a term I never use. Period. (In what follows, whenever the term 'goy' or 'goyim' appears, I am talking about the term rather than using it.) The term 'goy' is 'not one of my words', to echo what Oscar Wilde said about the word 'obscenity' (Glass House Philosopher, notebook 2, p.65). You can't (or, at least can't any more), separate it's descriptive and evaluative content.

Now to the joke:

In his 'new riddle of induction', Nelson Goodman defines 'grue' as 'An object which is green at any time up to t, or blue at any time after t.' The term 't' just refers to any arbitrary time. The riddle or paradox arises because examining emeralds and finding that they are green prima facie gives the same degree of support to the generalization, 'All emeralds are green,' as examining emeralds and finding that they are grue gives to the generalization, 'All emeralds are grue.' Yet we know that this cannot be so. 'All emeralds are grue' can only be true if something happens at t which makes all emeralds in the universe change colour. Hence the paradox.

I'm not going to discuss the paradox here, as appreciating the finer points of the debate over Goodman's new riddle of induction doesn't contribute in any way to understanding the joke.

Why is it funny? You can run Goodman's Paradox with 'boy' and 'girl' instead of 'green' and 'blue'. Or just about any pair of descriptive terms. However, you would only coin the term 'goy' if you were ignorant of the term's common meaning. Or deliberately intending to cause amusement.

One imagines a socially cloistered philosophy professor po-facedly expounding Goodman's paradox, while the audience titters. But there are other aspects too. I've already indicated how use of the term 'goy' implies uncomplimentary beliefs about goyim. To some persons, the idea of sex change is intrinsically funny. To imply that a goy changes sex at some arbitrary time, is just one more uncomplimentary belief. Not only do goyim get drunk at weddings but they also change sex when you least expect it!

Finally, there is an aspect that this joke shares with other jokes which imply, or rely on, prejudice. I said that 'goy' is 'not one of my words'. But a joke allows you to use words that are not words you would use outside the context of a joke. After all, one is only joking. The thought here is that, somehow, the joke you are telling is at the expense of those who do use the term. A kind of mock-chastisement, where you get to indulge in the very thing that you are purportedly criticizing.

Behind the humour there is tragedy, in the very fact that one perceives a need to compensate for being an outsider by putting all those who are not of one's own tribal grouping 'on the outside'. One wouldn't make this joke about Muslims and 'infidels' or about Christians and 'heretics'. It just wouldn't be funny. The infidel, the heretic is the exception, the deviation which must be corrected. While it is perfectly possible to convert to Judaism, as a rule Jews do not proselytize. The point of being Jewish would disappear if everyone became Jewish. As long as there are Jews, there must always be goyim.

Geoffrey Klempner

back

(16) Kat asked:

What do we know about the concept of infinity? Is it beyond humans capacity for understanding?

---

As for your first question (what we know about the concept of infinity), there is far too much material available to summarize here. The concept is employed in numerous branches of mathematics and physics. The best I can do I refer you to any of several encyclopaedias such as Wikipedia, Reference.com, Encyclopedia4u.com, Britannica.com, among many others. Try Googling 'infinity encyclopedia'.

As to whether humans have the 'capacity for understanding' the concept of infinity that depends on just what you mean by a 'capacity for understanding'.

If, by that phrase, you mean a capacity to employ the label 'infinity' (and its cognates) in a manner consistent with the successful communication of the concept to others, I think it is rather obvious that do have the capacity because we have the word in common usage, and generally agree on when and where it is appropriately employed.

But if, by that phrase, you mean some deeper grasp of the meaning of the concept then you are going to have to provide some description of just what it is you do mean by that 'deeper grasp of the meaning of the concept'.

There certainly are those philosophers who maintain that we can understand the concept of 'infinity' only as a procedure that never ends, but cannot comprehend the concept in some other, 'deeper' way. But I do not agree with this position. I do not accept that there is any 'deeper' meaning to a word/ label than is indicated by success in communicating the concept to others by using the label 'properly'.

Stuart Burns

back

(17) Andy asked:

Why teach philosophy to teenagers?

---

Philosophy, as a field of study, involves a particular manner of thinking. A manner of thinking that searches behind the superficial, to find the underlying premises that lie hidden behind the superficial, and to explore the logical consequences of those premises. Most people, most of the time, manage without any awareness of the premises hidden beneath their superficial assertion. And for most people, investigating the premises hidden beneath our superficial assertions reveals some degree of logical inconsistency in our beliefs. Since long experience has taught Mankind that it is usually better to base ones decisions on true beliefs rather than false beliefs, Philosophy serves the purpose of finding where we have conflicting beliefs (where at least one of them must be false).

In some fields of enquiry, it is science that investigates our beliefs and determines which are more likely true and which are more likely false. But in other fields of enquiry, especially those where the meaning of our words is critical, it is philosophy that performs this service.

To teach philosophy, is therefore to teach this particular manner of thinking, and to teach the habit of being conscious (when appropriate) of the premises underlying our assertions and the significance of our choices of words. (Many is the argument that would simply dissolve if people were more careful of the words they use to describe their position. Many arguments result from people simply 'talking past each other' each disputant not understanding what the other is saying because they misconstrue the intended meaning of the words being employed.)

Unfortunately for most teenagers, the community of professional Philosophers seems to be of the opinion that the way to teach Philosophy is to teach History. So teenagers are faced with such irrelevant questions as 'What was Aristotles argument in the Phaedo for the Immortality of the Soul'. As if that matters even the tiniest little bit to the learning of how to think philosophically. As if anything that Aristotle might have said 2500 years ago is going to be a meaningful contribution to an understanding of issues relevant to todays modern teenager.

The history of philosophical discourse is important if you want to come away with a sound understanding of how various philosophical disagreements have evolved over the millennia. But the history of philosophical discourse is totally irrelevant if what you want is an appreciation of the current state of thinking on currently relevant topics. (Although, obviously, if you are interested enough in some issue to investigate further, you will develop that historical knowledge along the way.)

So teaching philosophy to teenagers is supposed to be teaching them a way of thinking about what it is that they and others really mean when they make use of verbal arguments to make a point. Teenagers are notorious for their penchant of resorting to 'might makes right' solutions to disagreements. Perhaps if they were taught some skills and mental habits that enable a better use of words and logical argument to resolve disputes, there would be less of a tendency to resort to ill-considered emotional outbursts.

That is the hope, at any rate.

Stuart Burns

back

(18) Farnaz asked:

Hello!

Is it possible that a person can be in different places at the same time?

Kind regards,

Farnaz

---

The ability to be (or, alternatively, 'appear' — that's one of the questions we have to decide) at two different places at the same time is known as bilocation. This cropped up in a story I once wrote:

The giant out door auditorium was filled to capacity. Overhead, robot drinks and ice cream vendors darted about amongst the hovering TV cameras. On the podium a man in a blue tunic had just started to speak. Distorted images of his friendly features loomed on scores of giant video screens.

'...Some of you might remember me from the old television series, Star Trek. For the benefit of those who haven't seen any of the episodes, my name is Captain Kirk. And yes, I am a real Star Ship Captain. The series is substantially based on true events, though of course we had to simplify things to fit each story into a fifty minute slot. Followers of the series will be glad to hear that all your favourite characters are here. You might even get the chance to meet some of them. You will all have met Mr Spock of course...'

Captain Kirk's words were almost drowned in wild cheering. He paused to salute his Science Officer, who was seated behind the podium. Spock stood up briefly to take a stiff bow.

'...Like the rest of us here today, Spock has para-psychic powers. In his case it is the relatively rare but extremely useful gift of bilocation, the ability to appear in several different places at one and the same time. Some of the Catholic Saints were able to bilocate, I believe.

'Well that is by the way. The main question that seems to be on everyone's lips is, 'Where is Heaven?' That's a little difficult to explain. But if you give me a few minutes, I'll do my best to fill you in. Mr Spock has written a useful little book for those of you who've done a bit of maths and physics, complete with equations and flow diagrams, but I shall just try to keep things simple.'

Kirk paused for a few moments to collect his thoughts. The famous smile beamed down from scores of video screens. One thing you knew for sure. The maths and physics weren't above his head.

The Possible World Machine Unit 12: Space Hopper Downloadable from philosophypathways.com/download.html

In the story, a group of persecuted telepaths escape to an alternative universe existing in a different space from our actual universe (but not in a different time). The idea was to test Kant's claim that there necessarily can exist only one space using a thought experiment which doesn't rely, as Anthony Quinton's does, on a subject falling asleep and appearing to 'dream' of a life which is no less coherent than his 'waking' experience (Anthony Quinton 'Spaces and Times' Philosophy 37, pp 130-147 1962).

In my tale, there is said to be a fully scientific explanation of how there came to be two spaces. It's the 'simplest explanation' of the data. (There was a 'cataclysmic explosion', and a fragment of space 'split off' from the universe to form a space of its own.) There's no reason, in principle, why experimental evidence couldn't lead us to conclude that Kant was wrong about there being one space, just as Quantum Mechanics has shown that he was wrong about the a priori truth of determinism.

I don't know if that's acceptable as a response to Kant. It amounts to little more than stating the very thing that Kant denies. Unlike the case of QM, we don't have the least bit of scientific evidence for multiple spaces (ignoring things like the many-worlds interpretation of QM which seems to be a different thing entirely). It is pure speculation about what we would conclude if such alleged evidence turned up. In this case, we really need to consider the argument Kant gives (in the first part of Critique of Pure Reason), and whether the argument is in fact logically sound. (Many commentators agree — e.g. P.F. Strawson in Bounds of Sense [1966] — that Kant's argument for the necessity of determinism is over-ambitious: the most he can claim is that experience should exhibit sufficient regularity to enable us to make reliable predictions.)

If there were overwhelming logical objections to the very idea of a person being located at different places at the same time, then no amount of empirical 'evidence' would be sufficient to persuade us otherwise. We would have no choice but to offer an alternative explanation. However, it is worth pointing out, that at least some of the things said about the bilocating Catholic Saints can be understood in the weaker sense of the individual in question appearing to observers at a place (as a realistic apparition) as opposed to actually being there in the flesh.

But is genuine bilocation — actually being in two different places at the same time — such a nonsensical idea?

Before we can even consider that question, we have to address the prior question of what it is to be located at a space. For trees and rocks, or planets and stars, there is a simple and conclusive test. Spatial position is one of the criteria (or, indeed, the main criterion) for identity. If an object, say, a paperweight is seen at two places at the same time, then we have two exactly similar paperweights, not one paperweight. If I scratch the paperweight on my desk, and an identical scratch mark simultaneously appears on the matching paperweight on my coffee table, or if smashing one paperweight with a hammer immediately results in the destruction of the other, then the conclusion would be that some kind of unknown causal influence has occurred, not that this is proof that the 'two' paperweights were in fact one and the same object or entity.

Of course we are free to call the matching paperweights by a single name, describe it as an extended 'object'. This might even be a useful thing to do. (We might want to distinguish superficially matching paperweights from genuine pairs which exhibit this remarkable property.)

With persons, on the other hand, an entirely new factor is brought into play. Persons have a point of view. If I have a twin on Twin Earth — or for that matter Doncaster — even if the same things appear to happen to my twin as happen to me and at the very same time, we are not the same person. I have my point of view and my twin has his point of view.

The problem with this intuition, as Daniel Dennett entertainingly shows in his piece 'Where Am I?' (originally in Brainstorms 1978, reproduced in Dennett and Hofstadter Eds. The Mind's I 1981 pp 217-229) is that if we assume the materialist hypothesis that the mind is a kind of program which 'runs' on the brain, then there are various science fiction scenarios where we simply don't know how to answer the question, 'where I am'.

I'm not going to pursue Dennett's idea of brains being simulated by computer programs. If the self is a program, and a program is (as it necessarily must be) a kind of thing, a set of instructions which can be written in any language, realized on any suitable hardware (or 'wetware'), if that's all it is, then it's hardly surprising that you can't 'find' the location of the self, or even decide whether you are dealing with one self or more than one self. The 'GK program' would be like Windows XP.

So I'm going to assume we don't know whether or not you could 'write' the program for GK. In other words, I'm assuming that you can be a materialist without being committed to Dennett's version of materialism.

The US flying drone which destroyed the alleged Al-Qaeda cell last Saturday was 'flown' by a GI operative sitting comfortably at a laptop. In World War II, the Japanese kamikaze gave their lives to achieve the same objective. But what exactly is the difference between being there, at the moment the high explosive detonates, and not being there?

Let's notch this up a bit. Instead of a metal and plastic flying drone, let's have a fully functioning robot which reproduces my bodily movements via a broadband radio connection. To make this really effective, I need the ability to feel when my robot is damaged. This is a very expensive piece of equipment, what better way to protect it than to give the operator a suitable jab of pain? As my robot engages in battle (presumably with other robots) I have the most vivid sense of 'being there'. Only, I am not there. It's just an illusion, isn't it?

Let's say that as a result of carelessness or lack of sufficient fighting skill, my robot gets destroyed, and I feel the pain of its destruction. After receiving a severe dressing down from my commanding officer, I'm issued with another robot with the warning not to let this happen again, or else. This time, I will not only feel the pain, I will receive the same injuries, in the same body parts that my robot receives. If it dies, then I die.

Remember that my robot doesn't have a brain, or a computer simulating my thought processes. It is just a sophisticated drone. And yet, in this extreme case, wouldn't it be correct to say that where my robot is — where the action is happening — there I am also?

If I put my hand into a fire, then the fire doesn't only burn my hand, it burns me. Whereas a drone under my control is just like an extended artificial hand. What puts me there, in the flames, is nothing other than the fact that it is my life that is at stake. I am where my vulnerable parts are.

It helps to have 'eyes' and 'ears' where your vulnerable parts are located otherwise you will injure yourself too easily. But merely having eyes and ears at a location (as in the case of the Al-Qaeda drone) isn't sufficient for being there.

If Dennett is right about the possibility of a brain program, then human beings do not, in principle, have any vulnerable parts. As noted above, the self program can be endlessly reproduced. On the other hand, if Dennett is wrong, and brain function cannot be duplicated in a program (more precisely, by a Turing Machine) then the living human body which I call 'mine', or at least that part of me (say the brain) whose destruction would lead to my death, is necessarily where I am.

I have noted that 'genuine' bilocation must be more than just appearing in a place. The appearance must correspond to reality. As we have also seen, it must be more than my manipulating a robot or simulacrum of me at that place, because the destruction of the robot or simulacrum does not entail my destruction. To be in a place is to risk death at that place. If I can do this in two or more places simultaneously, then I can bilocate, but not otherwise.

Geoffrey Klempner

back

(19) Derek asked:

Dear Philosopher,

I do not want free will. Is free will forced on me without my choosing?? Is it possible to live a life using only random choices? Could one live to old age without using free will? Did I have free will when I was a baby? Was I born with free will. At what age does free will come into play? Do lunatics have free will. Please say, no!

Tomorrow the psychiatrist, by his own free will, is going to ask me if I'm well enough to leave the asylum. I want all the blame to fall on him.

---

The first question to answer is 'Just what exactly is Free Will??' There are numerous descriptions available as to just what is meant by the concept labelled 'Free Will'. It is, unfortunately, a poorly understood concept hence the plethora of alternate descriptions of it. The most important description is that 'Free Will' is that capacity of the human mind that is necessary for the assignment of moral responsibility. If your action, choice, decision is not made 'Freely', then you are not to beheld morally responsible for the consequences.

In a negative sense, 'Free Will' is contrasted with 'Determinism'. Determinism is the doctrine that all events have a cause. Your will, choice, decision, action is just another event that has a cause like all other events. Given a complete and sufficiently accurate description of the state of the Universe at any particular instant, and given the laws of nature, it should be possible in principle to forecast the entire evolution of the Universe from that instant forward. Hence, goes the Determinist argument, all of your choices, decisions, actions are entirely predictable and pre-determined given the state of the Universe at some earlier time. 'Free Will', in the negative sense, is just the denial of Determinism.

More positively, 'Free Will' is described as the ability to have chosen, decided, willed, or acted otherwise than you did, given the circumstances that applied at the time. This is also sometimes called the 'Principle of Alternate Possibilities' if you had available to you at the time real alternative possibilities, then your will, choice, decision, or action was 'Free'. (Of course, there is considerable debate amongst philosophers about just what constitutes 'real alternative possibilities'. But I will leave that debate for your own further investigation, if you should be interested.)

Now, as to your other questions:

(a) If human beings do have Free Will, then you also have Free Will — whether you like it or not, and whether you choose to exploit that capability or not. As to when, in the course of a persons development, Free Will becomes active is an interesting question that I have never seen addressed before. But as a guess, I would hazard that a persons ability to exercise their Free Will becomes effective at the age where they are capable of making choices for reasons. So (again as a guess) a baby would not have Free Will. But the age at which Free Will comes into play would be the subject of much debate I would think. Researchers in childhood development are finding that younger and younger babies appear to be making reasoned choices albeit within limits.

(b) Do lunatics have Free Will?? I suppose that would depend on just what particular type of lunacy is involved. But I would guess that in most cases, lunatics would have to be considered to have Free Will. I would think that it is not an absence of an ability to make reasoned choices that marks a 'lunatic' (although, I suppose in some cases it might). But rather a suite of beliefs that are not consistent with our best understanding of reality. Hence a 'lunatic' would be able to make 'Free' choices and decisions, but would make them on the basis of a false set of beliefs about reality. [Hmm?? Interesting side thought does this position make 'lunatics' out of Republicans, Democrats, those who believe in God, or Astrology?? Or in UFOs, Little Green Men, Alien Abductions, ESP, Psychic Readings, etc, etc, etc.?? Based on popular rhetoric, this seems to be the correct interpretation. But I suppose your psychiatrist would disagree? You might ask him/ her. It would make for an interesting discussion.]

(c) Could one live to old age without using free will? Yes, certainly. Many people do either our of choice or from an absence of alternative. If the exercise of Free Will is marked by the making of reasoned choices, decisions, actions then a life distinguished by an absence of Free Will would be a life marked by a lack of any reasoned choices, decisions, actions. Consider a Monk, Nun, or Maximum Security Prisoner. Their life is distinguished by a notable lack of any opportunity for decision or choice making. Most of what they think or do is prescribed by the community within which they live. If 'Free Will' is marked by the presence of 'real alternative possibilities' from which you must choose or between which you must decide, then consider a life lead in abject poverty, where there is never any choice each succeeding moment being dictated by the only available means of ensuring continued survival.

Unfortunately for you and your wishes with regards to the blame falling on the psychiatrist, what the psychiatrist will be looking for (among other things) is an indication that you are willing and eager to take responsibility for your own life and your own choices. In other words, what the psychiatrist will be looking for is some indication that you not only have Free Will but insist on exercising it. Since you seem (hopefully tongue in cheek) to prefer renouncing Free Will, and would seem to prefer to live your life according to the dictates of others (or the flip of a coin) rather than the dictates of your own reasoned choices, I would suggest that you are not yet ready to be released from the asylum.

Stuart Burns

back

(20) Liam asked:

Why is it human nature to be so curious?

---

It is part of human nature that we are provided with neither sharp teeth, dangerous claws, fleetness of foot, strength of muscles, wings, wheels, nor fins. Instead, we have evolved the ability to learn, and plan, and employ foresight to provision our table and avoid our enemies. But our intellectual capabilities to plan and anticipate would be worth nothing if we did not learn how the world works. You cannot make a plan unless you can accurately anticipate how the world is going to react to the choices you make. And you cannot learn to accurately anticipate how the world is going to react unless you play the game of constantly tweaking it to see what happens. This is called curiosity.

The reason that cats, for example, are so successful in finding their dinner and avoiding the dog, is that they are constantly exploring their territory and constantly investigating the things in it. Curiosity may wind up killing the odd cat or two. But the next generation of cats is birthed by cats who were successfully curious.

Likewise for human beings. The last millennium of history with its technological threats is a recent anomaly. For the vast majority of human history, the next generation was populated by those who were more successfully curious about how the world works. So curiosity is an evolved part of our nature.

Stuart Burns

back

(21) Dave asked:

I have not studied philosophy but I seem to keep gravitating to it unwittingly.

I spend a lot of time thinking and I recently discovered Solipsism and thought it fit almost perfectly with my beliefs.

However, upon trying to find out more about it I just found people using it as a device to make ironic jokes. I just seek clarification on what I am and whether I am a type of Solipsist.

People seem to think that Solipsists would not want to congregate because by definition they are denying everyone but themselves and see little value in others.

I believe in quite the opposite. If all other people are fabrications of my mind, I would find great value in meeting with them especially those with similar ideas. This is because I am not consciously creating them and the fact that they are aspects of my mind's creation means that they are aspects of myself and I have created them for a reason.

Have I got Solipsism right or wrong? Or am I specific type of Solipsist?

---

Dave, as your question infers, the literal meaning of Solipsism is 'only-oneselfism'. Essentially, at least as I understand it, there are two approaches to solipsism. The first is the view that nothing exists outside one's own mind, or that nothing can be known to exist outside one's own mind. The second, and one that appears in Kant's Critique of Practical Reason, is the more traditional view that it means egoism: the view that nothing is to be valued except one's own interests and pleasures. It seems to me that your own concept manages to go somewhere to meet both concepts. That is, you are prepared to accept the view that nothing, or at least no other person exists outside your own mind (that 'all other people are fabrications of my own mind'), whilst at the same time holding the view that other people, as 'fabrications of your own mind' exist primarily for you to satisfy your ego and to discover hidden aspects of yourself.

Throughout the history of philosophy, various thinkers from Parmenides to Berkeley have attempted, without success, at least to the majority of philosophers, to convince us that the only real world is that which exists within the mind. Notwithstanding the fact that solipsism makes for interesting philosophical discussion, I must say that I find any approach that reduces other human beings to 'fabrications' or figments of one's own mind extremely uncomfortable, if not dangerous. Whilst one can site various examples of the results of treating others merely as objects through which one can satisfy one's own interests, or indulge in one's owns pleasures, suffice it to say that history (even the most recent history) is replete with examples of the danger of this way of thinking. Moreover, for one to accept, on the one hand, that those who commit such atrocities as child abuse, rape, murder and other crimes against humanity, or on the other hand, that those who are victims of such atrocities, are merely creations of the mind suggests to me that one with such a fertile imagination really needs to move from this realm of fantasy and reconnect with the real world.

In parenthesis, to get a sense of solipsism at its most extreme, can I recommend Patrick McCabe's book The Butcher Boy (the film version is also worth watching for the same reason). Although a work of fiction, McCabe's depiction of how a young boy, from a dysfunctional background, becomes so absorbed in his own fabricated world that he believes he is free to treat those who occupy this world as he sees fit, is a fine example of solipsism at its most powerful and its most dangerous.

In Philosophy, perhaps one of the most interesting arguments against solipsism is given inn Sartre's Being and Nothingness where he describes a situation where a man, moved by jealousy or curiosity, looks through a keyhole and listens at a door. Whilst doing so he becomes so absorbed in what he is doing that he has no room in his consciousness for anything else that may be happening around him. He sees the door, the keyhole, and all his surroundings as objects, either as obstacles or aids, to his mission. Suddenly, he hears a sound in the passageway and he becomes aware that he is being observed. In that instance, his existence is reconstituted in a wholly different way — he sees himself in a new light. He realizes that he exists, not in a solipsist vacuum, but as an object to the 'other' who is observing him — as a 'Peeping Tom'. Hence, in this instance, his concept of his self is initiated, not from within the machinations of his own mind, but by his awareness that how he is perceived by an 'other' who is not a figment of his imagination. (see Vico's Road to Postmodernism, p. 122, by Tony Fahey) In Science, one of the proofs that there is a world outside the mind is given in 1800 by William Herschel when he accidently detected, what would later become known as infrared radiation, light that was invisible to the human eye — and by consequence, light that was not a 'fabrication' of the mind. (see Quantum, p.4, by Manjit Kumar)

So Dave, I suppose, in answer to your question, I would say that whilst you have perhaps a somewhat unusual concept of solipsism, it is not necessarily a wrong one. I would warn, though, that for the ego or self to see others merely as objects through which one can dialogue in order to discover hidden or uncovered aspects of one's self or selves is to engage in a practice that is fraught with danger.

Tony Fahey

back

(22) John asked:

Can non-existence be supported, if once existence has occurred? And by a good estimation, existence has always been. Is there such a thing as non-existence?

---

John's question is prompted by a legitimate sense of bewilderment at the very idea that there might have been nothing at all — that what might have been true but is not in fact true of reality or how things are is that nothing existed: that a state of sheer non-existence obtained.

I am assuming that we can still talk of 'how things are' or 'reality', as indeed one must in order to make any sense of John's question. As Wittgenstein states in the first two propositions of his Tractatus Logico-Philosophicus, 'The world is all that is the case. The world is the totality of facts, not of things.'

In the case where a state of 'sheer non-existence' obtains, it is a fact that there are no things. Nothing exists, in the sense of 'no things'. Whether or not you like this way of talking (one of my philosophy lecturers once commented on an essay I'd shown him, 'I'd much rather defend the necessary existence of God than the necessary existence of facts!') I can't think of any other way to approach this.

It seems, prima facie, to make sense to say that if the Big Bang hadn't banged, if there hadn't been anything to go bang, then there would have been no universe, no space or time, no galaxies, stars or planets. No us. But what about the laws of nature? Are we supposing they to be non-existent too? Or can laws still exist — which dictated what would happen if something else happened — even if nothing physically existed?

I'm not sure that this isn't a question for a physicist rather than a philosopher. Suppose that according to law L, there is a finite but very small probability of a grain of matter coming into existence (something to 'go bang'). However, if an event is only probable, or improbable (it makes no difference), then it is still logically possible that it never in fact occurs. It is logically possible, according to the laws of thermodynamics, that all the air will spontaneously rush out of this room (all the air molecules as a result of their trillions of collisions will just happen to all be pointing in the direction of the door). But it will never happen.

But can it conceivably be a 'law' that matter can spontaneously just appear, out of nowhere? I think John would say that an assumption we are making — which he will not allow — is that such a law, or indeed any laws, could 'exist' in the absence of any physical matter, or energy fields, or anything similar.

One could object that John isn't entitled to say, 'And by a good estimation, existence has always been.' How does he know? We know (or rather, according to the best cosmological theory currently available — if that counts as knowledge) that the Big Bang happened so-and-so many billions of years ago (13.7 to 14 billion was the answer I found when I searched on Google). Except that according to physics that's when time started too, so in that sense it is true that existence 'has always been'.

Richard Swinburne in his book Space and Time (Macmillan 2nd. edn. 1981) argues that we are not logically compelled to identify time with 'physical time', in which case it would not be self-contradictory to state that there was a time before physical time existed. At least (so he argues) an empty time is a more coherent idea than an empty space. In one way this satisfies our naive intuitions (that there is no 'first moment' of time) but creates another problem, which vexed the Presocratic philosopher Parmenides: 'And what need would have driven it [sc. the One] later rather than earlier, beginning from nothing, to grow?' (DK Fr. 8).

In other words, given a prior state of non-existence, supposing such could occur, you cannot get existence, because as each moment of eternal time ticks by, there is no reason why anything should happen at this particular time, rather than some other time. Maybe that's true, if you grant Leibniz's Principle of Sufficient Reason. Or maybe Swinburne is wrong about time and the physicists (e.g. Hawking) are right, and time and existence necessarily go together because time begins with the Big Bang.

That still does not get us any nearer answering John's question. Given that something does exist (and maybe always existed, but this isn't crucial) how can this make way for a state of sheer non-existence? Once you have something how can there ever be nothing? Or, perhaps more to the point, how can we so much as conceive of a state of non-existence?

I haven't got an answer to that. In my book Naive Metaphysics I argue that when we consider the question, 'why anything exists' there are two problems not one: why there is a universe, and why there is I. Those are baffling questions. But isn't it also true that it is as hard to conceive of my non-existence as it is to conceive of the non-existence of the universe?

You can't argue from difficulty in conceiving alone. Maybe some persons are better at forming conceptions than others. I tell you that I can't conceive of my non-existence and you reply that you have no difficulty in conceiving of your non-existence. End of discussion. Exactly the same applies, if the topic of our conversation is the non-existence of the universe.

Geoffrey Klempner

back

(23) Margery asked:

Can you suggest existential contemporary literature that will make me laugh?

---

Margery the problem is that it all depends on your sense of humour. Novels that have made me laugh and which have existentialist type themes include 'Hunger' by Knut Hamsun, although to fully appreciate this one you should not eat anything for a week before reading it.

'Seize the Day' by Saul Bellow also makes me laugh as does 'The Trial' by Franz Kafka. Most of Iris Murdoch's novels make me laugh especially 'The Sea, the Sea'.

Shaun Williamson

back

(24) Jonathan asked:

It would be impossible to perceive a nonexistent universe. There would be nobody there to perceive it! For equally obvious reasons it would be impossible to perceive a universe in which there is no life or subjective experience. That is simply because all perceptions require a subject and an object.

So the only available perception is one of ourselves (or something with subjective experience) in a universe (or in relation to any object which is there to be perceived).

Doesn't this prove that we must exist? It seems to prove that no other observation of the universe is possible except one in which we exist. It says nothing of the specific conditions of our existence, only that it is logically impossible to perceive our own nonexistence.

QUESTION: Does the fact that it is logically impossible to perceive our own nonexistence also mean that it is logically necessary that the universe exists?

WHY I ASK: If the answer is yes, doesn't this eliminate the need for metaphysics in general? If something is truly logically necessary then it requires no further account than a proof of its logical necessity. This deflates 'Why is there something rather than nothing?' as a springboard for gods and other metaphysical objects. It deflates every variation of the cosmological argument and the recent 'fine tuning' arguments. It (the anthropic principle, look it up) is a subtle idea with devastating consequences. It has never been a part of humankind's mental equipment even that of intellectuals, but its influence is gradually being felt in philosophy and other disciplines.

---

Almost all your reasoning here is spurious or wrong. You need to be clear about certain things such as.

1. It is not necessary that the universe exists.

2. It is not necessary that human beings exist. 3. It is not necessary that you or I exist.

Also the anthropic principle is a misunderstood and trivial proposition that has no important consequences for science or philosophy.

I suggest you should study logic and philosophy more before drawing such sweeping conclusions about things. The idea that we must exist is an absurd idea. Think about it.

Finally the anthropic principle was devised by human intellectuals and is a part of their mental equipment. It is an idea that we have been familiar with for a long time but it doesn't have the devastating consequences that you think it has.

All that the anthropic principle says is that if the universe were not suitable for human existence then humans wouldn't exist. Of course, at some time in the future, humans like dinosaurs might become extinct and then we will have to conclude that the universe wasn't really so suitable for human existence after all.

Shaun Williamson

back

(25) Ottis asked:

What is something a philosopher might think about?

---

I normally avoid this kind of vague question which invites you to write about just about anything you like. Unfortunately, this week the selection of questions submitted to Ask a Philosopher was so uninspiring that I'm actually glad to find a question which invites me to write about anything I like.

What do philosophers think about? The standard response is to toss out a few examples, showing why these illustrate the nature of philosophical problems or questions, or the techniques or methods used by the philosopher in answering them. But that's boring. In any case, it wouldn't be true. At least, not about me. I don't spend my time thinking about 'the problems of philosophy'. My students do this, and in my letters to my students I criticize their efforts and offer guidance. Of course, this involves thinking about those questions, but that's become second nature. I don't think about it, I just do it.

What I think about, what occupies every spare minute of my day, wherever I happen to be and whatever I happen to be doing, is something entirely different, something you won't find in the average introduction to philosophy. If you asked me, what it is that 'makes me a philosopher' it would be this, and not what I do in my day job.

What I think about is The Question. There is only one. Douglas Adams in The Hitchhiker's Guide to the Galaxy humorously puts the case that the joke is on philosophers for being so dumb as to believe that there could be just one question. ('The answer is 42. What did you expect?!') But on a closer reading, I see something that isn't a joke at all.

The Question isn't, 'The Universe, Life and Everything'. That's just a dummy phrase, a placeholder, for whatever-it-is that philosophers think about, ultimately think about, as part of the very essence of being a philosopher. The meaning of it all. Except, you can't just talk about 'meaning' as if you knew what that meant, or even 'it all' as if you what is involved in a universal generalization which isn't restricted to any particular subject matter or domain, just 'everything'.

I wrote my book Naive Metaphysics about this. Well, not exactly this, but about one component of the Question, namely What It Is that we ask the Question about. My answer is that there are two things, not one. 'A theory of subjective and objective worlds'. However you pose it, you have to ask the Question twice.

In the very first paragraph of Chapter One (which I'm sorry to say many readers don't get past) I give an argument for the impossibility of asking the Question, or at least, of interpreting it as a 'why' question:

Logically, the world ought not to exist. — Brute fact is an affront to human reason, which always seeks the sufficient ground for every contingent given. Yet neither can reason be persuaded to accept (pace Spinoza, Leibniz) that it is necessary that our little planet Earth should have come into being. Or that I should be writing this. Or that the Holocaust happened. Fortunately, as finite beings, we are never brought to the point of having to make that impossible choice. The chain from consequences to grounds is (as Kant observed) never-ending. However, a contradiction that will never have to be faced is still a contradiction. Between logical contingency and logical necessity there is no third modality: either our world is or is not the only logically possible world. If it cannot be either, then it cannot be.

Logically, the world ought not to exist! 'Well, so much logic,' you might be thinking. But you'd be dead wrong. There is no thinking without logic, only the babbling of an infant.

What is all this about? Ignore the allusions to Spinoza, Leibniz, Kant. That's for students of philosophy to dabble into. (Clue: the Kantian reference is to his 'Antinomies of Pure Reason' in the Critique of Pure Reason.)

What I'm saying is that the world, or universe, cannot be 'necessary' and it cannot be 'an accident'. Either alternative is unacceptable, an affront to sense and logic. I could just have easily argued that the universe cannot be 'rational' and it cannot be 'irrational'; that it cannot be 'meaningful' and it cannot be 'meaningless'; that it cannot 'have a purpose' and it cannot 'lack a purpose'.

Investigate this for yourself. 99.99 per cent of all the discussions of the Question in all its guises, on all the internet forums, take the form of rebounding endlessly between the two alternatives. Much of the history of philosophy is the same. Every 'new' answer, every tweak or dodge, is just a variation on what went before. You go for one side, or you go for the other, or you invent some clever reason for taking a position somewhere in the middle (philosophers love to 'reconcile contradictions').

I don't have an answer, or even a clue how to go forward. If you think you'd like to discover that the universe had a purpose, say, just imagine how you'd feel if you didn't like that purpose. Tough titty. The purpose is the purpose, willy nilly, and what you think is totally irrelevant. Or imagine that you discover, finally, that the universe really doesn't have a purpose. You can do whatever you like, life has whatever purpose you give it. Really? Is that your final answer? Are you happy with that?

I could roll of a list of philosophers who claimed to be happy with that answer (starting with Nietzsche, maybe) but I strongly suspect — no, I know that they only said that, only defended that view because of the unacceptability of the alternative. Logic again. If it can't be P then it must be not-P.

Let's just assume that both alternatives are false. (The example is about purpose but pick your favourite concept.) How could that be possible? Here are some more or less familiar strategies:

1. Overcoming the contradiction in a synthesis. Hegel is simply following old logic here, not inventing a new kind of logic (so-called 'dialectical logic'). If the contradiction is 'overcome', that is to say, if both alternatives are false, then that merely shows that the 'contradiction' so-called wasn't really a contradiction. The contradictories were merely contraries. In other words, Hegel is merely generalizing on Kant's strategy in the Antinomies.

2. Denying duality. The 'alternatives' so-called aren't really alternatives at all. They are merely different aspects of one and the same thing. Spinoza argued, against Descartes, that there is no 'mental substance' interacting with 'material substance', rather, there is only God and his mental and material 'modes'. So with regard to purpose you might try to have your cake and eat it: the universe is purposefully purposeless, or maybe purposelessly purposeful. There's no choice to make because it's all one and the same in the end. But that's just a transparent dodge. There's no problem if the contradictories apply in different 'senses' or 'respects', or from different 'points of view'.

3. Denying logic. I like this one because I tried it in Naive Metaphysics. So what if the objective standpoint and the subjective standpoint contradict one another? In that case, here's one example (there could be more!) of a contradiction which isn't false. Problem is, I've been stuck at this point for the last 20 or so years. Meanwhile, my mental capacity isn't going to undergo any miraculous increase. I've got to make do with what I've got, which doesn't seem a lot these days.

Why bother? Why keep worrying at a question (sorry, Question) which you know, or at least are pretty sure you will never be able to answer? I don't have the answer to that one. To hold the entire universe 'in question' seems liberating, in a strange sort of way. Like Neo in The Matrix, I know there is 'something wrong with the world'. — What a great movie that would be if only the answer wasn't so banal!

Geoffrey Klempner

back

(26) Maricris asked:

How did philosophy help you?

---

Philosophy didn't help me. Philosophy is not a form of therapy. It isn't there to help people. I became interested in philosophy when I was very young because philosophical questions seemed to me to be the most difficult questions that humans had thought of. I wanted to know what the answers to these questions were. So I devoted most of my life to the study of philosophy. This has kept me poor but I don't regret it.

Only study philosophy if you are interested in philosophy. Don't do it for the money (there isn't any) or as a form of therapy (philosophy won't make you happy or unhappy).

Philosophy is the ruthless pursuit of the truth. That is all it is.

Shaun Williamson

back

(27) Jonathan asked:

It would be impossible to perceive a nonexistent universe. There would be nobody there to perceive it! For equally obvious reasons it would be impossible to perceive a universe in which there is no life or subjective experience. That is simply because all perceptions require a subject and an object.

So the only available perception is one of ourselves (or something with subjective experience) in a universe (or in relation to any object which is there to be perceived).

Doesn't this prove that we must exist? It seems to prove that no other observation of the universe is possible except one in which we exist. It says nothing of the specific conditions of our existence, only that it is logically impossible to perceive our own nonexistence.

QUESTION: Does the fact that it is logically impossible to perceive our own nonexistence also mean that it is logically necessary that the universe exists?

WHY I ASK: If the answer is yes, doesn't this eliminate the need for metaphysics in general? If something is truly logically necessary then it requires no further account than a proof of its logical necessity. This deflates 'Why is there something rather than nothing?' as a springboard for gods and other metaphysical objects. It deflates every variation of the cosmological argument and the recent 'fine tuning' arguments.

It (the anthropic principle, look it up) is a subtle idea with devastating consequences. It has never been a part of humankind's mental equipment even that of intellectuals, but its influence is gradually being felt in philosophy and other disciplines.

---

In arguing as you do you should say 'I' rather than 'we' because at this point minds other than your own belong in your category of 'other metaphysical objects.' Secondly, your logic would be more clear if you avoided so many negatives: instead of 'It is logically impossible to perceive my own nonexistence' you could say 'It is logically necessary that I exist because I perceive it.'

The answer to your question is no, unless you define the Universe as all that you are conscious of now: that is, the content of solipsism. Whatever you are conscious of now exists necessarily; it has to exist in order for you to be conscious of it. But that does not prove the existence of anything else whatever. For common sense it is 'obvious' that what we perceive around us is reality, that it continues to exist when unperceived; but philosophically speaking this is mere opinion, unproved dogma. Similarly for the existence of other minds. By all means stick with common sense if you want to, but you will not be being philosophical if you do. If you can prove these common sense beliefs then both will be true, but so far no one has done it.

The anthropic principle has two interpretations. One is trite: because I exist the universe I live in is one that can support life. The other is an explanation of cosmic coincidences. There are various parameters in quantum physics and cosmology that are seeming free to take on any of a large range of values: parameters such as the masses of various particles and the relative strengths of the four forces. It so happens that if any of them were a little bit different from what they are then life in the Universe would be impossible. There would be no stars, or stars would exist but be too short lived or too dim for life to evolve. Lee Smolin ('The Life of the Cosmos' Oxford University Press, 1997) has calculated that the probability of all these parameters randomly having the right values is about one in ten to the 229th power. Hence the name cosmic coincidences. On explanation of them is that the Universe consists of very many mini universes, in each of which all these parameters take on random values; if there are enough of them then sooner or later there will be one which has them just right for life to occur. We are a part of life, therefore we live in such a universe. This is the anthropic explanation of the cosmic coincidences. It is not a good explanation because it grossly abuses the principle called Occam's Razor: do not multiply theoretical entities beyond the need to explain the empirical phenomena.

Helier Robinson

back

(28) Amanda asked:

Which ethical theory is most 'natural'? That is, which theory requires the least alteration of our natural inclinations in order for us to live ethically?

---

Amanda, this is quite a broad issue and one which I to me that we should consider a number of those ethical theories that might approach the criterion set out by you in your question. Whilst, there are more ethical theories than those included in this response, I have attempted to deal primarily with those which I believe have held most influence on our lives — of course others may not agree, or find other theories more appropriate, but that is what philosophical discourse is all about. In this response I intend to consider, in particular, utilitarianism, Kant's duty theory, Aristotle's, and more recently, G.J.Warnock's, virtue ethics, and Hobbes' 'social contract'.

Utilitarianism is a moral theory that argues that an action is right if and only if it conforms to the principle of utility. Founded during the Victorian era, its founder, Jeremy Bentham, came to believe that there was a need for society to rely on reason rather than metaphysics. The central tenet of utilitarianism is what is called the 'Greatest Happiness Principle'. Because the human beings are rational self-interested creatures, says Bentham, they seek to maximize their pleasure and minimize their pain. Thus, a morally correct action is one which results in the greatest possible pleasure within a given set of circumstances.

Set against utilitarianism is deontology. Deontologists are concerned with the concept of duty. That is, they are concerned with fulfilling (what they believe is) their moral duty — whether or not it makes people happy. In short, deontologists hold that right actions are defined by duty. Once we know what it is that we are duty bound to do morally, then we can carry out this 'natural' right action regardless of the consequences. What matters, they argue, is that we do what is right what is right, and what is right is that which conforms with moral law. One of the leading exponents of this theory is Immanuel Kant. For Kant, right actions are those which are done purely and simply from a sense of duty and not by following impulses, inclinations, or adherence to the 'Greatest Happiness Principle'. Human beings, says Kant, are, by nature, rational beings and as such need have a rational basis to their lives: they need to know what make right actions right. Ethics, he maintains, is concerned with identifying moral imperatives, and providing rational explanations as to why we should obey them.

Central to Kant's duty ethics is the view that right actions are those actions that are not instigated by impulses or desires, but by practical reason. Right action is right only if it is undertaken for the sake of fulfilling one's duty, and fulfilling one's duty means acting in accordance with certain moral laws or 'imperatives'. To help us identify those laws which are morally binding Kant has provided us with the ultimate calculus: the 'categorical imperative' which states 'Act only in accordance with the maxim through which you can at the same time will that it become a universal law'. To the categorical imperative, Kant offers a codicil which relates specifically to human will; 'so act that you use humanity, whether in your own person or in the person of another, always at the same time as an end, never merely as a means'. Thus far we have seen that both utilitarianism and deontology hold different views in regard to what is most natural ethical theory. For utilitarians it is the 'Greatest Happiness Principle', whilst for Kant it is the 'Categorical Imperative'. Now it is time to consider what has become known as the Virtue Theory.

It is in his Nicomachean Ethics that Aristotle sets out his ethical theory (later to become known as 'Virtue Theory'): his concept of what it is, for human beings, to live well. For Aristotle, the end or final cause of human existence is eudaimonia. Eudaimonia is most commonly translated as 'happiness', but a more accurate translation is 'flourishing'. Aristotle believed that the desire to live a fulfilled life is part of what it is to be human. A eudaimon life is a life that is successful. It is important to relies that what Aristotle means by happiness/ flourishing has nothing to do with physical pleasure, but is an activity of the mind/ soul in accordance with virtue.(NB for the ancient Greeks, soul was a synonym of mind). For Aristotle there are two parts to the mind/ soul: the intellectual and the emotional. Correspondingly, there are two types of virtue: intellectual and moral. Moreover, virtue, whether intellectual or moral, is a disposition (a natural inclination) of the mind/ soul, which finds its expression in voluntary action -that is, it is consciously chosen. Moral virtue is expressed in the choice of pursuit of a middle course between excessive and deficient emotion, and exaggerated or inadequate action: this is the famous doctrine of the Golden Mean, which holds that each virtue stands somewhere between two opposing vices.

Thus, courage or fortitude is a mean between cowardice and rashness; and temperance is the mean between licentiousness or profligacy and insensibility. Justice, or 'fairness',the most important virtue of the moral virtues, is also concerned with a mean in the sense that it aims at each person getting neither more nor less than his or her due. However, it is not like other virtues, flanked by opposing vices since any departure from the just mean, on either side, involves simply injustice. Moral virtue prevents disordered emotion from leading to inappropriate action. What decides, in any situation, what is appropriate action and the correct amount of feeling, is the intellectual virtue of prudence or practical wisdom (phronesis): this is the virtue of that part of reason that is concerned with action. The virtue of the speculative part of the reaction is learning, or philosophic wisdom (Sophia): this virtue finds its most sublime manifestations in more or less solitary contemplation (theoria). Supreme happiness, according to Aristotle, would consist in a life of philosophical contemplation. However, whilst this would be the ultimate in human fulfillment, it is also a life that is beyond the realization of mere mortals. The best we can aspire to is the kind of happiness that can be found in a life of political activity and public magnificence in accordance with moral values.

Following Aristotle, Warnock, in his The Object of Morality (1971), also takes the view that there is no universal criterion by which our actions can be classified as either right or wrong. 'It is clear', says Warnock, 'that moral principles may point in opposite directions, and I can pronounce no ground on which one could pronounce in general which... is to predominate another'. Virtue theorists, then, accept that human beings must be governed by moral principles, what they do not accept is the view that these principles are bound by moral absolutes or 'imperatives'. For virtue theorists, it is not whether one answers to the demands of the categorical imperative, nor is it one's determination to opt for pleasure over pain that determines whether or not one is ethical, rather it is one's natural disposition to lead a virtuous life. Whilst deontology, or 'duty ethics,' can be said to hold considerable merit, in that it advocates that human beings should be treated as ends in themselves rather than means to ends, I would argue that it fails to meet the criterion set out in your question in that it looks on people, not as sentient beings, but as duty automatons.

Moreover, an ethical theory, such as Utilitarianism, that advocates that the happiness of the majority takes precedence over the minority cannot be counted as a reliable ethical model. Thus, of the three theories discussed so far, it seems to me, by virtue of its rejection of closure in relation to what it is that determines right action, and its view that it is one's natural disposition to seek to lead a life of excellence, Virtue Theory is the closest we have come to identifying an ethical theory that requires the least alteration to allow us to lead an ethical life. However, before drawing this discussion to an end there are two more ethical theories that deserve consideration. These are the ethical theory set out by Thomas Hobbes in his magnum opus, Leviathan.

Central to Hobbes' thesis is the view that 'the human desire for peace is the motive that moves humans from the natural condition to civil society. Hobbes operates on the premise that all men are equal. Nature has created men so equal in faculties of body and mind that, when all is reckoned together, the difference between man and man is so slight that no man can claim superiority over another. Moreover, 'as to the strength of body, the weakest has strength enough to kill the strongest, either by secret machination, or by confederacy with others, that are the in the same danger with himself.' In this, hypothetical, state of equality there is no sovereign power, each man believing himself to be equal, is motivated purely by self-interest to preserve his own life and liberty, and also to acquire power over others. These desires are motivated by an innate impulse, or 'inclination', for self-preservation. In this 'natural condition of humans', says Hobbes, there arises conflict: a state of 'war of all against all' and the life of man would be 'solitary, poor, nasty, brutish and short'. Left to its own devices, says Hobbes, mankind would eventually destroy itself. The struggle for power, for no other reason than self-preservation and self-interest, defines the natural condition of humans. Altruism, compassion and self-sacrifice find no place in Hobbes' scheme of things. Hobbes wants to show that people in such a state of anarchy will naturally want to move to change their circumstances. To show how they might go about this he introduces four interrelated concepts: the state of nature, right of nature, law of nature, and the 'social contract'. He presents us with the concept of a situation in which man is in a state of nature: a state without a sovereign or common power to enforce rules and to restrain behavior is naturally a state that any reasonable man would want to move to a situation in which he could live in peace.

The ambition of Hobbes' Leviathan, then, is to show how man can move from a state of nature to a state of peace. For Hobbes, man's self-interest does not have to lead to a life of misery. By giving up certain rights, and adhering to certain rules, the individual can move to a situation of greater comfort, harmony and security. This can be achieved, says Hobbes, by surrendering certain rights to a sovereign power, who, in turn, would guarantee the conditions necessary to live a 'harmonious and commodious' life. This could be achieved by entering into a social contract. If people were prepared to give up their individual rights to a powerful authority which could force them to keep their promises and covenants, then a peaceful, civil society could be formed. By virtue of the social contract a new political power could be created. Thus established people would now have an obligation and duty to obey the sovereign. In such a state, the sovereign would have to be empowered to use as much force as necessary to retain order, any failure to do so would entitle the subjects to remove the sovereign power and return to a state of anarchy. However, given that it is Hobbes' view that people are naturally predisposed to prefer a state of peace to a state of nature — a state of 'war of all against all' — it is to be felt that they would soon choose to elect a new sovereign and return to their harmonious and commodious lives.

Considering each of the above theories, you will note that the predominant theme is that ethical lives must be governed by reason. The implication of this is that, whilst we may be predisposed to seeking to live 'good lives', unless our emotions and appetites are brought under control by reason, our 'natural inclinations' could, and in many cases would, inhibit this ambition. Thus, whilst it grieves me to admit it [my own preference, since it involves not only the obligation to live ethically, but also ensures that one's relationships with others is also governed by ethical principles, comprises of a mixture of Kant's categorical imperative and the virtue theory], it seems to me, since it involves, in any situation, simply adding up the positives on one side and comparing them to the negatives on the other side — utilitarians call the 'felicific calculus', that the theory that would require least alteration to live our lives ethically is utilitarianism. But I would add that an ethical theory in which the consequences for those of the minority who fall on the wrong side of the 'happiness sums' can be disastrous (just look at the current French government's policy to its Romany community, for example) is not one that I could embrace.

What do you think Amanda?

Tony Fahey

back

(29) Aviral asked:

Is it not that thinking deeply is in some way equivalent to thinking negatively. If it is not so why were some great preachers so moved by seeing the dark side of life. Was their thinking not negative initially. Was it awareness or a sort of fear to face the same things later in their life. Was it the fear that made them discard this materialistic world.

---

I'm not sure I fully understand Aviral's question. 'A sort of fear to face the same things later in their life' — what does that mean? But you could read the question as a response to an earlier post of mine, Human test tubes:

Actually, I rather like looking into the abyss. When I cast my eyes around this dingy world, the tawdry sideshows that human beings call 'culture', the abyss is the only thing with any real depth. Anxiety is the only real human emotion. (I think Freud said that.) But philosophy isn't just about plumbing the dizzy depths. It's about remembering and focusing. About being present. It can sometimes be a pleasurable activity (especially if you have a taste for Schadenfreude) but it's not something you do for pleasure.

'Tawdry sideshows' is a phrase suggested by a topic I was looking at around about this time last year, Poshlust and moral incontinence. The problem with diagnosing poshlust is that such diagnoses so easily become examples of the very thing they deprecate.

I could talk about Freud. Or a thinker I know a bit more about (because it's my field) Schopenhauer. Now there's a gloomy philosopher for you. But, actually, Schopenhauer is the best example of a Western philosopher I can think of for whom philosophy is a kind of eschatology, not in the Christianizing sense but much closer to Buddhism and the idea that this world is an illusion created by our slavery to desire. All one needs to do, in order to end the suffering, is to free oneself of desire. For Schopenhauer, the magic key is art. For Buddhism, there's the practice of meditation.

I'm putting this in a deliberately crass way, because it really doesn't interest me. The best example of this line of thought is something I remember from Colin Wilson's new Preface, written many years later, to his first book The Outsider (1956). As a young man, determined to lose your virginity, nothing seems more wonderful and desirable than the sexual act. Finally, you succeed in getting some hapless girl into bed. And afterwards you lie there thinking, 'Is that it?'

The existential sentiment is expressed perfectly in the Lieber and Stoller song, Is That All There Is?. (Google the title to find the lyrics.) That thought is the beginning of philosophy.

I've seen things you people wouldn't believe. Attack ships on fire off the shoulder of Orion. I watched C-beams glitter in the dark near the Tannhauser gate. All those moments will be lost in time, like tears in rain. Time to die.

— Dying soliloquy of the android Roy Batty (Rutger Hauer) in Blade Runner

In the end, everything goes. But isn't that a good thing? Isn't that what a good Buddhist wants? To achieve a state which is not death, but the nearest damn thing to it. Nietzsche and Freud saw through that.

This leaves me feeling a bit sick. That's not my shade of black. My black is much closer to a Nietzschean black. But even Nietzsche is ultimately too religious for my taste. I'm trying to think of a philosopher who epitomizes the contemptuous rejection of 'all things white and wonderful'. Can't think of any. Most of the thinkers who venture to the dark side, in whatever way they do it, secretly hanker after the colour white.

Maybe Stirner. Why do anarchists like the colour black? Does anyone know? (This article has some suggestions.)

I once wrote something about philosophy and the colour black in my Glass House Philosopher notebook (Notebook 1, page 51), accompanied by the soundtrack to Escape From New York. Was I just being cute? Or did I see something — out of the corner of my eye? The hero of the hour is Descartes in his Meditations on First Philosophy: 'Black is the prevailing colour of this all-time classic.'

You can discount Descartes' ostensive religious aim in proving the existence of the soul. Locked up in his stove room, his aim is to find one nugget of absolute, indisputable truth. Even an all-powerful evil demon couldn't persuade me of my own non-existence.

It's the problem, but also the cure. It's what gets you into the existential predicament (see my post on Camus on absurdity). But it's also the solution, because if you make sufficient effort in directing your gaze inwards, you see through it. If nothing has meaning then everything has meaning. 'The world of the happy man is a different one from that of the unhappy man' (Ludwig Wittgenstein Tractatus Logico-Philosophicus 6.43). But it isn't. Not really. There is only the world ('all that is the case'). The rest (which 'cannot be said') is just your mood. Snap out of it!

And when you have snapped out of it, you will find that nothing remains to be done except to pursue the question of what is. Everything else is a distraction (which is why one needs the dark).

I very rarely read fiction. The last novel I remember reading from cover to cover was a pulp karate novel, a clichéd story about a young man, lost and confused, who comes upon a group of dedicated karate students. He joins them and learns the true meaning of pain. Forget your Bruce Lees. The aim of karate is the brutal refashioning of the human body into a blunt weapon, which you learn to wield with exquisite grace and speed. You smash your forearm to pulp until it becomes sufficiently hardened to block any blow without flinching.

(A neighbour who once did karate — I think he was the one who lent me the book — told me that karate practitioners have terrible problems with piles. All those body hardening exercises are at the expense of weakening the pelvic floor. They should go to maternity classes.)

Think of philosophy as karate for the mind. You learn to surmount every kind of mental pain. The mind is refashioned into a weapon whose only purpose is seeking out the truth, aletheia. Emotions, moods, desires are all distractions. Philosophers like the dark side because they love to tempt themselves, test themselves. I understand this gung ho attitude but at the same time something about it also repels me. Perhaps for that reason I will never be a true philosopher.

As Dirty Harry (Clint Eastwood) once famously said, 'A man's got to know his limitations.' Do you feel lucky, punk?

Geoffrey Klempner

back

(30) Penny asked:

Wondering about the morality of accepting gambling revenue to help the victims of gambling

---

As far as we know gambling has existed for as long as human society has existed as has the making and consumption of alcohol. Attempts to ban these things are always likely to be unsuccessful.

There is nothing immoral about gambling in itself. This doesn't mean that we should all approve of the gambling industry. We need to ensure that this industry is regulated by law and that people are not allowed to gamble recklessly. Some people become addicted to gambling just as some people become addicted to alcohol and some people become addicted to eating.

There is no harm in accepting money from the gambling industry to help people who are unable to control their gambling addiction as long as you are not required to publicly endorse the gambling industry in return.

However if you believe that all gambling is wrong then it would be wrong for you to accept this money.

Shaun Williamson

back

(31) Alan asked:

Do we have free will?

Just before I typed this, I toyed with the idea of not submitting the question. Then I decided to submit the question although it seems that I could effortlessly have decided not to submit the question. This seems to be a process of me doing the deciding. Is it not meaningless to say that it is just an illusion that I have a voluntary choice of whether or not to submit this question when it feels so real that I have this choice?

---

I have a similar story to Alan's. It being a Friday afternoon, with the prospect of a weekend of relaxation and enjoyment ahead of me, there are several items on my list of things that I had meant to get done this week — which have still not been done — and my Tentative Answer is just one of them. And it's not necessarily the most urgent, either. However, all things considered, having done enough this week to keep the good ship Pathways afloat, having not disappointed too many people, I feel I'm justified in doing what I would most enjoy from my task list and leave the rest until next week.

Some people would hardly bother to go through this rigmarole of deliberation. Others would not consider their feelings of enjoyment to be a relevant consideration, but would just plough ahead do the most important task whether they enjoyed it or not. I'm somewhere in the middle. Anyone who knew me well enough would be able to predict my decision. It's not that I always take the easiest or most enjoyable option; only sometimes. But you can bet that if there's any time I'm likely to do this it will be on a Friday afternoon.

Well, Alan's case on the face of it is slightly different. He claims that his decision was made 'effortlessly' by which I take it he means that there was no particular reason to submit the question or not submit the question. He could just as easily have not submitted it. Problem is, if he hadn't submitted it, we wouldn't be able to give him an answer. My grandmother used to say, 'If you're lucky, you can win the lottery without filling in the coupon.' But Ask a Philosopher doesn't work that way. We're not mind readers.

I know what Alan means. What he means is something that we find ourselves doing when we first consider the idea of free will. We want to prove it to ourselves, by doing something — freely. But what exactly does that entail?

This is familiar territory in philosophical discussions of freedom of the will. The default view, which you will find defended by many philosophers from David Hume onwards, is that an action is free provided that it is done from our own choice, not under duress and in full possession of our mental faculties. This view is known as 'compatibilism' — defined in this way, freedom of the will is fully compatible with determinism.

This won't satisfy Alan (and it doesn't fully satisfy me either) because this kind of 'freedom' hardly looks the kind of thing that we would want or be satisfied with. We want more. We don't want there to be a story about causes and effects that ultimately explains every action that we do. We want the action to come from us not from the world grinding on, doing its thing.

Confusingly, however, we also want it to be the case that when we do good, not only do we receive praise but also recognition that the action in question was to be expected, given our character. If the action is praiseworthy, it's an insult if someone says, 'I'm surprised you did that!'

But aren't we all part of the world? If I have the sort of character which would lead me to do something praiseworthy, or blameworthy, is that not a fact about the world? In that case, where does my 'process of deciding' fit in, if not as a process taking place in an entity situated at a place and time, following its nature or character?

There is something wrong with the statement I have just made; and it's wrongness was pointed out by the philosopher Jean-Paul Sartre. There is something that happens, at the moment of making a decision, that cannot be fully accounted for in terms of any amount of knowledge of one's character or predispositions. Every situation is unique. It doesn't matter if you have been here a thousand times before. You still have the opportunity to confound those who would predict your action. In this respect, it is wrong to see human beings as merely 'part of nature'. There is something added to the equation every time you decide, regardless of what other people expect, or even what you expect from yourself.

But how to prove this?

I've thought a lot about our meeting, Mr. Hofman. Since the beginning, I felt the need to see you. When you left the cafe, I realized I couldn't wait any longer. What you said on television persuaded me. I gathered the courage you spoke of. You can kill me. I acknowledge your right to do so. I'll take the risk. But I'm banking on your curiosity. You want to know what happened to Miss Saskia.

When I was 16, I discovered something. Everyone has those thoughts, but no one ever jumps. I told myself: 'Imagine you're jumping.' Is it predestined that I won't jump? How can it be predestined that I won't? So, to go against what is predestined, one must jump. I jumped. The fall was a holy event. I broke my left arm and lost 2 fingers. Why did I jump? A slight abnormality in my personality, imperceptible to those around me. You can find me listed in the medical encyclopedias under 'Sociopath' in the new editions.

The Vanishing 1988 (Dutch: Spoorlos) http://www.imdb.com/title/tt0096163/

What did Raymond Lemorne, Saskia's abductor, think he had proved all those years ago by jumping off the wall? There was a very good reason for not jumping off the wall — it's sheer height from the ground, the consequent risk of injury — but he also had a very good reason for jumping: to prove a point.

If Alan had wanted to 'prove' his free will, wouldn't it have been better to choose something he had a strong reason not to do, but do it nonetheless, in spite of his character and circumstances, in spite of himself? Lemorne gives the lie to that conceit. He knows what he is: a sociopath. And it's not as if you could just choose something trivial to prove the thesis. I could go home now, leave this post unfinished, leave my computers on (much to the annoyance of my office landlord) but I won't, because even if I did, it would prove nothing.

If you can't prove your free will by doing something predictable, you can't prove it by doing something unpredictable either.

But do I have to prove it? Don't I just know? As Alan states, it's 'meaningless' to assert that free will is an illusion when it 'feels so real'.

Isn't Alan just being a good empiricist here? How else do we find out about the world and about reality but from our experience? And some things just can't be meaningfully doubted. At the end of the day, you have to go along with your best take on how things seem, the best explanation. And how things seem, in the case of human action, is that actions come from us, not from the world. It's called 'saving the phenomena.'

I'm not going to dispute the claim about explanation. It could be argued that the whole purpose of seeking explanations is that we dig below the surface. Sometimes explanations can be counter-intuitive or paradoxical, yet we know them to be better than the explanations that seem easier to accept, because they take more into consideration. However, in the case of free will it's a moot point. As soon as you leave the perspective of the agent, in your attempt to 'take more into consideration' you lose the very thing you were trying to focus on.

My objection is different. To call something an 'illusion' implies that you grasp the difference between how things appear in respect of the entity in question, and how things are in reality. I know what it means to say that it is an illusion that a straight stick partially immersed in water appears bent, because I grasp the difference between what it is for a stick to be straight and what it is for a stick to be bent. If is an 'illusion' that I am freely deciding what to type next, then this is a claim about how things appear to me, at this moment. But that implies the possibility of there being some other way of seeing those same events. You immerse the stick in water, or you remove it. But there is no corresponding alternative in the case of human action.

Thomas Nagel in The View From Nowhere (1986) refers to this as the 'necessary penumbra of ignorance' of the causes of those events we regard as our actions.

In short, I don't know what I would be denying if I denied that free will is an illusion. I don't have any conception of 'how things might be otherwise'. Therein lies a possible solution to the free will problem. We think we know 'what we want', but the very attempt to state what we 'want' from freedom of the will falls into confusion.

Geoffrey Klempner

back

(32) Derrick asked:

I recently read a report on a requested made by the Russian Minister of Finance who asked that Russians Smoke and Drink more as the country needed the revenue, is this not as a result of their adoption of the capitalist system, a system that has been faulty since its exception.

Communism did not work and the West did its utmost to see it failed, the Capitalist system is no better as it benefits only a small segment of the population and the myth of the creation of wealth which is now the holy grail is all smoke and mirrors and has value as long as the paper Dollar retains its value.

When it comes to finance we have people who are awarded the Nobel prize for the creation of systems that are supposed to improve how systems work, I have yet to see this actually effect anything, in fact things keep getting worse.

We are told how well we are doing while pensioners don't know how they are going to survive.

We are also told the markets know best, best for who? A shareholders interest is never a countries interest, self interest is the only consideration.

What do we need to break the cycle of greed, a 3rd World war, but then war is profitable.

I understand Greek Philosophers had thoughts on matters of finance, does Philosophy have solutions or is man so flawed that we are to far into the abyss to pull back.

---

Forget about about Capitalism and Communism. Forget about slogans and simplistic truths. Try to think rationally about human economic activity and how it should be carried on.

The best way of producing the goods and services that people want is called 'Free enterprise' just as the best political system we have been able to devise is Democracy. Neither Free Enterprise nor Democracy are perfect and they need to be regulated and controlled by law.

The idea of Free Enterprise is that any individual or group of individuals is free to start an enterprise which produces goods and sells them to the public. The enterprises that produce goods that the public want will prosper. Those that do not will go out of business. There is nothing wrong with this basic idea of Free Enterprise.

It is the job of the government to ensure that the economic system is properly regulated by law and it is our job as citizens to ensure that the government do their job. It is also the job of the government to provide services that cannot be provided effectively by free enterprise and to decide which service are best provided by government.

Of course both people and politicians are imperfect. If the government fails to regulate the economic system then it will be taken over by crooks. All our past and present economic collapses have been caused by the government allowing the markets to be taken over by crooks. Communism collapsed because the system was taken over by communist party crooks. It is not greed or capitalism that is the problem, there always have been and always will be stupid short sighted greedy people, we have an endless supply of such people.

There are always democratic voters who can be persuaded that 'Greed is Good' although history shows again and again that greed doesn't work. What we must all learn is that money doesn't grow on trees, the markets aren't self regulating, there are no financial experts who can be trusted to run things. So called financial experts are as prone to superstition about economics as everyone else.

We have to control our own economic system and our politicians. Life is complex and there are no simple answers. Trying to make things work is always going to be a constant struggle for imperfect humans. There is no perfect system that will make everything work without effort.

Shaun Williamson

back

(33) Jae asked:

What are the four elements that Aristotle argues are required for an action to count as virtuous? Do you agree with him that all four are important?

---

This looks uncommonly like a school assignment question, and the Ask a Philosopher web service does not make a habit of doing peoples homework for them. But in this case, your question happens to coincide with a personal interest of mine, so I am going to attempt an answer.

I have only heard of four elements in association with Aristotles physics, never in association with his concept of virtue. (Aristotles four elements in physics are: earth, air, fire and water. But I don't think that you had that in mind.) I suspect that what you (or perhaps more correctly, your teacher?) are referring to is: balance, knowledge, the capacity for right choice, and an enduring virtuous disposition.

According to Aristotles conception of virtue and the Doctrine of the Mean, the virtuous action is always intermediate between the opposed vices of excess and deficiency: too much and too little are always wrong; the right kind of action always lies in the mean. [Nicomachean. Ethics II 6] Of course, what Aristotle means by the mean in this context, is the mean relative to us and to the circumstances, not any sense of arithmetical mean between two extremes. (Hence the suggestion that Aristotles doctrine of the mean recommends mediocrity or moderation in all things, is based on a misconception.) A proper sense of balance between the excesses of too much (too often or too soon) and the deficiencies of too little (not often enough or not soon enough) is therefore necessary in order for the virtuous to be able to identify what is the proper action (just enough), under the circumstances.

For Aristotle, the knowledge relevant to virtuous action is not theoretical knowledge like logic or mathematics or even physics, but rather the knowledge that comes from experience of the actions in life that is acquired by those who have been brought up in fine habits [Nicomachean Ethics 1095a3-b5]. A virtuous person can not simply study what virtue is, but must actually do virtuous things. We are not studying in order to know what virtue is, but to become good, for otherwise there would be no profit in it. [Nicomachean Ethics 2.2] The kind of knowledge involved is knowing how (to play tennis for example) rather than knowing that (the Earth orbits the Sun, for example). And unlike knowing that, knowing how can only be acquired by practice.

The capacity for right choice cannot be exercised as such unless one is already virtuous. It is generally thought to require the possession of a reasoned outlook of [one]s life as a whole [Aristotle on Learning to be Good, M.F.Burnyeat, pg 70]. However, the student of virtue can exercise the passions, capacities, tendencies, etc. to the extent that the student is disposed to perform virtuous actions. These are parts of the capacity for right choice which do not by themselves constitute virtue. They need to be perfected by practice and put together with the other parts of virtue before they can become the proper source of virtuous motivation in the student. We do not possess the virtues before we exercise them, but we must exercise the virtues before having them, since only by exercising them do we get to have the corresponding virtuous dispositions in us. [Metaphysics 9.8] You don't start learning how to play tennis by playing Rafael Nadal. You start with simple drills that everyone can do at some level (simply hitting the ball, for example), and then expand ones skills as ones skill expands.

The enduring virtuous disposition involves the total absence of any contrary motivations. In the properly trained student of virtue, the appetitive part of the soul is trained to recognize the differences between the proper pleasures of virtuous actions, and the alien pleasures that accompany the vices of excess and deficiency. Hence the enduring virtuous disposition is not tempted by the alien pleasures in the way that plagues the vicious, temperate or akratic dispositions. That the disposition is enduring is necessary because Aristotle considers virtue to be a property of soul or character, and not a property of any specific action or motivation. What matters is the nature of the habitual response to circumstances, not any particular choice or action.

As to whether I agree with Aristotle that these four elements are important considerations within his conception of virtue that all depends on whether I accept Aristotles definition of virtue. As long as I accept his conception of virtue (even if just for the sake of argument), then I would have to agree that these four elements I have identified are important components of his conception. Each is a fundamentally necessary component of Aristotles concept of what virtue means, within his larger theories of human psychology. But these four elements are consequences of his larger psychological and ethical theories, rather than constitutive of them. So their importance lies in their significance as consequences (symptoms) of those larger theories, and not in their contribution to them.

Hope this answers your question.

Stuart Burns

back

(34) Drewry asked:

Why are there only 3 skin colours?

Is god real?

If I went up in a rocket would I find heaven?

---

Such an odd concatenation of questions tweaked my curiosity.

(a) Skin Colours

Your question is possibly based on a misconception. (Possibly induced by erroneous instruction?) As any casual observer of the Human condition would attest, there are not just three skin colours. In fact, the range of skin colours exemplified by existing people pretty much spans the spectrum from lily white to coal black, with lots of shades of brown in between. Perhaps you are being confused by the foolish suggestion that there are three races black, white and yellow? But this also is quite erroneous. First of all, there is serious debate as to just what constitutes a race. Second of all, if any roughly acceptable of race is employed, there more than three. Here is a list that is frequently found acceptable (although, as I said, the concept of a race is much debated.) Caucasoid (Europe, West Asia and North Africa); Mongoloid (East Asia, Oceania, the Arctic and the Americas); Negroid (Sub-Saharan Africa); Capoid (Southern Africa); and Australoid (Australia and Melanesia).

(b) God

The concept of God is real because a concept is a creation of Man. And in fact, there appears to be as many different concepts of God as there are people who talk about God. But any coherent description of that concept that I have come across has been internally self-contradictory. It is generally assumed that reality is self-consistent (ie. whatever is real does not contradict itself). Hence it is logically demonstrable that there can not exist any entity that corresponds to any of the concepts of God that I have come across. If you would like to attempt to provide a logically self-consistent description of what it is you mean by your use of the label god, I will be happy to re-consider my answer.

(c) Heaven

First question what is it that you mean by the label heaven? I have had the same problem with this concept as I have had with the concept of god. I have yet to encounter a coherent description of heaven. Second question where is heaven located? Most of the answers I have encountered have indicated that heaven is located outside of our current space-time continuum. On the basis of that description, I would have to answer that no relocation by you within the space-time continuum would get you to heaven. So going up in a rocket will not suffice. (Tongue in cheek its the coming down without a parachute that would get you there if you have been a good boy/ girl!)

Stuart Burns

back

(35) Amanda asked:

Which ethical theory is most 'natural'? That is, which theory requires the least alteration of our natural inclinations in order for us to live ethically?

---

Given the very specific way in which you posed this question, the answer would unquestionably be Evolutionary Ethics.

Evolutionary Ethics proposes that the morally right/ proper/ good thing to do in a given set of circumstances is that which maximizes the probability that moral agents genes will continue to flourish over the longest term possible. Given that we are an evolved species, we are already genetically predisposed to pursue the welfare of our genes to the best extent that evolution has managed to program us. All that Evolutionary Ethics adds is the ethical rule that we ought to actively employ our cognitive abilities of foresight, planning, and predicting to pursue with conscious intent, and to the very best of our abilities, what our genetic predisposition has already prepared us for, and what evolutionary processes demand. The future, after all, will be populated by those who are most successful in this endeavour.

Stuart Burns

back

(36) Derrick asked:

I recently read a report on a request made by the Russian Minister of Finance who asked that Russians Smoke and Drink more as the country needed the revenue, is this not as a result of their adoption of the capitalist system, a system that has been faulty since its exception.

Communism did not work and the West did its utmost to see it failed. The Capitalist system is no better as it benefits only a small segment of the population and the myth of the creation of wealth which is now the holy grail is all smoke and mirrors and has value as long as the paper Dollar retains its value.

When it comes to finance we have people who are awarded the Nobel prize for the creation of systems that are supposed to improve how systems work, I have yet to see this actually effect anything, in fact things keep getting worse.

We are told how well we are doing while pensioners don't know how they are going to survive.

We are also told the markets know best, best for who? A shareholders interest is never a countries interest, self interest is the only consideration.

What do we need to break the cycle of greed, a 3rd World war, but then war is profitable.

I understand Greek Philosophers had thoughts on matters of finance, does Philosophy have solutions or is man so flawed that we are to far into the abyss to pull back.

---

(a) Does philosophy have solutions? I certainly wont suggest that Philosophy has solutions. That is not what philosophy is about. But philosophy can certainly offer some contribution for some part of the list of woes that you cite in your question. In particular, philosophy has given me the ability to recognize that you have made numerous errors of fact and theory in the content of your question. Perhaps it may be possible that by exposing some of these errors and showing you some examples of your erroneous thinking, I might make is easier for you to see that circumstances are not nearly as bad as you seem to think they are.

(b) that Russian Ministers request for added revenues. I saw a similar report. However, the report that I saw made it clear that the Minister in question was not being serious in his request that his people smoke and drink more. The report I saw made it clear that his request was made as an effort to highlight his governments difficulties in obtaining sufficient tax funding for necessary government expenditures as it were tongue in cheek. However, we can ignore this possible misconstrual of the Ministers intent, and pretend for arguments sake, that the Minister was serious in his suggestion that his people drink and smoke more as a means of increasing tax revenues.

This would not have happened as a result of Russia adopting a capitalist economic system (instead of their previous communist system). It is obvious to me that you do not clearly understand what a capitalist economic system is. One of the things it is not, is a prescription of the extent of government requirements for tax revenues, nor a prescription of just how that revenue is to be raised. It is, on the other hand, an economic system that prescribes how economic capital is to be best allocated amongst competing alternative uses. Fundamental to a capitalist economic system is a free market a (possibly virtual) place where people can freely exchange that which they value less for that which they value more. Critical to the existence of a free market is an absence of force or fraud.

What a government chooses to do (and thus have a need to pay for), and how a government chooses to raise the funds necessary to pay for what it chooses to do, is no part of a capitalist economic system. What students of capitalism do study, and do debate about, is what the consequences are likely to be, given certain government decisions about what it chooses to do, and how it chooses to pay for those things. For whatever a government chooses to do, and however a government chooses to raise the revenues it finds necessary, those actions will constitute interference in the functioning of free markets. Hence any government action, however well intentioned, constitutes an interference in a capitalistic economy, and can be shown to be a source of economic inefficiency.

The Ministers suggestion that tax revenues could be raised by increasing the amount of smoking and drinking that his electors do, has therefore little to do with capitalism. Even under communism, the Minister could easily have been faced with exactly the same problem of raising revenues to pay for government expenses. And even under communism, the Minister could have made exactly the same suggestion. The only difference, at this level of discussion, between capitalism and communism, would be the range of options available for Minister to consider. Under communism, because of its central-command organization of the economy, the Minister would have had very many more options.

(c) The Capitalist system is no better. You claim that the Capitalist system is no better [than Communism] as it benefits only a small segment of the population. But this is simply false as to fact. Capitalist economies have delivered far more welfare to far more people than any economic system ever devised. In those countries that have been capitalist economies for over 70 years (so that comparison is possible), even the poorest people today are far better off than were the wealthy 70 years ago. On any scale of measure you might choose to apply, over the 70 years of relevance (1920-1990) capitalism has delivered greater benefits to more people than communism did. The difference in the welfare (however you choose to measure it) of the average Russian of 1920 and the average Russian of the 1990s was insignificant compared to the difference in welfare of the average American/ Canadian/ Brit (etc) of the 1920s compared the 1990s.

It is certainly true that a capitalist economic system tends to promote a disparity between the very wealthy and the very poor. That is because capitalism, as an economic system, rewards productive and creative talent (where communism does not). If, by magic, we could suddenly redistribute all the wealth equally amongst all citizens, the natural differences between peoples as to their talents and abilities would in short order reproduce disparities in wealth distribution. Capitalism rewards those who produce and create new wealth.

(d) the myth of the creation of wealth is all smoke and mirrors Communism, as a system of economic organization, is very good at spreading an existing stock of wealth amongst a population. Communism thus has attractions in a social environment where there is great disparity between the very wealthy and the very poor (as, say, in Europe in the 1860s and 1870s, Russia in the 1910s). But communism is very poor at encouraging productivity, innovation and creativity because it does not reward these things. Capitalism, on the other hand is very good at encouraging productivity, innovation and creativity because it rewards these activities very well. And it is productivity, innovation and creativity that creates wealth.

Capitalism has traditionally been attacked by its opponents on the basis of two historical phenomena. One is the appearance of Robber Barons in the period 1880-1920 in the United States a period that critics of capitalism have called Laissez Faire Capitalism. The second is the extent of interference by the various levels of government in the United States an economy that is held up by critics of capitalism as the premier example of a capitalist economic system in action. But both of these historical phenomena have been seriously misinterpreted and misunderstood by the critics of capitalism.

It was called Laissez Faire Capitalism to imply that it was a period in which the economic rules of capitalism were allowed to operate free of government interference. The implication is then that the obvious flaws were the result of capitalism rather than government interference. Unfortunately for the critics, the period of Laissez Faire Capitalism was anything but a laissez faire capitalist economic system. Instead it was an historical period that was remarkably similar to recent Russian economic history. In the two decades since the collapse of Soviet communism, the Russian economy is notable more for its kleptocracy than for its capitalism. The workings of the modern Russian economy are controlled less by capitalistic market forces than by who you know and what you can steal. Force and fraud, collusion and conspiracy govern Russian economics today. Capitalism is just the underlying foundation.

The same sort of social environment permitted the rise of the robber barons in the United States at the turn of the last century [the likes of John Jacob Astor (real estate), Andrew Carnegie (steel), Edward Henry Harriman (railroads), Milton S. Hershey (Chocolate), J. P. Morgan (banking), John D. Rockefeller (oil), Leland Stanford (railroads), Cornelius Vanderbilt (railroads)]. As common as it is to speak of robber barons as the proof that capitalism only enriches the very wealthy, most such critics are confused about the role of capitalism and fail to make an important distinction. One needs to distinguish between what might be called a market capitalist and a political capitalist. A pure market capitalist succeeds by selling a newer, better, or less expensive product on the free market without any government subsidies, direct or indirect. The key to his success as a capitalist is his ability to please the consumer, for in a capitalist society the consumer ultimately calls the economic shots. By contrast, a political capitalist succeeds primarily by influencing government to subsidize his business or industry, or to enact legislation or regulation that harms his competitors.

Fair trade and market forces had little to do with the rise of the railroads, the steel monopolies, the oil monopolies, and other similar foundations for the wealth of the robber barons. Influence in governments permitted the railroads to acquire, without cost, great stretches of land and associated land rights. Coercion and unfair trade practices with government collusion, not market forces, permitted the rise of the great monopolies of the early 1900s.

If you are going to look for a period in history to hold up as a great example of capitalism in action, you must not look to the robber barons. Look instead at the great flourishing of wealth that occurred in the 50 years between 1850 and 1900 as the new immigrants to North America flooded into the west and mid-west to exploit the lands captured from the Native Indian population. Here, at this local level, unhindered by any serious governmental interference, was capitalism in action at its best.

This earlier era, and not the modern United States, is the premier example of capitalism in action. Capitalism, as an economic system, depends on cooperative fair exchange and a minimum of governmental interference. In its modern manifestation, with its excessive governmental interference in markets, capitalism, as a system of economic organization, is hardly recognizable. Things may keep getting worse, as you put it. But that is not the fault of capitalism. It is entirely the fault of the well meaning, but sorely economically ignorant, legislators who keep meddling in things they oughtn't. They, and their electors, have simply not grasped the basic economic truth of capitalism if you meddle with the market, you will reap unintended consequences. The market forces of capitalism are far more efficient in managing an economy than any central planner can possibly be. But that is entirely because any economy is so complex, with so many different and often conflicting feed-back loops, that to any kind of analysis it is a chaotic thing. Hence, any legislature, contemplating a change in any kind of rule, can never predict what the economic consequences are going to be.

(e) pensioners don't know how they are going to survive. Pensioners may indeed not know how they are going to survive. But that is not the fault of capitalism. It is, instead, the fault of government. Pensioners are in dire straits for two reasons. First, the very existence of pensions is due to government rules. Pensions, as we know them, would not exist but for those rules. You cant, therefore, blame pension granting organizations for playing the pension game by the rules dictated by the government. If you don't like the fact that the Enron employees had their entire pension savings wiped out when Enron failed, then don't blame the Enron executives. Blame instead the government for setting up the rules to allow companies to invest their pension funds into company stock. Any reasonably prudent investor will advise you to spread your risk if your job depends on your company, make sure your savings do not. But the political capitalists who run the larger companies managed to persuade (buy?) legislators so that they could invest their pension funds in company stock. Who is at fault here? The businessmen who are paid to look out for their company, or the politicians who are paid to look out for the people?

If, as individuals, we are concerned about our economic future in a retirement without a family to support us then as individuals we can, should, and as many do, make preparations for retiring on an economically sound basis without a reliance on pensions. But then along comes the government to tax our earnings, to tax our capital gains, and to persuade us that Big Government will provide, and we need not as individuals, concern ourselves with our future.

And even if, as an untrusting soul is often prepared to do, we make preparations anyway and set aside some savings to keep us comfortable in our retirement along comes the government with its inflation policies to inflate our savings into worthlessness. This is not capitalism. This is big government doing this to screw the people albeit with the best of intentions.

(f) the markets know best, best for who? The reason that capitalism is so enormously successful at wealth creation, is that it depends on the forces of free markets to allocate productive capital (money, land, labour, etc.) to the projects where it will earn the greatest rewards. There is no other means that is anywhere near as efficient as a free market in this function. And this is best for everyone who has any role in the economy.

As a consumer, you want cheap but high quality goods to buy. So open up the marketplace to as wide a collection of competitors as possible international markets. Producers of anything that you are willing to pay for, will line up to offer you the cheapest alternatives and then let you decide how much quality you are willing to pay for. If you don't like the cheap Chinese imported toys at Walmart, then go to a different store where you can find more expensive but better quality toys. Toys is just the example. It holds good for every single product that you might want to buy. So who benefits? The consumers benefit. Without free international markets in toys, toys would be far more expensive.

As a labourer, looking for a well paying job, you want the greatest paycheck for the least amount of work on you part. So open up the marketplace to as wide a collection of competitors as possible keep an open mind and be willing to go where the jobs are. Consumers of labour will line up to offer you the highest paycheck they can afford to offer you, given the skills and abilities that you are willing to offer. Consumers of labour can only hammer down the wages they have to pay if the labourers are unwilling to go where the wages are. So who benefits? The labourers benefit. Without a free market in labour (jobs) and wages, paychecks would be a lot smaller than they are.

Of course, as I mentioned above, free markets demand fair and uncoerced exchanges, and they demand free exchanges of information as well. False advertising in either consumer products or in the jobs market, is tantamount to fraud. In either case, you have to be able to expect to get what you pay for, and not a pig in a poke. But lying and cheating, fraud and thievery, are not problems exclusive to capitalism. It is just that the free markets that are the hallmarks of capitalism are more sensitive to abuses.

But, since everyone is both a consumer and a labourer, free markets benefit everyone!!

(g) self interest is the only consideration. Yes, self interest is (or at least ought to be) the only consideration. But you state this as if it were unfortunate that it is so. However, no other consideration is as effective at motivating people. Even within the communistic system, it came to be recognized that the reason it did not work, is that everyone, at every level, in all situations of choice, is first and foremost, self-interested. (Careful a caveat is needed here. To be self-interested is not necessarily to be narrow minded and parochial about it. Intelligent self-interests looks to the long term, and the wider consequences of things.) Capitalism, as a system of economic organization, takes advantage of this innate human characteristic.

(h) is man so flawed that we are to far into the abyss to pull back So I would argue that looking at man as flawed is to have the wrong perspective on human nature. Man is not flawed. But our governments are. That abyss you speak of is not a creation of capitalism and human nature. It is a creation of our governments, and our misplaced attraction to altruistic ethics principles. Lets go back and re-emphasize self-interest and self-reliance, and get the government out of the way. What we need is less good intentions on the part of our legislators, and more benign neglect.

I hope that some of this response seeps in, and you recognize that the real source of that litany of woes that you bemoan is not capitalism and the workings of an capitalist economy, but the machinations of government legislators and the electorate that keeps demanding more government actions to fix what is really government created problems.

Stuart Burns

back

(37) Ruth asked:

(I apologise for asking such a basic question, but I have googled and googled and... nothing.)

I was reading a discussion online the other day and one of the participants posited that 'all experience essentially takes place in the mind'. My question is, if there is no such thing as 'objective' reality, are people altered by the things they experience and change because of outside stimulus, or do they 'change' the things they experience to suit their own framework? Which choice is preferable?

---

At first glance, Ruth's question looks like a question about idealism. But I don't think it is. The idealist doesn't say that 'there is no such thing as 'objective' reality'. On the contrary, Berkeley's immaterialism, Kant's transcendental idealism, or Hegel's objective idealism are all theories about the nature of objective reality. In these theories, mind plays an important role, but it is not your mind or my mind but Mind (with a capital 'M').

It is fair to say that the current philosophical climate is predominantly realist rather than idealist. Yet even the staunchest realist would agree that our point of view is not the 'View from Nowhere' as Nagel terms it (Thomas Nagel The View From Nowhere 1986). The way we gain knowledge about the world outside us, our ability to access the 'objective' facts, depends on many factors including our mental constitution, sensory capacities, concepts and linguistic resources. Human beings differ from one another in this respect, although there is a also sense in which there exists a specifically 'human way' of perceiving and gaining knowledge of the world, by contrast, e.g., with that of a whale or a bat.

So in response to the question, 'Do we change because of outside stimulus, or do we 'change' the things we experience to suit our own framework?' my answer is, both.

I am writing this tentative answer today because when I checked my 'Questions In' mailbox I found Ruth's question there. If there hadn't been a question that interested me, I might have been doing something else. When Ruth clicked the button at Ask a Philosopher to submit her query, that action in a small way changed the course of my life.

Yet it is also true that the things I experience, the way the world impresses itself on me and stimulates me to action, depends on my desires, attitudes, moods. By working on myself, by reflecting on the way I feel and think, I effectively change my world. The world is the world, the same world for all of us; but I can choose where to live in that world, my intellectual habitation — be it high or low, austere or lush. In a very real sense, it is up to me to create my own reality.

Which choice is preferable? How do you choose when to open yourself up and let the world impress itself on you, and when to work on yourself in order to make the world — or your world — different? That's a fair question. Each person, I would argue, differs in this respect. It is a particularly tricky question for the philosopher.

As a seeker after truth, my aim is or ought to be to make my subjective contribution as small as possible so that I can accurately reflect the nature of objective reality. It isn't up to my free choice whether to be a materialist, or a dualist, a realist or idealist. I have to let the arguments impress themselves upon me, and then decide. I am nothing and reality is everything. That attitude is often taken as definitive of the 'philosophical standpoint'.

And yet, truth seeking would be a pointless exercise, if it were not part of a strenuous effort to make sense of things. It's not as if any 'truths' will do. A philosopher is only concerned with ultimate or universal truths, truths which would remain true even if the actual world as we find it was different in so many different ways. But that's still too many. My world is meaningful, or meaningless, depending on choices I make, for example, choices about which truths to focus upon, which questions to live with.

As regards 'how to live' in a practical sense, there don't seem to be many choices open to me, given my resources and ongoing commitments, my place in society. And yet as regards making sense of things intellectually, all the work is yet to be done. As I remarked last time, the world seems to me like a puzzle that doesn't add up. That impression, that feeling: is it an accurate reflection of reality, or is it rather the reality I have merely chosen to inhabit?

It feels like a choice. I have chosen to be gripped by a question which, if the reactions of students, colleagues, friends are anything to go by, not many people nor even many academic philosophers find puzzling. I don't have to spend all my time thinking about it. I don't have to slip into this mood. But I do, because that is what I will.

I don't find much comfort in the thought that the thoughts I am thinking now are merely the product of two and a half thousand years of the history of philosophy. That somehow I am merely 'continuing a tradition'. The past is the past, water under the bridge. It's true that 'those ignorant of the history of philosophy are doomed to repeat it' (as I often tell my students). After nearly 40 years doing this, I think I know enough about the history of philosophy to get by. (Not nearly as much as the late Anthony Harrison-Barbet, author of Philosophical Connections but I doubt whether many working academic philosophers do.)

I said last time that 'to hold the entire universe 'in question' seems liberating, in a strange sort of way'. Why do I need to be liberated? liberated from what? The idea that philosophy has its 'consolations' is as old as Boethius, or older, but I'm not ashamed to admit that I chose philosophy all those years ago because I needed it, because it seemed to be the only way I could stop my world from falling apart. And it's done a pretty good job ever since.

Some will sense that the G-word is in the background to all of this. The theist says, 'Of course the world has a purpose, the purpose given to it by God.' My response: If it turned out that God did exist (don't ask me how we would know), it would be our duty to kill him, or her (don't ask me how we would achieve this). If it turned out that God didn't exist (don't ask etc.), it would be our duty to create him, or her (don't ask etc.).

This isn't some mad idea; better minds have been here before me. But I'm not really interested in the God question, I see through all these facile moves. This isn't where the answer is going to be found. (Like philosophy, religion is a life choice. I just don't think that it is a very good choice, but I'm here speaking for myself not for you.)

So, in my own mind, I have found something better than religion. I've spent two thirds of a lifetime creating my world, and the job is not done yet. When it is, I'll let you know.


If the sun refused to shine,
I don't mind, I don't mind.
If the mountains fell in the sea,
Let it be, it ain't me.
Got my own world to live through
And I ain't gonna copy you.

(Jimi Hendrix 'If 6 was 9')

Geoffrey Klempner

back

(38) Mahdee asked:

which is the true religion?

---

What makes you think that there is any true religion?

The world is over-full of different religions. Each claims to be the true religion, and the one and only means of getting closer to God. Each maintains, in language of various degrees of nastiness, that those who do not adhere to the one true way are going to hell. Since they all proclaim this, they cannot all be correct.

Given the absolutely complete and total lack of any evidence what-so-ever that might be cited in support of any one of these claims, and given the sufficiency of evidence that suggests that none of these claims has any merit, it is best to conclude that all religions are wrong.

Or, to put it another way the one true religion is science! (Not that I am suggesting that science is anything like a religion).

Stuart Burns

back

(39) Paul asked:

The old question how do I know that I exist, and does it really matter if I cannot prove that I exist with certainty.

Thanks.

---

You have to remember that language is a joint invention of human beings so that they could talk to each other about things that interest them and doubt is just a word in that language.

It was Descartes who first suggested that we should doubt everything that could be doubted but maybe it would have been more sensible for him to have suggested that we doubt everything that could sensibly be doubted.

It may make sense for a two year old to doubt that 2 plus 2 equals 4. When you are a bit older it becomes incredibly hard to doubt this.

Just look in a mirror and say to yourself 'Do I really exist?' Now do it again and this time with real feeling.

Now suppose you discover that you were kidnapped as a child and that even your name may not be your real name. Now it may make sense to doubt that the supposed you really exists.

You may have to prove in court that you are the person you say you are and you could look on this as proving that you really exist. However looking in a mirror and saying 'Does the speaker of this sentence really exist' makes no real sense except maybe as a joke. However that doesn't make it a proof that you exist either. We have proofs in mathematics and logic and maybe in courts of law. In general we have nothing called a proof of my existence or a doubt of my existence either.

Shaun Williamson

back

(40) Lisa asked:

What would happen if every cell phone in the world was deactivated for one year?

---

There would be no more cell phone calls. There would be queues outside telephone boxes. There would be fewer car accidents.

Fewer people would injure themselves walking into lampposts. There would be no people on the bus saying in a very loud voice I'M ON THE BUS'.

We would realise that people talking to themselves in the street probably have mental problems.

Shaun Williamson

back

(41) Richard asked:

Setting aside the archaic origin of the word democracy.

If the majority of people and Parliament decided that Popes are, or can be, infallible as regards social ethics.

Would we thereafter have democracy in this country?

---

Democracy is about people being able to choose their government at regular intervals.

Democratic governments can make laws but they cannot require people to believe that X is true or X is false. A government might make laws that are in accord with the Pope's social ethics if they believe that this is best for the country but this can always be overturned by a future democratic government. Democratic governments have no power to decide what it true and what is false.

Governments are not qualified to pronounce upon matters of church doctrine such as papal infallibility and no one needs to pay attention to what they say on such things.

Any democratic government which decided to overturn democracy becomes a tyranny and is no longer a democratic government.

You might like to note, if case you don't already know it, that the powers and scope of papal infallibility are very limited and they don't extend to social ethics. The pope's views on things like contraception or abortion or homosexuality are merely the pope's views. They are not and never have been part of the doctrines of the Catholic Church.

Because of the pope's position as the head head of the church many Catholics feel that they should respect his views on these things. However an increasing number of Catholics feel that his views are wrong and should be ignored.

Governments make laws but people people make morality. However morality can never be decided by majority votes and neither can truth.

Shaun Williamson

back

(42) Brandon asked:

I was reading about psychological egoism, and its criticisms, and Feinberg popped up in the latter portion. Anyway, he presented a circular argument, supposedly, against psych. ego.:

'All men desire only satisfaction.'

'Satisfaction of what?'

'Satisfaction of their desires.'

'Their desires for what?'

'Their desires for satisfaction.'

'Satisfaction of what?'

'Their desires.'

'For what?'

'For satisfaction' etc., ad infinitum.

I was wondering: isn't this 'argument' not really valid? It's not really any structured argument against the subject, but, rather, pointing out fallacious wording. Even though it is circular, it doesn't do anything to quell the implications of egoism, nor any other problems. And, doesn't the first statement satisfy the conditions? Even if it's tautological, I see no reason to expound such a redundant example.

And, a bit further, he submitted further arguments/ papers against psychological egoism, but, doesn't this, in an ironic twist, make his efforts an example of egoism?

---

Well let's forget about the ironic twists because the ironic twists just beg the question.

There are judgement terms that all humans use to classify actions, terms such as selfish, unselfish, egoistic, non egoistic and so on. It only makes sense to use such terms if you use them to make a distinction between two sorts of things.

There have been certain moral theorists who have said that all human actions are selfish. However when you examine such claims you find that they are really claiming that all human actions MUST be selfish under their new definition of the word selfish.

If all actions MUST be selfish than the word selfish ceases to be a word that can be used to classify human actions into two types. It no longer makes any sense to describe an action as selfish if you deny that the opposite could exist. It is this sort of fallacy that some thinkers attribute to psychological egoism.

They redefine the word 'egoistic' so that all actions become egoistic under their new definition. All they have done (by an ironic twist) is rob the word 'egoistic' of all meaning. You cannot say 'He was not an egoistic person' because such a person CANNOT exist given their new definition of the word egoistic.

Shaun Williamson

back

(43) Dake asked:

I am studying marketing though l don't believe if its my field. of course I'm thinking of changing my career but not sure which area is best for me. anyway my question is how do philosophers think. how do they argue present their arguments come to a conclusion. my friend always say l don't ask question , they say l interrogate people it makes me feel uncomfortable. lol

How can you reason like a philosopher and derive meaning

---

Philosophers are interested in truth, the truth about everything no matter where it leads. Philosophers do not argue in any special way but they are always arguing. I would not recommend that anyone study philosophy without first studying the history of philosophy and seeing if it interests them. Philosophy is unlikely to make you any money, in this respect, it is like art and music. It is something that people do because they have to do it.

Marketing is about winning friends and influencing people and it also offers you the opportunity to become rich. You are unlikely to meet the keenest inquiring minds on a marketing course because that is not what marketing is about. It is also not what knitting is about but there is nothing wrong with knitting.

I wouldn't worry too much about what your friends say. If you interrogate people, is it in a personal way i.e. about their lives and their feelings. If so then you might have to try to be more sensitive about that.

However curiosity and wanting to know things is generally a good thing. I can't really advise you about what you should do. You could go to see an occupational psychologist or you could think about what you wanted to be when you were a child. Maybe this will give you some ideas. I have never known a child who wanted to be in marketing.

While you are trying to make up your mind read some novels and poetry to give you inspiration. For poetry try the collected works of W.H. Auden. For novels read 'The Little Friend' by Donna Tartt and 'Wide Sargasso Sea by Jean Rhys. For philosophy read 'History of Western Philosophy' by Bertrand Russell. In the meantime do the best you can with the marketing course.

Shaun Williamson

back

(44) Courtney asked:

A man named Morpheus approaches you on the street and tells you that the world is not real. Specifically, he makes the claim that you are plugged into a machine, and the world that you believe to be real is nothing but a computer simulation. He then challenges you to prove him wrong. With reference to Descartes, make an argument that either agrees or disagrees with his position. After establishing your Descartes based position on the external world, argue against the opposite one. Make sure not to take any red or blue pills until you do!

---

This is a typical philosophy instructor's question, and I reckon from the language an American philosophy instructor. The last sentence suggests female rather than male. I can't say exactly why it does, it just does. But that's mere guessing. I'm not being responsible in making this assertion. Descartes would say, I am willfully abusing my God-given powers of judgement. The truth is, I don't know and wouldn't know even if by pure luck my guess turned out to be correct.

But I'm jumping ahead. In Meditation 1, where Descartes considers the possibility that he is being deceived by an all-powerful evil demon, the fear is that exercising one's judgement responsibly is no more likely to arrive at the truth than wild guessing. The corrosive world-destroying doubt only ends when Descartes succeeds in convincing himself that God exists and is not a deceiver. God wouldn't give me ample evidence for the existence of an external world when no such world exists.

But that doesn't mean we can't make false judgements. The best we can do, in the face of the ever-present possibility of empirical error is to remind ourselves that we have made errors in the past and keep our eyes open for new evidence that overturns what we previously believed. That's part of what it means to exercise one's judgement responsibly.

In Meditation 6, Descartes goes further and explains in considerable detail how it is that illusions and misperceptions arise. Our perceptual powers such as sight and hearing, our ability to sense when we have suffered an injury, depend on physical processes which God has designed to lead us to truth. But even the best, most optimal design doesn't guarantee that we will always attain the truth. The very same laws of nature which lead us to knowledge can also lead us into error.

So what would Descartes say about the Matrix scenario? It's possible. It could happen in reality — if we grant the hypothesis that human beings will one day create artificial intelligence. Of course, Descartes would disagree on this particular point: he believed that intelligence requires a non-physical soul. Non-human animals are just machines, he thought, like the clockwork birds twittering in cages that amused the idle rich. But that detail is easy enough to fix. We can change the Matrix story to one where an evil angel, with finite not infinite powers, puts us asleep and makes us dream of a world of 2010.

Would God allow this? Why not? There are evil angels (Satan and his host) whom God could destroy if He wanted to, but out of his infinite wisdom and benevolence chooses not to. The point is that, even while dreaming, we are not deceived into thinking that an external world exists. We are physical objects existing in space, even while asleep. Even though we are deceived, there remains the possibility of discovering the deception, provided we exercise our judgement responsibly. That's what Neo does in the Matrix when he concludes (rightly, as we the audience know) that after he is 'woken up' in a pod with tubes attached to his back, this was his first taste of reality, and not just the beginning of a science fiction nightmare.

So in answer to the instructor's question, nothing Descartes says in the Meditations proves that Morpheus is wrong.

I'm going to take a bit of a jump now. Actually, it's a huge leap, but you'll see the point in a minute. In my post a couple of weeks back on what a philosopher might think about I confessed that 'Like Neo in The Matrix, I know there is 'something wrong with the world'.' Something tells me that this isn't real. I don't mean that I am not awake, at home (because the heavy snow made it impossible to get into my office today), writing a post for my Tentative Answers blog. I've no doubts about that. I mean something deeper, not just 'more of the same' which is all you discover if you take the red pill.

Hegel thought about this. In one of the most difficult passages in his Phenomenology of Spirit (a text which I've struggled with and never mastered), he turns the tables on every attempt at drawing a distinction between 'the apparent world' and 'the real world' — a project which traces back to the earliest Greek Philosophers Thales, Anaximander and Anaximenes. The final, most sophisticated version of this story is Kant's distinction between the phenomena and noumena, or the 'world of appearances' and 'things in themselves'.

The passage in question is entitled, cryptically, 'The Inverted World'. (I apologize to Hegel scholars in advance, because I don't have the text in front of me.)

Why an 'inverted world'? Hegel considers the idea of a reality 'behind' the world of appearances. This world is 'different', indeed radically different. The extravagant idea that everything in this other world is the 'inversion' of what it is in this world is meant to be a metaphor. Scientific inquiry is all about this world, the world of appearances, just as Kant believed. Yet there must be something more, Kant thought, than just the world of science: the ultimate reality, which we can never know or comprehend because our knowledge is limited to the world of our possible experience.

Now you can say (with Wittgenstein) that 'a nothing will serve as well as a something about which nothing can be said'. But Hegel goes further, and that's what makes this passage so brilliant. He gets right into the brain of someone who believes, wants there to be something more. Yet all we know about this 'something' is its sheer 'difference'. The inverted world is opposite to all we know. What does that mean? Nothing, says Hegel! We are deceiving ourselves with a picture (as Wittgenstein would have remarked).

In Hegel's metaphysics, no 'ultimate reality' is revealed, or posited, because the Absolute is none other than this world, seen aright. Seen aright, the world has an irreducibly teleological structure by virtue of which we can construct a suitable object for religious awe, even though God or Christ in a Hegelian universe is a different entity from anything the non-philosophical believer would recognize. From the perspective of the Absolute, the meaning of human history is finally revealed. This is it, there is nothing else.

Are you still with me?

In the 20th century, arguably the two seminal philosophical texts in Metaphysics are Heidegger's Being and Time and Whitehead's Process and Reality. Right at the beginning of their respective works, these philosophers nail their colours to the mast. According to Heidegger, what phenomenological ontology seeks to reveal is (as I would put it) the 'wood we fail to see for the trees', the ontological structure of appearances. According to Whitehead, the task of the philosopher is to 'frame the best set of categories that we can', categories which apply to the world of our experience, more general than the categories of physics because they depend on only the most general features of experience. In Whitehead's memorable metaphor, we don't notice the elephant which is always present.

(It goes without saying that both philosophers owe a massive debt to Hegel.)

Now I remember that that was the topic of my very first post, August 1009 The elephant in the room...

Funny that I seem to have gone full circle. Have I made any progress? I see through myself. I have nothing to offer, nothing to contribute to the academic debate. Nothing hidden away in some dark corner of my mind. I won't wake up tomorrow and 'remember' the truth. And, supposing I did, it would just be more of the same. Whatever it is I want, whatever it is I'm looking for, I can't even give a name to. (It sure ain't religion, so don't even think of going there. Read my other posts.) All I know for sure is that Heidegger and Whitehead are old hat. — Nor even Emmanuel Levinas author of Totality and Infinity which some philosophers would rate even higher, whom I once thought was the veritable bees knees.

The elephant is sitting right next to me. Staring at me. Chuckling silently as I scramble through every possible permutation. Logic isn't enough!

But you know what? I don't give up that easily. So what if I took the red pill and nothing happened. That just goes to show that you shouldn't rely on pills.

Geoffrey Klempner

back

(45) Sophy asked:

When making arguments with things like modus ponens, are there any standards for what you are allowed to call 'True'? I am saying the argument is valid but unsound, because something that is 'somewhat likely' is being called true. Is there some reference on what you are allowed to call true? If something happens even most of the time it doesn't seem right to call it True.

---

Any argument can only be considered to be a real argument if it is a logically valid argument. If an argument is not logically valid then we do not have to consider it as a real argument.

The definition of a logically valid argument is one where IF the premises are true THEN the conclusion CANNOT be false.

However the fact that an argument is logically valid does not mean that the conclusion is true. It does not mean that an argument is sound or relevant or interesting.

Any sort of argument whether sound or interesting or relevant must first be a logically valid argument. If it isn't then it is only a pretend argument and we can ignore it. A valid argument may not be a sound argument but a sound argument must always be a valid argument.

Shaun Williamson

back

(46) Klein asked:

Is there any way to refute the idea that 'might makes right'?

---

In his Republic, Plato or rather, his character of Socrates tried to refute Thrasymachus' proposition the 'might is right'. Jean-Jacques Rousseau also attempted to refute the proposition in his The Social Contract.

Plato

Thrasymachus states the Morality is the advantage of the stronger party. Right is Might. In any country, the stronger party is the Government. Government makes laws. Citizens obey the laws and the Government benefits. Socrates replies that Governments make bad laws as well as good. Obeying the former disadvantages the Government. So obeying the laws of the stronger, the mightier does not make it right. Thrasymachus retorts that professional rulers who make mistakes, should not be rulers.

Socrates states that genuine doctors attend the sick just as genuine sea-captains attend to their sailors. Both have expertise in their respective fields; it is the interest of expertise to be as perfect as possible. If not, it is not expertise and needs the supervision of a real expert. Expertise concerns the content of which it is expert of and not itself. Again, if not, it is not expertise. Expertise is by definition, something the non-experts lack. Lacking is in this sense, to be weaker. The experts have power and authority over the weaker.

Experts such as Doctors and Sea-Captains do not use their expertise to their own advantage, they use it for the advantage of their patients and crew-members respectively. Expert medical knowledge is used to the advantage of sick-people, not to the advantage of a non-sick Doctor.

Hence contra Thrasymachus, any one in authority in capacity of ruler, does not consider or enjoy ruling for his own advantage. Everything they say and do, is done in the interests of the people over which they practice their expertise. So ruling is not based on might, it is based on possessing and using expertise. Those with knowledge are the right rulers.

Despite this, Thrasymachus maintains that morality is followed in the interests of the stronger, the mightier. Further, being immoral can bring advantage. The problem with it is not that people don't follow it, they just don't enjoy being on the receiving end of it. Otherwise, people willingly practice immorality. Moreover, immorality of the criminal brings results. For immorality is clever whereas, 'playing by the rules' and being moral is stupidity. For criminals can dominate states and nations. So immorality is good.

Socrates reasons that to be clever at something like music, is to be good. Not to play music well, is to be bad, a bad musician. Having knowledge is good therefore. Knowledgeable people do not compete against other knowledgeable people. A knowledgeable person is clever and clever is good. Knowledgeable people do not compete against other knowledgeable people. Likewise the moral person does not seek to compete with or dominate other moral people. They do seek to compete and dominate the immoral. On the other hand, the immoral person does seek to compete and dominate both moral people and other immoral people.

Therefore, being moral is to be knowledgeable; knowledgeable is to be clever and clever, as opposed to being stupid or ignorant-is good. Playing by the rules of moral knowledge is good as all will know and agree, on what is Good, Bad, Right and Wrong.

Not possessing this knowledge, the ignorant, the immoral, will compete against their fellow ignoramuses on the promotion of what is Good/ bad etc. A band of thieves, pirates will not therefore, be able to function together as a band. They will harm and wrong each other. Immorality breeds conflict and, hatred and antagonism. The view of might is right belongs here: it will bring nothing but conflict etc. Hence states and nations run by criminals will collapse into anarchy. This is the point made much later by Thomas Hobbes in his work The Leviathan.

Possessing Knowledge, dispels the view that Might is Right. Knowledge, according to Plato, will bring right action. Right is not might nor is might is right.

Jean-Jacques Rousseau

In Book One of The Social Contract, Rousseau examines the contention that the strongest must rule by means of Right: Right is might. If force, strength is a physical power-what Moral power does it possess? There is only might, not right. A naturalistic fallacy is committed in maintaining the proposition 'Might is Right'. I can see the fist and the gun, what they can do but, nowhere, do I see 'Right' or 'Rightness.

To yield to an act of Might is an act of cowardice, fear, expediency but it is not an act of free consent or an act of Right. Therefore, it cannot be moral, have a moral sanction. Obeying might simply because it is the force of the strongest contains no moral prescription or 'ought' as to why we must obey. At most, it only demonstrates the fact that we do. If there is no moral 'ought' then might has no 'Right'; they are mutually exclusive. If it cannot be justified by a moral argument then there is no 'Right' of the strongest or the mightiest. Therefore, the contention 'Might is Right' has no moral force.

Martin Jenkins

back

(47) Sabyasachi asked:

What did Nietzsche mean by 'amor fati' ?

---

'My formula for greatness in a human being is amor fati, that one wants nothing to be other than it is, not in the future,not in the past, not in all eternity. Not merely to endure that which happens of necessity, still less to dissemble it-all idealism is untruthfulness in the face of necessity-but to love it...'

Friedrich Nietzsche 'Why I am so clever' Ecce Homo

Amor Fati is literally love fate, love your fate. Whether this is Nietzsche's judgement about his life, who he became is debatable. After all, he is writing autobiographically in Ecce Homo. Following Spinoza, a person becomes who they are through a complicated interaction of forces of Will to Power, drives, emotions. [Or for Spinoza, Encounters between bodies of differing degrees of motion and rest Part II, Ethics For Motion and Rest, read Will to Power. There are similarities between Spinoza and Nietzsche to be explored elsewhere]. The person reacts to such forces , drives, emotions, according to the forces that constitute his/ her body. Thus I react to certain encountered circumstances and not to others due to past encountered circumstances reacted to/ not reacted to, that made me become who I am and how I will react to future encountered circumstances. Hence, in Ecce Homo, Nietzsche is [genealogically?] recounting how he became who he did. And he affirms it: amor Fati.

Eternal Return of the Same: Existential Imperative

Loving fate immediately links to Nietzsche's concept of The Eternal Return or Eternal Recurrence: 'the highest formula of affirmation that can possibly be attained'. In #341 of Book IV of The Joyous Science [1882] Nietzsche introduces the concept. The passage is too long to quote, but essentially, Nietzsche poses the question to the reader: What if this life you have lived and live, you will have to live again and innumerable times again. What

would be your response? Would you regard it as a curse or would you embrace it? This puts your life starkly into question. It may provoke you to change your life, to do what you have always wanted to do and not deny it, or shy away from it.This question becomes an existential imperative. It calls for active activism.

Eternal Return of the Same: Ontological Determinism

This is at odds with amor fati and another reading of the Eternal Return. In Of the Vision and the Riddle, Part III of Thus Spoke Zarathustra, Zarathustra has a vision in which he poses the question whether what is happening now, has happened infinite times before and will so again and again. The Eternal Return here, is an ontological determinism. What happens is going to happen, necessarily because the fabric of reality determines it to be so.

'Are not all things bound fast together in such a way that this moment draws after itself, all future things?' [ibid]

The ontological determinist reading of Eternal Return of the Same sits, for me, uncomfortably with the rest of Nietzsche's works. These call for the revaluation of Judeo-Christian values and the revitalisation of life. Why bother to call for this is everything is determined?

Further in the vision, Zarathustra finds a man choking with a snake hanging from his mouth. It had crawled into his mouth when he was asleep. Zarathustra fails to tug the snake away, so he cries for the man to bite. He does and the snake falls. The shepherd is then transformed: surrounded by light he joyously laughs unlike laughter before heard on earth.

So, if all must be, why would Zarathustra actively try to remove the snake? Maybe it was meant to happen -asserting ontological determinism. Maybe the snake snook into the shepherds mouth when asleep and it would then be taken as a given, he would die: for he was not aware. Hence the ontological determinist view of fate. It too teaches us to be asleep, to be passive in the face of life, to accept our fate reactively. Alternatively, Zarathustra, by actively acting, helped remove the snake and saved the shepherd. The snake represents the ontological determinist view of Eternal Return. A view contradicted by Zarathustra's active intervention. Hence the transformation of the shepherd is the transformation of life-the Existential Imperative.

Over to you Sabyasachi, Amor Fati!!!

Martin Jenkins

back

(48) Jonathan asked:

It would be impossible to perceive a nonexistent universe. There would be nobody there to perceive it ! For equally obvious reasons it would be impossible to perceive a universe in which there is no life or subjective experience. That is simply because all perceptions require a subject and an object.

So the only available perception is one of ourselves (or something with subjective experiences) in a universe (or in relation to any object which is there to be perceived).

Doesn't this prove that we must exist ? It seems to prove that no other observation of the universe is possible except one in which we exist. It says nothing of the specific conditions of our existence, only that it is logically impossible to perceive our own nonexistence.

QUESTION: Does the fact that it is logically impossible to perceive our own nonexistence also mean that it is logically necessary that the universe exists ?

WHY I ASK: If the answer is yes, doesn't this eliminate the need for metaphysics in general ? If something is truly logically necessary then it requires no further account than a proof of its logical necessity. This deflates 'why is there something rather than nothing ?' as a springboard for gods and other metaphysical objects .It deflates every variation of the cosmological argument and the recent 'fine tuning' arguments.

It (the anthropic principle, look it up) is a subtle idea with devastating consequences. It has never been a part of humankind's mental equipment even that of intellectuals, but its influence is gradually being felt in philosophy and other disciplines.

---

Get rid of metaphysics. Great idea ! But, sadly, your arguments don't quite do the trick.

Your comments about whether you or whether the universe necessarily exist commit the fallacy of equivocation. Specifically, illegitimate conflation of two different scopes for the modal operator ('necessarily'). An example relating to my friend John illustrates:

John is a bachelor

Therefore, necessarily, John is unmarried True but trivial (true by virtue of the meaning of the word 'bachelor')

Compare:

John is a bachelor

Therefore, John is necessarily unmarried False, he just happens to be so (is contingently so), he would have been married if he'd accepted that proposal he got in Hawaii last year)

Similarly, as regards you, we can say that given you exist, then necessarily you exist, a trivial logical truth. But we cant say that given you exist, then you necessarily exist — you (and I) exist contingently. We might not have existed. Consider the stupendous fluke by which you exist. Of the millions of generations of your ancestors going back 4 billion years, not a single one of these organisms failed to reproduce.

Similarly, the universe is contingent, it's actual existence entailing no more than: necessarily [the universe exists]; not: the universe necessarily [exists].

As for the anthropic principle, the weak version (WAP) simply points out that the observed values of physical/ cosmological quantities are restricted by the requirement that there exist sites where carbon-based life can evolve and by the requirement that the universe be old enough for such life to have already evolved. It's really a Bayesian approach as to the prior probability of what we will find when we investigate the universe. The strong anthropic principle (SAP), seems to me invalid, not to mention the participatory one (PAP), final one (FAP), and somebody suggested these more speculative versions might be called the Completely Ridiculous Anthropic Principle (CRAP). I doubt that any of these will have the devastating consequences you hint at.If you haven't read it, 'The Anthropic Cosmological Principle' by Barrow and Tipler (OUP1988) is an exhaustive, entertaining, exasperating and informative account, tracing the history of the idea from the Presocratics to its modern 1950s formulations, and discussing its relevance to physics, cosmology, quantum mechanics, biochemistry, extraterrestrial life possibilities and the future of the universe.

Craig Skinner

back

(49) Richard asked:

Do you not think that much controversy over morality is abortive being about particular acts rather than the basic philosophy that advises attitudes to those acts?

That is to say: there are three essential attitudes to ethical values moral codes.

There is what may be called the Authoritarian, which essentially means 'God' in which obedience to that authority is the definition of virtue, and anything else is vice. The good of people is in obeying that authority.

The second type is the Libertarian or Relativist. For whom virtue is in obeying ones own conscience, and what is right for ones self, with no external standard by which acts can be judged.

The third type is what may be called Altruist. For whom there is an absolute standard of ethical values, although they may not be clearly determined. The good of all society and every individual. That ethical value is prior to any authority, or 'God'.

---

Yes, Richard. I would agree. Most of the visible moral debate appears to be over the morality (or immorality) of particular actions, rather than over the underlying moral principles that result in those disputed opinions.

So, for example, we see in the press, on TV, and over the Internet, lots of stories about the good things or bad things that some people are doing, and how other people are so upset by those actions. But we never seem to see any discussions about the underlying moral principles that appear to guide the superficial attitudes that are being expressed over specific actions. Is the issue of the moment abortion, right-to-life, Arizonas new illegal alien law, taxation, tax breaks for the wealthy, the effects of welfare on the poor, etc., etc., etc. All we seem to see is one person, center stage, yelling at the top of their lungs that whatever is going on is a good thing, while someone else is yelling at the top of their lungs that whatever is going on is a bad thing. No one seems at all curious about the underlying moral principles that motivate those overt (and noisy) expressions of attitude and whether those principles are reasonable, or true (whatever true might mean in that context).

You would think, wouldn't you, that if two (groups of) people are so noisily (and sometimes even violently) disagreeing with each other about the morality of some action, there might be some benefit in exploring their differing moral principles. But no one seems at all interested in the possibility that at least one of the positions might be based on a more fundamental error about the proper moral principles.

The general attitude seems to be, however, that because we have (to various degrees in various countries) the freedom of speech and freedom of religion, it is permissible for anyone to believe and to say whatever they please even if what they believe or say is demonstrably false, and demonstrably contrary to the general welfare. The generally accepted public attitude is solidly relativist the truth for you is whatever you choose to believe it is, and each person has the right to believe and proclaim their own notion of truth.

I agree that it is absurdly stupid. But until people in general wake up to the fact that truth is not a relative matter, things will not change. And people will never wake up to this basic fact, until we stop teaching Relativism in the schools. We don't teach it in Math, or the Sciences. It is time we stopped teaching it in Social Studies as well.

A separate topic, I do not agree with your three-way split of attitudes towards moral codes. I think your characterization of the Libertarian and Altruist labels is quite incorrect. I would suggest as an alternative split the following:- 1. Authoritative Rules/ Divine Command 2. Subjective Opinions/ Cultural Relativism 3. Objective Standards/ Natural Law

Authoritative Rules

If you believe that the 'right thing to do' is specified by rules of behaviour (e.g. 'Thou shalt not . . .', 'Do unto Others . . .', etc.), then you believe in an 'Authoritative Rules' or 'Divine Command' notion of Ethics. The ethical thing to do is that which most closely conforms to the relevant rules. It is unethical/ immoral/ sinful to behave contrary to or in violation of the rules.

Many different sources of Authority for these rules have been cited by various philosophers. The most obvious, of course, is some Deity — as in the Ten Commandments of Moses, and the 'Word of God' captured in print in the Bible, Koran, or Torah. For some people, the words of Buddha or Confucius fulfill that role. And even, for some people, the works of Karl Marx, Adolph Hitler, Lenin, or Mao Ze-dong.

A key challenge for this notion of Ethics is that it is impossible to convince someone by logical argument that one source or one particular interpretation of the commands of Authority is any better than another. Any philosophical discussion approached from the perspective of an Authoritative Rules notion of Ethics will almost always degenerate into emotional posturing. In a social environment, therefore, 'persuasion' in ethical disagreements almost always degenerates into ad hominem attacks or outright coercion.

A notable characteristic of most flavours of Authoritative Rule Ethics is an intolerance of any curiosity about the rationale for, and justification of, any of the Rules. A Rule is the Rule because the Authority says so — and no further response is available. 'Life will be Better' if you adhere to the Rules — 'Trust me!', because no explanation of why or how is available or permissible. The popular term for this sort of Ethics, when encountered outside of mainstream religions, is 'cult'. Although followers of main-stream Authoritarian Rule Ethics would vociferously object to that characterization.

Subjective Opinion

If you believe that the 'right thing to do' is a matter of personal or public opinion, then you believe in a 'Subjective Opinion' notion of Ethics. The ethical thing to do is whatever you 'intuit' is the right thing to do; or whatever you feel most emotionally 'good' about; or whatever you feel is the consensus of other people's opinions on what you should do.

Many different sources of opinion have been cited by various philosophers. The most obvious, of course, is the individual's personal opinion, feeling, or emotional attitude. But other approaches have included the Social Consensus, social customs, habits or thoughts, and cultural norms. There is, of course, a fair degree of overlap between Subjective Opinion Ethics and Authoritarian Rule Ethics.

A key challenge for this notion of Ethics is similar to that of an Authoritarian Rule notion. It is impossible to convince someone by rational argument that one opinion, or source of opinion, is any better than another. Any philosophical discussion approached from the perspective of a Subjective Opinion notion of Ethics will almost always degenerate into emotional posturing. In a social environment, therefore, 'persuasion' in ethical disagreements almost always degenerates into ad hominem attacks or outright coercion.

Adherents of any of the various flavours of Subjective Opinion Ethics also must deal with two additional challenges. Firstly, there is nothing within such a system of Ethics that requires the various opinions be logically consistent and not mutually contradictory. And secondly, there is nothing within such a system of Ethics that guides the adherent in choosing which of a set of contradictory opinions to apply in any situation, or protects against a ludicrous misapplication of some opinion. It is impossible, therefore, to employ logical reasoning or rational analysis in an exploration of the consequences and implications of the relevant opinions. Consequently, in practical application 'Moral' behavior is almost always the result of a non-rational and non-logical subjective opinion as to which (or whose) opinion to apply when, where, and how. As I mentioned above, it would appear that as a society, we operate on the basis of an assumption that Subjective Opinion Ethics is the proper basis for ethics.

Objective Standards

If you believe that the 'right thing to do' is whatever will best meet some standard, or fulfill some purpose, or achieve some goal then you believe in an 'Objective Standards' notion of Ethics. The ethical thing to do is whatever is most likely to be most efficient and effective at meeting the standard, or fulfilling the purpose, or achieving the goal. Within Objective Standards Ethics the concepts of 'Good' and 'Bad' are established by appealing to some objective 'Natural Law' that is outside of and independent of people, societies, or authority.

Many different Purposes have been cited by various philosophers. They have varied from 'eudaimonia' for Aristotle, 'happiness' for the Epicureans, and 'pleasure' for the Hedonists, to 'Utility' for the Utilitarians, and 'Life as Man qua Man' for the Randian Objectivists, among others.

The great advantage of Objective Standards Ethics (regardless of the particular standard that is adopted), is that rational discussion can be profitable. That is why, for example, the utility of Utilitarianism is employed as a rough approximation in many debates about ethical issues. In a social environment, therefore, 'persuasion' in ethical disagreements almost never degenerates into ad hominem attacks or outright coercion.

But of course, you will never convince anyone that their own preferred understanding of the proper basis of Ethical analysis is not the correct, proper, and only reasonable one to adopt.

Stuart Burns

back

(50) Whitney asked:

What is life?

---

An excellent question, Whitney. But unfortunately, one better addressed to a Biologist rather than a Philosopher. Just what it is that separates life from non-life is the subject of much debate. There is no universally accepted definition.

For an excellent introduction to the biological debate, I would highly recommend What is Life by Erwin Schrodinger. It is a little dated (1944), but is a very readable introduction to the possible answers to your question.

If you are looking for a more up to date, and a deeper technical discussion of the question, then I would recommend What is Life by Lynn Margulis and Dorion Sagan. It was published in 1995. It also deals with some of the philosophical issues raised by your question.

Stuart Burns

back

(51) Daisy asked:

What is necessary for personal survival?

---

I guess that Daisy isn't looking for an answer along the lines of, 'a compass, a pen knife, a torch, a box of matches, and a can of Mace.'

This is one half of a question which analytic philosophers call, 'the problem of personal identity.' I won't say whether this is the easier or more difficult half. The problem of personal identity concerns the necessary and sufficient conditions for the identity of person A at t1 with the (allegedly) same person A at t2. This isn't a question that arises in everyday conversation. However, there are particular circumstances where the issue of personal survival becomes urgent: Can a person be said to 'survive' if they suffer total amnesia? or in an advanced stage of Alzheimer's disease? or if they fall into a permanent coma?

Added to these relatively few practical challenges — which would hardly justify the vast industry of philosophers who've worked on the topic of personal identity — are all the resources of science fiction. One feels, and as an analytically trained philosopher I think this feeling is largely correct, that if you can't say whether a person 'survives' in this or that imaginary scenario, if you are puzzled and are not sure what to say, then there is an understanding which you lack — an understanding of what it is to be a person. Maybe in everyday life (modulo the medical cases) we can get by perfectly well without this understanding. But that's just part of the genius of philosophy: it poses questions you never thought to ask.

But once you see the question, you are gripped.

We will consider some of these problem scenarios in a minute. But actually I don't think this is all of it. There is a deeper question about survival and identity, which I considered in my book Naive Metaphysics. I reached the scary conclusion that there is no such thing as survival. The 'I' — the essential I — does not survive from one moment to the next:

[T]he subjective standpoint is a world every bit as rich and detailed as the world of the objective standpoint. Yet its reality hangs by the slenderest possible thread. It is real because I take it to be real, and only for so long as I take it to be real. By the slenderest possible thread the objective world is held at bay, yet no power in the universe can break that thread, so long as I exist. (Chapter 8, p.101)

Only something that continues through time can cease to exist. Yet my subjective world, as a reality constituted by its own appearance, only appears to continue; and that appearance itself is something which neither continues nor fails to continue. My subjective world can never die, can never cease to continue, for with every new moment it is as if it had never existed, and will continue no longer than that very moment. (Chapter 9, pp.119-20)

But let's get back to basics. One of the things I tell my students is, if you get stuck, try to think like a detective. Go back to the very beginning and don't assume anything. So I won't make any assumptions about what a 'person' is, or might be. Instead, I will list all the things, or kinds of thing, that might be necessary for personal survival. Here's the list:

  1. Something physical
  2. Something psychological
  3. Information (e.g. pattern, structure)
  4. Something metaphysical (whatever that means)
  5. None of the above

As you see, I'm not leaving anything to chance. At this point, one can't imagine what might be covered by item 5. but you never know.

However, I will make this assumption: if a thought experiment or science fiction scenario inclines us to say that survival would, or would not have occurred, then that should be considered as a datum, so long as we are unable to find a compelling argument against that intuition.

1. Is something physical required — logically required — for personal survival? My intuitions tell me, no. It seems to me perfectly possible that (as in Anthony Quinton's much discussed thought experiment in his article 'Spaces and Times' Philosophy 37 1962: 130-4) I could wake up tomorrow morning on Planet X, and know who I was, in the absence of any evidence of something physical having made the journey from Earth to Planet X, or indeed evidence that Earth and Planet X were in the same universe.

2. Is something psychological required for personal survival? John Locke thought so. Indeed on Locke's account memory is not only necessary but also sufficient for personal identity. Locke's theory of personal identity appears to confirm the intuition expressed in the previous paragraph. What matters is consciousness of my own identity. The essential thing, when we praise, or punish, is that the person be aware that such praise or punishment is merited, which they cannot be in the absence of memory of what one did in the past.

I don't agree that this is an inviolable intuition, and as evidence for this I put forward the case of Cypher in The Matrix:

Cypher: I don't wanna remember nothing. Nothing, you understand? And I wanna be rich. You know, someone important … like an actor.

Agent Smith: Whatever you want, Mr. Reagan.

Cypher bitterly regrets his decision to take the red pill. He's sick and tired of 'reality'. He feels duped by Morpheus. What's interesting about this is that the scriptwriters evidently thought (and I agree with their intuition) that it is reasonable that someone might wish to have a total memory wipe. I wouldn't, but some would. From Cypher's perspective, this isn't death, not at all. He will be the famous actor, feasting on juicy digital steaks, living a life of ease and luxury. He will survive, even though the famous actor has no memory of Cypher's present existence. (I can't help wondering if the name chosen for this character, 'Cypher', isn't a sly joke on the part of the scriptwriters.)

3. It would not be an unreasonable inference from 1. and 2. that what is necessary for personal survival is either something physical (as in Cypher's case) or something mental (as in Quinton's thought experiment). Analytic philosophers don't like such 'disjunctive' answers. When you ask for necessary or sufficient conditions, a disjunctive, 'either-or' answer just sounds like equivocation. Either we're talking about 'physical survival' or we're talking about 'mental survival'. But that would be missing the point. In the case of the necessary conditions for personal identity, there is just one thing we are interested in: survival of what matters.

There is a way around this, however. One could say that 'what matters' is the continuity of information, a certain unique structure or pattern that can be carried either in a physical or a mental medium, or both. However, this is not a view that is universally held. While the idea of reincarnation or rebirth appears to require continuity of some aspect of the consciousness of the person who dies, there is no agreement amongst different schools (e.g. of Hinduism or Buddhism) on what exactly that aspect is. There appear to be some who have argued that what survives is the sheer point of view as such, distinct from all psychological attributes or contents of consciousness. That's how I understand the notion of an individual 'atman'.

I think this is sufficient to warrant a 'pass' on the question of transmission of information, as necessary for personal survival. To massage this intuition, one might envisage a combination of the Quinton and Cypher thought experiments. Imagine that Cypher wakes up on Planet X and spends a few years there, where his existence is totally miserable. Then Agent Smith offers him a splendid life on Planet Y, where all his memories of Planet X will be wiped.

4. At this point you will be champing at the bit to argue that in our new thought experiment, Cypher must believe that what he is essentially is an 'atman', a sheer point of view, which exists now on Planet X and will exist on Planet Y, even though nothing physical or psychological survives.

My response is, believe this if you like. It just doesn't make any sense to me. I lose the thread at this point. But I can't rule the possibility out because I just don't know what I'd be ruling out. It's an 'unknown unknown'. I have the feeling, though, that if Daisy gives this as her answer (I'm assuming, Daisy, that you have an assignment to write) she won't be very popular. Perhaps a better line would be to argue for an aporetic conclusion, as I have done. We've tried all the alternatives and none of them work, end of essay.

5. By now, you may have guessed where this is heading. We've tried all the alternatives and none of them is satisfactory. I'm happy to accept that not everyone will agree with me about this. At any rate, I'm not satisfied. 'Person' is a concept with a valid use within our linguistic community, our moral, legal and political practice. The philosopher's 'problem cases' aren't really problems. But they are to the philosopher.

Then again, if you are looking at the concept of a person from a philosophical standpoint, it is arguable that we put far too much emphasis on identity (see Derek Parfit's acclaimed book Reasons and Persons, 1984). However, I think Parfit overstates the case. I wouldn't like to live in his preference utilitarian utopia where the notion of being a person, or of personal identity or integrity is no longer considered 'important'.

We need to look again at the question of 'what matters'. When you consider the sufficient conditions for personal identity (something I haven't done here), it becomes apparent (as some philosophers have argued, e.g. David Lewis) that the best theories we have allow for multiple survival. E.g. I go to sleep, and two — or a hundred and two — GKs wake up, each individual version of GK fully satisfying the criteria for personal identity, according to our best theory. In this kind of scenario, our intuitions go AWOL. We don't' know what to say. We feel drawn to insist on something 'metaphysical' (such as a Cartesian soul substance, or Buddhist atman) which exists in one, and only one of the GKs, but common sense and logic tell us that there simply is no foothold here for picking out one of the GKs from the multitude in order to bestow this special honour.

So what really matters? I don't know about you, but I want to know is what it is by virtue of which it is true that I am GK (Thomas Nagel's 'I am TN'). That's what matters. The fact that I am here, that there is a world for me, when it is perfectly conceivable (as I would argue, you may disagree) that the world might have been exactly as it is now, in the absence of I. In other words, the existence of my subjective world is a contingency which depends on nothing at all. It's just a brute fact, evident to me now as anything can be, and yet nothing in my knowledge or experience justifies or accounts for the existence of my subjective world one single moment from now.

I realize that many will regard this conclusion as fantastical. I — the essential or ultimate 'I', the thing that matters — do not survive. I will not survive to see this blog post finished. Not even to see the next sentence that GK will write. I remember once my old Prof David Hamlyn (who did a writeup for my book) commenting in a letter that he sometimes worried that I took Plato's advice to 'follow the argument wherever it may lead' beyond the point that most would consider reasonable. I don't have a reply to Hamlyn, except to say, 'that's just me, innit?'

Geoffrey Klempner

back

(52) Jack asked:

Do we truly have free will?

---

That very much depends on just what it is you mean by free will.

The generally accepted attitude towards modern living, is that the world and the things in it that we encounter, behave and react to us in predictable and comprehensible ways. In short, the generally accepted attitude towards everything (other than people), is that they are governed by a collection of causal laws. Given a specific set of circumstances, the future behaviour of things is pretty much predictable. In philosophy, this attitude is called causal determinism the principle that all events have a cause. All effects are in principle (if not always in practice) predictable if we understand the cause.

Towards people, two alternative positions are therefore possible. You can take the position that people are just more things that obey the rules of causal determinism. Or you can take the position that people are exempt from the rules of causal determinism.

The latter view, where people are exempt from the rules of causal determinism, you need to define just how it is that people whose bodies are made of the same sorts of materials as are other things that are ruled by causal determinism can be exempt. There are two different alternatives that show up in the literature.

The first is the approach of Dualism. This is the position that whatever constitutes our mind is not a physical thing, and therefore is not constrained by the rules of causal determinism. This is the approach made famous by Rene Descartes. It does offer the advantage that such an immaterial mind can be seen to have absolute freedom of will. There are no material constraints on what such a mind might will. The difficulty faced by dualism, however, is explaining just how it is that an immaterial mind might influence, and be influenced by, the very material body. The currently common reaction to this latter problem is that the free will offered by dualism comes at too high a price.

The second way of exempting the human mind from the rules of causal determinism is to look to a source of indeterminism within physics. Some philosophers suggest, for example, that the mind can be influenced at times of uncertainty, by some form of quantum randomness. The difficulty that such an approach creates is that the notion of free will that is generated is limited to those situations where the mind involved finds itself in a critical balance between different alternatives. The freedom involved is then the freedom of quantum randomness. Of course, this raises the additional difficulty of explaining just how a decision made as the result of some random event is truly a free decision of yours.

If the human mind is not exempt from the rules of causal determinism, then there is a real challenge to the traditional notion of free will. Either we do not have free will. Or the kind of free will that we do have is not the kind traditionally associated with the concept of free will.

The first position is that taken by so called hard determinists. They argue that since everything in nature, including the human mind, is governed by the rules of causal determinism, we do not have free will. Free will is just an illusion. A necessary illusion, but an illusion none the less. It is an illusion born of the fact that we do not, in actual fact, have the necessary information to be able to predict the future course of events except in a very rough approximation. Hence, the future course of events looks to us as if it were the product of our free will.

The second position is that taken by so called Compatibilists. They argue that our traditional concept of 'free will' is so poorly defined that we do not understanding what it is that we mean. What we ought to mean by free will is that kind of freedom of will that is admitted by the hard determinists. Our choices and decisions are the caused effects of our beliefs, experiences, needs, desires, hopes and dreams all according to the rules of causal determinism. But since we do not have sufficient information to be able to predict the future course of events except in a very rough approximation, we might as well consider that the future course of events is the product of our free will. The fact that when we get down to the nitty gritty detail, there is no freedom is irrelevant because we do not live and choose at that level of detail. We live at the level where information about causes is never sufficient to predict the effects. This gap gives us room for the concept of 'free will' — we are 'free' to choose, at this level, within the 'uncertainty gap' provided by our lack of information.

So back to my question just exactly what do you mean by free will??

Stuart Burns

back

(53) Bob asked:

If 'God' is defined as 'an omnipotent being' does that mean that everything must be exactly as God wills it? Can there be limits on God other than God's own nature?

---

Yes it does, if God created the world out of nothing then in some sense everything must be as he willed it.

As far as limits go, it depends on what you mean by limits. God cannot tell you what the greatest prime number is, he cannot tell you how to arrange all the decimal numbers between 0 and 1 into a countable list. God cannot prove that our system of mathematics is both consistent and complete. However God has willed these things also.

Shaun Williamson

back

(54) George asked:

What is the best way to learn complex theories from Kant, especially with things such as synthetic a priori and how will Philosophy help me in terms of reasoning and future career prospects?

---

George, while you do not say what type of career you aspire to, it must be said that a degree in Philosophy does not automatically offer a direct link to employment in the same way as a degree in Law or an Engineering degree might (unless of course one intends to go on to teach Philosophy). That being said, skills such as the ability to think logically, to express oneself clearly and articulately, both in the spoken and written word, and people management — all philosophy related skills — are all abilities that employers value in potential employees — and all abilities most useful if one's ambition moves one in the direction of teaching, writing (both fiction, non-fiction and/ or journalism) and other academic pursuits. Personally I have not found a love of wisdom and the search for knowledge an impediment to finding employment — which over the course of my life has included running a small but successful business, being a librarian, a musician, a civil servant, a teacher of English, and of course, a Philosophy lecturer. It might interest you to know that an acquaintance of mine who is a successful businessman insists that his love of Philosophy not only plays a significant role in the day to day running of his businesses, but it helps him to find a balance between work, play and, most importantly, family.

With regard to your interest in Kant, while there are many excellent introductions to Kant available in good academic bookshops, until you discover one yourself, you may find the following piece on synthetic a priori judgements and other aspects of Kant's philosophy useful. (for more see www.tonyfahey.com)

Immanuel Kant: Synthetic a priori judgements

In his essay An Essay Concerning Human Understanding (1690) John Locke declared that the mind was a tabula rasa — a blank slate. Human beings, he argued, are born with nothing other than the capacity to experience through the senses. The knowledge we acquire is not due to any innate power to reason, but by the accumulation and organisation of experience. David Hume (1711-1776), one of Britain's most eminent empiricists, followed Locke's argument. 'We know the mind', said Hume, 'only as we know matter: by perception'. Hume maintained that the mind is not a substance, an organ of ideas, but an abstract name for a series of ideas, memories, and feelings, which all have their source in experience.

The German philosopher Immanuel Kant was impressed by the Empiricist argument that experience is the basis of knowledge. Indeed, he claimed that reading Hume caused him to awaken him from his 'dogmatic slumber'. However, he could not accept that all knowledge was derived from experience. 'Though all our knowledge begins with experience', he said, 'it by no means follows that all arises out of it'. In 1781, in response to the claims of Empiricism, Kant published his famous Critique of Human Reason; his ambition was to show pure reason's possibility, and to exalt it above the impure knowledge which comes through the channels of sense. By 'pure reason' Kant means knowledge that does not come by way of sensory perceptions. There is knowledge, he argued, which, though it may derive from experience, is understood to have its source in other than experience: knowledge that is inherent in the human mind; knowledge which is a priori. In the Critique of Pure Reason Kant relies extensively on the term 'synthetic'. By 'synthetic' he means knowledge which adds to, or extends previous knowledge. It was Kant's contention that there are, in philosophy, judgements that are both synthetic and a priori.

David Hume argued that metaphysical propositions are neither empirical nor analytical. That is, they are neither verifiable nor falsifiable by experiment or observation, nor or they propositions whose denials are self-contradictory. For example, propositions such as 'God exists' or 'Man is morally responsible' are propositions which are plainly neither empirical nor analytical. Hume's argument is that since metaphysical arguments are neither verifiable nor falsifiable, they must be meaningless. Only empirical or analytical propositions, he said, have meaning. Kant rejected this argument. He believed that there is a classification which falls outside Hume's two classes of empirical or analytical. This classification is not of propositions per se, but of judgements. By 'judgements' Kant means propositions asserted by somebody. He was not concerned with the proposition in itself, but with the judgement by some person to that effect. Every judgement, Kant maintained, is either analytic or synthetic:

In all judgements in which the relation of a subject to the predicate is thought (I take into consideration affirmative judgements only, the subsequent application to negative judgements being easily made), this relation is possible in two different ways. Either the predicate B belongs to subject A, as something (covertly) contained in this concept A, although it does not indeed stand in connection with it. In this case I entitle the judgement analytic, in the other synthetic. (CPR. 1929, A7, B11, p. 48)

An analytic judgement is one whose truth follows merely from the meaning of he words used to express it, a judgement in which the concept of the predicate is 'included' in that of the subject. Thus, the judgement 'an equilateral triangle is a triangle' is analytic: it follows from the meanings that we give 'equilateral triangle' that the second term 'triangle' applies to everything that falls under the first. A synthetic judgement, in the other hand, is one that goes beyond the meaning of the subject terms, and brings some new idea or information not already contained in the subject: it is a synthesis of two different notions, one being the subject about which the other, the predicate, is asserted. For example, if 'I say the sun is shining, but the day is cold', I am making a judgement which is the synthesis of two different ideas. With a synthetic judgement, then, the predicate must contain some information not contained in the subject, whereas an analytic judgement merely elucidates the meaning of terms but is otherwise uninformative.

It was Kant's contention that a judgement is either a priori or a posteriori. A judgement is a priori it it 'is independent of all experience and even all impressions and senses'. (CPR., B2, p. 28) A posteriori judgements depend logically on other judgements which describe experiences or impressions of sense. It is not only judgements that describe a particular sense-impression that are a posteriori, even general judgements may be logically dependent on such descriptions and therefore a posteriori. For example, the judgement that 'all bodies, if deprived of support, fall downwards, is a posteriori because it entails the description of particular experiences.

Thus, it seems that Kant's classification offers four possibilities: (i) synthetic a posteriori, (ii) synthetic a priori, (iii) analytic a priori, and (iv) analytic a posteriori. However, analytic a posteriori must be discounted as there can be no such judgements. An analytic proposition, being about the meaning of terms, does not give any other information. An a posteriori proposition, on the other hand, does. This leaves us with three classes of judgement which Kant holds to be not only possible but patently obvious: synthetic a posteriori, synthetic a priori, and analytic a priori. All analytic judgements must be a priori since they merely elucidate the meaning of the terms and are thus logically independent of judgements describing sense-experience. It would seem to follow that if all analytic judgements are a priori, then all synthetic (non-analytic) judgements are a posteriori. However, Kant holds that there is a third classification of judgements whose predicates are not contained in their subjects, and are yet logically independent of all judgements describing experience: synthetic a priori judgements.

For a synthetic a priori judgement to be possible, says Kant, they must contain some information not purely of a logical nature, while at the same time they must rely on empirical information for their truth. But where can one find such evidence? Kant maintains that mathematics and physics are precisely of this character. They are both possible and synthetic a priori judgements. There are, he says, two kinds of mathematical truths: geometry and arithmetic. Even the elementary arithmetical judgement that 5 and 7 equals 12 is a synthetic a priori judgement, for, says Kant, 'the concept of a sum of 7 and 5 contains nothing over and above the uniting of both these numbers into a single one' (CPR., B1, p. 37) this proposition is true, insists Kant, not because of the veracity of the definitions of terms involved, but also because it contains more information in the predicate '12' than in the concepts '7' and '5'. In order to join these two concepts together, says Kant, a kind of intuition is necessary which introduces something new in the conclusion. In other words, we can take '7' and '5' as two separate entities, but without 'intuition' we would never arrive at their 'synthesised' sum to form a completely separate conclusion. 'All mathematical judgements, without exception', says Kant 'are synthetic'. (ibid., p. 52)

The same is true of geometrical truths. For example, if we take the proposition 'A straight line is the shortest distance between two points', we note that the concept of a straight line does not represent any suggestion of it being the shortest route, and yet the judgement is a universal and necessary truth. Physics, says Kant, also holds such truths, which we see in the proposition, 'Every event has a cause', which is both synthetic and a priori. This evidence, argues Kant, shows that in addition to sensory experience, there are also certain relating activities of mind itself: activities on which a priori truths depend.

The mind, says Kant, receives data of the phenomenal world through sensory perceptions. However, in order to understand this information these sensory perceptions must be processed by certain conditions inherent in the human mind. As well as the 'intuitions' space and time, Kant lists ten categories which were meant to define every possible form of prediction: substance, quantity, quality, relation, place, time, position, possession, action, and passivity. These concepts (or categories) were reorganised to consist of four types: quantity, quality, relation, and modality. In short, everything we, as humans, experience we can be certain will be imposed within the a priori framework of the intuitions space and time, and subject to the law of causality — the law of cause and effect. These conditions, says Kant, operate as a formal apparatus to bind together a priori judgements. These functions are the pure concepts of synthesis which belong to the understanding a priori, and for which alone it is called pure understanding. The phenomenal world, says Kant, is a combination of something which our senses present to us and a priori conditions inherent in the human mind. The mind, then, determines the kinds of answers given but not the specific content, which only experience can provide. Space and time, and the law of causality, impose on the mind necessary conditions of both experience and knowledge, but the actual content arises out of something independent in us: before sensations can be known they must be brought into a unified consciousness, which thus is no mere additional sense, but an intellectual synthesis, presupposed by every possible experience.

According to Kant, the world, for humans, is not a datum given by some external power. It is not some objective fact 'out there'; it is a product of the laws of our own understanding, acting in no arbitrary way, but according to specific principles, which are not peculiar to our separate individuality. For Kant human experience gives a point of view for the interpretation of everything that we can know; between the world, and ourselves there is an inner identity. As human beings we have sensory experiences, that is, we perceive impressions of phenomenon from the outside world through the senses; these sensory impressions are thus shaped by conditions inherent in the human mind. In other words, the mind assimilates the information perceived through sensory perceptions, and the judgements it arrives at will conform to the a priori intuitions of space and time, and the law of cause and effect. In the case of synthetic a priori judgements, while the judgement may appear to derive from sensory experience, its validity does not depend on it. For example, I can enumerate 7 and 5 on my fingers and conclude, by sensory perception, that the sum total is 12; however, such judgements, according to Kant, exist as universal and necessary truths whether I have sensory experience or not. They are a priori but they are discovered by experience. That is, they are synthetic a priori judgements.

In examining Kant's view on synthetic a priori judgements we come to realise that, in Kant's view, there are two sets of elements that contribute to our understanding of our world. The first set involves external conditions, which we cannot know before we have perceived them through the senses. The second involves the conditions inherent in the human mind. Empiricism argues that the human mind is but a 'passive wax' which is pummelled and shaped by sensory impressions. David Hume had reduced the mind to little more than a sponge which absorbed impressions and formulated complex ideas, not by virtue of any innate power, but by force of repetition and habit. Kant refused to accept such a skeptical approach. While accepting that our knowledge of the world enters the mind via sensory experience, he rejected the notion that all our knowledge arises out of these experiences. If this is the case, the question arises as to from whence comes order. According to Kant, the world is ordered, not in itself, but in that the mind already contains certain innate power — laws, which impose an order on the data received through sensory impressions. The human mind, says Kant, assimilates these impressions and makes judgements on these perceptions by virtue of the power inherent in the mind. These powers allow the human mind to make sense of, and function in, the phenomenal world. Access to this world, then, is only that which our intellectual and sensory powers, operating in tandem, permit. In other words, our capacity to understand the world in which we live depends on the intuitions 'space and time' and the concept of cause and effect. It is within this framework that we can arrive at certain judgements which, while they derive from experience, they do not depend on empirical evidence to determine their validity: they are synthetic a priori judgements.

Kant's Transcendental Philosophy

For Kant the term 'transcendental' means knowledge that concerns the a priori conditions of knowledge.

However, 'transcendent' means 'going beyond' or 'being beyond'. According to Kant there can be no knowledge of anything transcendent. That is, there can be no knowledge beyond the limits of the world of experience.

Transcendental Idealism

By transcendental idealism Kant means the doctrine that appearances are to be regarded as being, one and all, representations only, not things in themselves, and that time and space are therefore only sensible forms of intuition, not determinations given as existing in themselves, nor conditions of objects viewed as things in themselves.

Essentially, Kant's Critique of Human Reason can be divided in two parts: The Transcendental Analytic and the Transcendental Dialectic. The Transcendental Analytic considers the a priori principles that determine the scope and validity of the operations of the human mind: it investigates the limitations of reason. The Transcendental Dialectic deals with the sophistries and illusions to which the mind is prone. That is, it examines the illusions of reason, which originate from reason's propensity to draw conclusions of things that are beyond its capabilities. As he says in the preface to the first edition of the Critique, 'Human reason has this particular fate that in one species of its knowledge it is burdened by questions which, as prescribed by the very nature of reason itself, it is not able to ignore, but which, as transcending all its powers, it is not able to answer' (CPR A vii) The questions the mind cannot ignore, and at the same time cannot answer are those related to the soul, the cosmos, and God.

As already said, Kant's arguments are intended to show the limitations of our knowledge. Transcendental idealism, then, holds that metaphysical knowledge: knowledge of God, of souls, and of substance, is ideal, not real. Rationalists held that such knowledge was real. Kant argued that we cannot have knowledge of the realm beyond the empirical. There are two reasons why this is so: two constraints to this knowledge. That is, inherent in the human mind are two a priori sources which confine our knowledge to things as they appear to us as derived from experience. They are: i) the receptive capacity, or sensibility, and ii) the conceptual capacity, or understanding. Sensibility is what Kant calls the 'Transcendental Aesthetic (the term 'aesthetic' being derived from the original Greek meaning 'to have feeling'). By this Kant means that the mind contains, a priori, the sensible 'intuitions' space and time. According to Kant, sensibility is the minds way of assessing objects. The reason synthetic a priori judgements are possible, is that space is an a priori form of sensibility. It is not possible to understand the object as an object unless we delineate the region of space it occupies. Without this a priori 'sensibility', we would not be able to ascribe properties to particular objects. Time, he says, is also necessary as a form or condition of our intuitions of objects. The idea of time, like our idea of space, does not derive from experience because succession and simultaneity of objects, the phenomena that indicates the passage of time, would be impossible to represent if we did not possess, a priori, the capacity to represent objects in time.

In short, for Kant, it is impossible to have any experience of objects that are not in time and space. However, space and time themselves cannot be perceived directly, so they must be the form by which experience of things is obtained. Subjecting the sensations to the a priori conditions of space and time, however, is not enough to make judgement of objects possible. Understanding must provide the concepts: the rules by which what is common or universal in different representations of things (CPR. A 106). 'Without sensibility no object would be given to us', says Kant; 'and without understanding no object would be thought. Thoughts without content are empty; intuitions without concepts are blind'. (ibid., B, 75) In the Transcendental Analytic section of his Critique of Pure Reason Kant argues that in order to think about the input of sensibility, sensations must conform to the conceptual structure that the mind has available to it. By applying concepts, the understanding takes the particulars that are given in sensation and identifies what is common and general about them. A concept of 'shelter' for instance, allows me to identify what is common in particular representations of a house, a tent, and a cave. Hume had argued that for a sort of association to explain how we arrive at causal beliefs. The idea of a cue ball in motion becomes associated with the black ball being struck and falling into a pocket. Under the right circumstances, repeated impressions of the second effect following the first produces the belief that the first causes the second.

According to Kant, the problem for Hume is that he failed to recognise that the association of ideas already presupposes that we can conceive of identical, persistent objects that have regular, predictable, causal behaviour. Being able to conceive of objects in this way presupposes that the mind makes several a priori contributions. Empirical derivation is not enough to sufficiently explain all of our concepts. I must be able to separate the objects from each other in my sensations, and from my sensations, I must be able to attribute properties to the objects. I must be able to to conceive of an external world with its own course of events that is separate from the stream of perceptions in my consciousness. These components of experience cannot be found in experience because they constitute it: they put shape on it and give it meaning. The mind's a priori conceptual contribution to experience can be enumerated by a special set of concepts that make all other empirical concepts and judgements possible. These concepts cannot be experience directly; they are only manifest as the form which particular judgements take. The special set of concepts is Kant's Table of Categories, which are taken. By and large, from Aristotle:

Of Quantity — Unity, Plurality, Totality Of Quality — Reality, Negation, Limitation Of Reality — Inherence and Substance, Causality and Dependence Of Modality — Possibility — Impossibility, Existence — Nonexistence, Necessity — Contingency

For Kant, this is the complete and necessary list of a priori contributions that the understanding brings to its judgements of the world. Every judgement the understanding can make must fall under the table of categories. And subsuming the sensations of space and time under the formal structure of the categories makes judgements, and ultimately knowledge, of empirical objects possible.

Transcendental Knowledge

As we have already seen, David Hume's view was that all knowledge derives from experience. In 1781 Kant published his reply to Hume in his Critique of Pure Reason. The Critique is a critical analysis of pure reason. Through this examination of reason it is Kant's aim to demonstrate pure reason's potential and to exalt it above impure knowledge which comes to us through the distorting channels of the senses. Kant held that while all knowledge begins with experience, it does not mean that all knowledge arises out of experience. By this he means the pure reason is knowledge that does not come from sensory perceptions: knowledge that is independent of all sensory experience, and knowledge that is the inherent nature and structure of the mind. Kant calls this type of knowledge Transcendental Knowledge. Knowledge, said Kant, is not all derived from the senses, as Hume believed he had had shown, but it is derived from both sense and reason.

However, it should be said that when Kant says that reason makes a contribution to experiences, it should not be mistaken for the argument of the Rationalist's that the mind contains 'clear and distinct' (transcendent) ideas such as 'God is a perfect being' and so on. The notion that there are such complete propositions is completely rejected by Kant. When he talks of the mind, understanding or reason possessing a priori ideas, he means that the mind provides a formal structuring that allows for the conjoining of concepts into judgements, but the structuring itself has no content. The mind lacks content until interaction with the world actuates these formal constraints. The mind contains a priori templates for judgements, not a priori judgements. Empiricists held that habit arises as a consequence of knowledge which happens after, or succeeding, contact with sensation: it is a posteriori. Rationalists proposed that knowledge is analytic: it attempts to anticipate experience by constructing a logical deduction from basic axioms. This results in the possibility of a priori ideas of reason. By considering both Empiricism and Rationalism, Kant created a sophisticated model of knowledge which overcame the simplistic notion of the subject either anticipating or reacting to experience. He called this sophisticated model Transcendental Knowledge.

According to Kant the mind is neither passive wax nor a blank slate, rather it is an active organ that moulds and coordinates sensations into ideas: an organ that transforms the chaotic multiplicity of experience into the ordered unity of thought. He calls this process, 'transcendental philosophy'. For Kant then transcendental philosophy is the study of the inherent structure of the mind: the innate laws of thought. Kant calls it transcendental philosophy because it concerns that which transcends sense experience. Transcendental knowledge, for Kant, is concerned not so much with objects, but with our a priori concepts of objects. There are two stages in the process of developing sensations into a finished product of thought. The first involves the coordination of sensations by applying to them the forms of perception, space and time. The second involves applying to them the forms of conception the categories of thought.

Hume had maintained that it was only the force of habit that made us see the causal connection behind all natural processes. Kant refuted this argument: the law of causality, he held, is eternal and absolute: it is an attribute of human reason. Human reason, he said, perceives everything that happens as a matter of cause and effect. That is, Kant's transcendental philosophy states that the law of causality is inherent in the human mind. He agreed with Hume that we cannot know with certainty what the world is like in itself, but we can know what it is like 'for me' — or for all human beings. We can never know things — in -themselves (noumena), said Kant, we can only know them as they appear to us (phenomena). However, before we experience 'things' we can know how they will be perceived by the mind — we know a priori.

Thus, for Kant, the mind contains conditions that contribute to our understanding of the world. As well as the law of causality these conditions include the modes of perception, space and time. Space and time, he says, are not concepts, but form of intuition. Everything we see, hear, touch, smell, and so on, happening in the phenomenal world occurs in space and time. However, we do not know that space and time is part of the phenomenal world; all we know is that they are part of the way in which we perceive the world. Time and space, he says, are irremovable spectacles through which we view the world. They are a priori forms of intuition that shape our sensory experience on the way to being processed into thought. Space and time are innate modes of perception that predetermine the way we think. It cannot be said that space and time exist in things themselves, things 'out there' in the world, rather they inherent intuitions through which we perceive and conceive our world. Time and space, says Kant, belong to the human condition. They are first and foremost modes of perception, not attributes of the physical world. Kant called this approach the Copernican Revolution in the problem of human knowledge. That is, it was just as radically different from earlier thinking as Copernicus' claim that the earth revolved around the sun.

Kant, then, concluded that the human mind is not, at birth, a blank slate to be filled with sense impressions. On the contrary, in order to make sense of sensory experience the human mind must already possess certain basic organising categories of reference into which these sense impressions are fitted. These organising categories are often called 'innate ideas' or 'concepts' because they precede any individual human experience of life. We are born with them. They are an integral part of what it is to be a human being. These categories constitute the core of our faculty of reason. Time and space are subjective. They are our way of perceiving the world. However, they are not the only subjective elements which assist us to understand our experience. As well as space and time, our 'intuitions' there are the a priori concepts which include such things as quality, quantity, relation and causality. It is only in virtue of these innate conditions that we perceive and understand our world. However, the world we perceive is only the phenomenal world. We never perceive actual noumena: the true reality which supports and gives rise to these phenomena. Nor can we ever know transcendent things. Knowledge of certain things such as the true nature of souls, the existence of God are all beyond our limitations. Whatever is beyond our power of knowing Kant called transcendent. That is, it is beyond the realm of human experience. Transcendental knowledge, however, is knowledge which is concerned not so much with objects, as with our a priori concepts of objects (see CPR., p.10). Transcendental knowledge (or 'pure reason') then, goes beyond sense experience and deals with what is known before sense perception: it is a priori.

Drawing from both Empiricism and Rationalism, Kant formed a synthesis between two schools of thought and created his own model. He argued that both sense and reason are integral to our understanding of the world. He accepted Hume's theory that all our knowledge comes from sensory experience, but he also agreed with the Rationalists that our reason contains certain decisive factors that determine how we see and understand our world. Everything we experience will first and foremost be perceived as phenomena in space and time, and for everything that happens we will want to know the reason for its occurrence: its causality. For Kant these conditions are inherent in our minds: they are a priori, and they are what it is to be a human being.

Kant's Critique of Pure Reason

By 'transcendental' Kant means a priori or necessary experience. That is, experience that does not depend on outside influences — empirical experience.

Kant's Critique, it can be said, is divided into two parts: the Analytic and the Dialectic. The Analytic involves the Aesthetic. By 'aesthetics' he means aesthetic, not as we understand in the sense of art, but in the sense that it was understood by the ancient Greeks: as sensation, as in 'anaesthetic' — without sensation. The Analytic is largely positive; in it are determined the a priori principles of understanding: we are also shown the proper use of metaphysics in providing the basis for our objective knowledge. The Dialectic is largely negative. In it we are shown the misuse of metaphysics in using concepts to go beyond that which we can possibly experience, to a world of illusion and contradiction. We are also shown why we are prone to be tempted to this kind of speculation. The Analytic and Aesthetic give us a metaphysics of experience; they display what must be the basic features of experience and reasoning. The Dialectic shows how we err when we attempt to extend our knowledge beyond that which it is impossible to experience.

The Transcendental Analytic

In the section of his Critique entitled 'The Transcendental Analytic', Kant analyses how the faculty of understanding makes information presented by the mind into so — called objects of thought. 'Without sensibility', he says, 'no object would be given to us, without understanding no object would be thought'. The understanding can intuit nothing, the senses can think nothing.

Kant borrows the notion of category from Aristotle. However, where Aristotle spoke of the categories as (i) substance (as the physical or materiality of a thing), (ii) quantity (two metres long etc.,), (iii) quality (colour), (iv) relation (how one thing can be measured against another thing), (v), place (as in location: in the house etc.,) (vi), time (yesterday), (vii), position (sitting, standing(, viii), possession (owning or belonging), (ix), action (walking etc.,), (x), passivity (has walked, was sitting, is cut and so on), Kant's categories consist of four types: quantity, quality, relation and modality. However, within each category he listed a number of what might be called 'sub-categories. Quantity contains Unity, Plurality Totality; Quality contains Reality, Negation and Limitation, Relation has Substance and Accident, Cause and Effect and Reciprocity, and Modality has Possibility and Impossibility, Existence and Non-existence, and Necessity and Contingency.

Kant's concern is with sensibility or sensible intuition. Sensibility is a passive power for receiving information. He wants to bracket off or identify that which is specific to intuition, and to discover the relationship between understanding and that which is specific to intuition.

Kant decided that there are two forms of intuition: space and time. This approach revolutionised metaphysics, where, traditionally, space and time were held to be 'out there' — as conditions of the existence of things. Kant changes this idea and proposes that space and time cannot exist because there is no empirical experience that would allow us to form concepts of space and time, because a void and infinity cannot be thought — it is argued that only someone who had never seen a mountain (as Kant hadn't), could make such a claim. What he means is that because we cannot actually conceive or have notions of the void or infinity, our understanding of space and time is not derived from empirical experience, but are a priori sensible intuitions: intuitions that exist in the mind which are not dependent on outside influences.

Space and time, then, are analogous to filters on a camera: the only images formed are those that have passed through or been subject to the filters. They are not empirical: they are not derived from experience, but are the necessary form of all experience. Neither are they concepts, for there can be no object (like a cup, a dog, a mountain even) corresponding to space and time.

As well as the intuitions space and time there is knowledge. Knowledge involves the use of the basic concepts or categories of the faculty of understanding. The knowledge that that which we see as a table involves having and applying the concept of a table by a judgement of the understanding, as well as seeing it in space and time.

Tony Fahey

back

(55) Jacob asked:

How to answer the question, who am I?

---

A question is a request for information and the information that you are being asked for depends on the situation. In many situations people ask who are you because they want to know what your name is. In a bank when you want to open an account they may want your name and a passport or driving license. In a court of law you often can only establish your identity by by producing witnesses to your birth who have known you and your family since before your birth.

Now you might wonder what is the answer to the philosophical question 'Who am I' but there are no real philosophical questions about your identity and no philosophical answers either.

In a more general it only makes sense for you to ask 'Who am I' if you are suffering from amnesia or maybe if you have just discovered that you were adopted and are wondering who your real family is. It makes no sense to ask for information that you already have.

Shaun Williamson

back

(56) Richard asked:

Do you not think that much controversy over morality is abortive being about particular acts rather than the basic philosophy that advises attitudes to those acts.

That is to say there are three essential attitudes to ethical values moral codes.

There is what may be called the Authoritarian, which essentially means 'God' in which obedience to that authority is the definition of virtue, and anything else is vice. The good of people is in obeying that authority.

The second type is the Libertarian or Relativist. For whom virtue is in obeying ones own conscience, and what is right for ones self, with no external standard by which acts can be judged.

The third type is what may be called Altruist. For whom there is an absolute standard of ethical values, although they may not be clearly determined. The good of all society and every individual. That ethical value is prior to any authority, or 'God'.

---

If you study philosophy you will find that there are many other attitudes to morality besides those you have outlined. For example there is the Logical Positivist attitude which is that moral statements are merely an expression of feeling or there is the Existentialist attitude that there can be no rational reason to prefer one set of moral beliefs to any other. so being a good Christian or being a good Nazi is just a matter of personal choice.

Shaun Williamson

back

(57) Sadikie asked:

Explain the meaning and importance of Socrates teaching Know Thyself and how it relates to another one of his important teachings, i.e., the unexamined life is not worth living.

Use Plato's work The Apology to support and illustrate your explanations.

---

Sadikie, whilst this is a question that I believe is both interesting and important in that it encourages the 'questionee' (sic) to carefully examine two of the central issues that arise in Socrates'/ Plato's philosophy: the invocation extended by the Oracle of Delphi to 'Know Thyself', and to Socrates' view that the unexamined life is not worth living, since it seems to me to be this question may be a reformulation of a question that is often put to philosophy undergraduates, I have chosen not to respond to it in the form of an essay, but to break it down into a series of, what might be called, 'snippets'. These 'snippets' appear under the following headings: Socrates; The Theory of Recollection and the World of Ideal Forms; Apology; The 'unexamined life'; What, in Apology, is Socrates' view of life after death?; Phaedo; and the Development of Platonic thought in Apology/ Phaedo. Although it may appear that I am touching on issues of Plato's philosophy not specifically mentioned in your question, I believe a better understanding of the issues you raise may be gained by a perusal of these wider issues, particularly in the development of thought between Apology and Phaedo.

Socrates (469-399BC)

Socrates was the first great Greek philosopher to be actually born in Athens. Socrates shared with the Sophists a concern for practical issues and particularly for education; but he questioned the extravagant claims of some Sophists that they could teach virtue. He himself was concerned with questions of moral education and moral character, and he seems to have held that the pursuit of moral improvement was the most important human task. How he lived hardly anybody knew. He never worked, and he is said to have never been concerned about the future. He ate only when requested by his disciples to share their food; but they must have found him agreeable company, for there is no account of him having gone without food. At home, however, it was a different story. He neglected his wife Xanthippe and children; and his wife considered him a good-for-nothing idler who brought his family more notoriety than food. In spite of all this she loved him.

This portrayal of Socrates has led some people to take the view that he was quite unsophisticated. However, in his paper entitled 'Wittgenstein, Plato, and the Historical Socrates' (The Journal of the Royal Institute of Philosophy, Vol. 82, No. 319 Jan 2007), M.W. Rowe tells us that this may not be the case. It seems that Socrates' father was a well-known stone mason at a time of great extensive public building e in Athens, and that it is probable that Socrates followed him in this profession. It is also thought that Socrates was married twice. First to Myrto, daughter of Aristides the Just, and secondly to Xanthippe. The 'ipp' of which identifies it as an aristocratic name. Moreover, many other names in his family were of aristocratic origin. This suggests that Socrates, like his father, was a member of the bourgeoisie, and his private means were sufficient to attract the attention of aristocratic fathers on the lookout for potential spouses for their daughters.

Socrates was modest about his wisdom. In fact he did not claim to have wisdom, only to seek it lovingly. The Oracle of Delphi had pronounced him the wisest of Greeks; and Socrates had taken this as approval of his agnosticism which was the starting point of his philosophy: 'One thing only I know', he said, 'and that is that I know nothing'. Philosophy begins when one begins to doubt — when one begins to question the accepted wisdom of tradition. Particularly the one's cherished beliefs, one's dogmas and one's axioms. Puzzled by the priestess of Delphi's statement, Socrates felt obliged to seek the meaning of her remark. By questioning others who had a reputation for wisdom, he came to see that he was wiser than they, because unlike them he did not claim to know what he did not know. The life of Socrates is known mostly through the Plato's dialogues. Possibly through Plato's' Meno we come to understand something of Socrates philosophical method, elenchus, and its primary purpose. And through we learn of his moral character and fortitude through Apology and Phaedo.

Theory of Recollection and the World of Ideal Forms: The theory of recollection is first mooted in Meno. According to Socrates, because the soul (mind) is immortal and has been born often and seen all things in the underworld, there is nothing that it has not learned, about virtue and other things. Thus, there is nothing to prevent one from recalling that which one already knows. What is needed is a process, a method, which allows one to reconnect with knowledge that one already possesses. Thus, for Socrates, there is no learning, only recollection (see Meno 81 c,d). This theory is not without its difficulties. One is what I call the 'chicken or egg' dilemma. That is, if, as is argued, for the soul (mind) there is constant movement between life and death, how do we know which came first: the soul/ mind in a pre-corporeal state during which it has access to ideal forms which it brings into the material world at birth, or the soul/ mind in the body that brings certain ideas gleaned from sensory experience with it into the underworld when the body dies. Surely, it can be argued, if it is the case that the soul/ mind experiences perfection a pre-corporeal state it would have no desire or reason to surrender such a state to enter into the imperfect world of the body.

Another difficulty is the concept that there is an altogether too sharp contrast or distinction between the two realms: between the ideas and particular things. For Plato, the only way that the soul/ mind can experience real truth or real knowledge is by detaching itself completely from sensory experience. Whilst he acknowledges that freedom and separation of the soul from the body can only occur in death (see Phaedo, 67, b), Plato himself, whilst still in the alive, claims not only to know that an ideal world exists, but also to know all that it contains. Thus, he presents us with a concept in which there appears to be a hiatus that is impassable. Not only when one turns to true knowledge does one get no assistance from the senses, but the senses are an actual impediment to the attainment of true knowledge. To behold the ideal, one must eliminate, as far as one can, any knowledge gleaned through the senses, and depend only on the pure light of the mind. As he says,

...the body is the source of endless trouble to us by reason of the mere requirement of food, and is also liable to diseases which overtake and impede us in the search for truth, and by filling us so full of loves, and fears, and fancies, and idols, and ever having, as people say, so much as a thought. From whence come wars, and fightings, and factions? Whence but from the body, and the lust of the body

(Plato Phaedo 66).

For Plato, it is whilst the soul is imprisoned in the body it can catch only the faintest glimpse of the perfect world which it desires so much. It is this very marked dualism, between the world of ideas and the world of things, which presents the greatest difficulty with Plato's system. If the senses are such a burden, if the prevent us from attaining our true destiny, why are we cursed with them? Moreover, if the world of Ideas alone is the rue reality, why should anything else exist? Finally, although it may be some grounds for accepting the view that the soul/ mind precedes physical existence, it does not follow that it survives the demise of the body.

Apology

The Apology professes to be the speech made by Socrates in his own defence at his trial — or rather it is an account of Plato's recollection of Socrates' defence given some time after his trial. In a typical Athenian trial of that period the defendant was given a limited time (measured by a water-clock) to answer the charges and, although he had to defend himself, he could, if he so desired, buy a suitable speech from a professional speech writer — a Sophist. Socrates, of course, rejects this approach and declares that he will speak plain and unvarnished truth. It can be argued, of course, that his disavowal of any knowledge of rhetoric (rhetoric is the art of speaking eloquently and persuasively) and that his ambition is to tell nothing but the truth, is itself a form of rhetoric in that it implies that his statements can be trusted implicitly.

Socrates had been accused of being an 'evil — doer and a curious person, searching into things under the earth and in the sky, and of making the worse seem the better cause, and of teaching all this to others'. He was found guilty by a majority and was, in accordance with Athenian law of that time, to propose an alternative penalty to death. The judges had to choose, if they found the accused guilty, between the penalty of demanded by the prosecution and that suggested by the defence. Therefore, it was in Socrates interest to suggest a penalty that would be accepted as a reasonable alternative to death. However, he chose the sum of 30 minas. While this was much more than Socrates could possibly afford (the sum was guaranteed by Plato, Crito, Critoboulus and Apollodorus) it was considered insufficient by the court and he was sentenced to death. From this we can conclude that Socrates actively sought this verdict, since, to suggest an alternative penalty that would be acceptable to the court was tantamount to admitting that he was guilty of the charges against him — this of course he could not do for central to the charges made against him were that he was guilty of not worshipping the gods that the State worshipped, but of introducing new divinities, and of corrupting the minds of the young by instructing them accordingly.

The Apology, then, is, according to Plato, Socrates' answer to these charges. Socrates opens his defence by accusing his prosecutors of eloquence (what he means by this is rhetoric- the art off speaking persuasively), and rebutting the same charge which was made against him. The only eloquence he admits to, he says, is that of the truth. If this approach offends the court, he says, the court must forgive him for, not being familiar with the ways of the court, he is not familiar with its un-forensic way of speaking. Socrates goes on to relate the incidence where the Oracle of Delphi was once asked if there was anyone wiser than Socrates, to which the Oracle answered that there was not. Socrates claims to have been bemused by this statement, since he always claimed that he knew nothing. However, he also accepts that the god cannot lie so he set out to see if he could find someone wiser than himself.

This sequence is central to the Apology because it is from here that Socrates infers his raison d'etre derives. That is, he regards the Oracle's reply as a puzzle that has to be resolved. Therefore he sees it as his life's mission to expose false knowledge. The first person he goes to is a politician, who is thought to be wise by many people, and even wiser by himself. He soon discovers that the man was not wise at all, and as a consequence is hated by the politician for exposing his ignorance. Next he visits to the poets, and asks them to explain passages of their writings. When they were unable to do so, Socrates concludes that it is not in virtue of being wise that they write poetry, but by a sort of genius and inspiration. Then he tries his luck with craftsmen, but he finds them to be equally unwise. They think they are wise, he discovers, because they know their own trade, but in reality that is all they know. Finally he concludes that only God is wise, and that the wisdom of men is worth little or nothing.

Question: do you see any contradiction in Socrates' claim that his mission in life stems from the Oracles's statement?

According to Socrates, his mission arose from the sense of obligation he felt to discover the truth behind the oracle's statement that he was the wisest. In fact, it must have been the case that he was had already embarked on his philosophical mission, why else would the question have been out to the oracle (it was put, by the way, by Socrates' friend, Chaerephon). It should be said that it was traditional for the oracle to respond to questions in an obscure fashion, and it was accepted that her answers always required interpretation. It is worth mentioning that the Socratic method of enquiry, by its nature, had the effect of undermining the basic assumption of ancient democracy — that is, that all men had the knowledge necessary for the conduct of public affairs. Therefore, by exposing the ignorance of those who were most powerful in Athenian society, not only to themselves, but, since these investigations were carried out in public, to all and sundry — particularly the young aristocrats who had nothing else to do but follow Socrates around all day.

The second part of Socrates' Apology concentrates on charges made against him by Meletus, that he was guilty of corrupting the minds of the young, and that he did not acknowledge the gods of the city, and even introduces new divinities. Since Meletus is in court, Socrates can question his charges directly — which is legally entitled to do. With regard to the first charge, Meletus is forced into the absurd position of claiming that every Athenian citizen improves the minds of the young and only Socrates corrupts them. The conclusions to this premise are self-explanatory. That is, the outlandish claim by Meletus shows that he had never thought seriously about the education of the young, that his charge against Socrates is not based on any concern for their welfare, and that even Socrates, regardless of his wisdom, was no match for the collective wisdom of the entire community.

The charge of introducing new divinities must be understood against the background of the official religion of the state. In contrast to monotheistic religions (one god religions), Greek religions were polytheistic (they had many gods) and undogmatic in the sense that they had no bible or set of orthodox beliefs that the faithful were obliged to accept. The only written account of the Greek gods were found in the poetry of Homer and Hesiod, but these stories did not have to be believed by those who performed the prescribed rituals to appease these deities. However, while there was no set of orthodox beliefs, each city had its own pantheon of divinities — its own group of gods and goddesses- that had been gradually accepted over the ages. Athens, for example, was named after the warrior goddess Athena, who was born out of the head of Zeus. Many of the public buildings on the Acropolis were dedicated to her; the temple of Athena Nike was built to celebrate the defeat of the Persians, and her festivals would have been the most important in the Athenian official calendar. All these public rituals had a profound significance, and Greek religion may be regarded as a kind of worship of their native city by its citizens. There was an officially sanctioned set of gods in each city, and their festivals were carefully regulated, since that was part of the political order. There was also a strict ban on blaspheming against the accepted divinities, and the introduction of new gods was strictly forbidden. This was the legal basis for the charge of impiety brought against Socrates who had often spoken in public about his personal daimon, describing it as it was a warning sign against any kind of wrongdoing. When Meletus is forced by Socrates to clarify the charge of introducing new divinities, he goes to the extreme of accusing him of not acknowledging any gods. Socrates is able to point out that he is being confused with Anaxagoras (one of the natural philosophers) whose book denied that the sun and the moon were gods. Furthermore, Meletus contradicts himself because he also accuses Socrates of introducing new gods (like his daimon) which implies that he does believe in some deities.

Question: What lies behind the apparently contradictory charge that Socrates is an atheist and that he is introducing new gods? Answer: This charge refers to the Socratic talk of a personal daimon which did not belong to the official pantheon (hence the charge of the introduction of new gods), and which led him to challenge traditional pieties on the basis of reason (hence the charge of atheism).

The 'unexamined life'

According to Socrates, the unexamined life is not worth living because it does not prepare us for the next life, nor does it allow us to see things as they really are. It is only when the mind is free from bodily passions that we can see and know the truth. Rather than blindly accepting what tradition tells us, we should search for the truth ourselves.

What, in Apology, is Socrates' view of life after death? Towards the end of Apology, Socrates says that there is a good hope that death is one of two things: either the dead are nothing and have no perception of anything or death is a change and relocating of the soul to another place. If it is a complete lack of perception, he says, like a dreamless sleep, then death is a great advantage, for who does not wake from a dreamless sleep feeling refreshed. If it is a change to another place, as tradition has it, he reckons that it would be wonderful to spend his time testing and examining those there in order to see which of them were wise.

Does this suggest that Socrates has an open mind to the question of life after death? Why, you may wonder, did Socrates choose this particular mission when there were more than likely many other types of political activities that he could have become involved with? The answer is that he was warned by his daimon against participating in democratic politics because he would be destroyed, and so be of no benefit to the city (in the light of subsequent events, one is forced to question the wisdom of Socrates' daimon). In short, Socrates refused to be corrupted by politics and pursued his own personal mission of urging his fellow-citizens to care for their own souls — to examine their own lives — rather than being concerned with wealth and power.

Phaedo

The Phaedo is a record of the conversation between Socrates and the friends who have come to visit him in prison on the day of his execution and deals with the reason why Socrates is not afraid of dying.

In Phaedo, Cebes expresses doubt as to the survival of the body after death, and urges Socrates to offer arguments, which he proceeds to do. The first argument is that all things have opposites and that the opposite of anything is generated from the thing itself: life and death are opposites and therefore each must generate the other — to have life, you must first be dead, and vice versa. Hence, it follows that souls of the dead must exist somewhere, and come back to earth in due course. The second argument is that knowledge is recollection, and therefore the soul must have existed prior to its involvement with the body. This argument is supported by the fact that we have ideas, such as equality, which cannot be derived from experience. We have experience of approximate equality, but absolute equality is never found amongst sensible objects. (In the same way as there is no such thing in real life as an absolute straight line or a complete circle, yet we can conceive of both in our mind). He extends the same argument to other ideas. Thus, the existence of essences, and of our capacity to apprehend them, proves the pre-existence of the soul (or mind) has certain knowledge before it is attached to the body.

To behold the Ideal, argues Socrates/ Plato, the individual must disassociate him/ herself with the senses and rely solely on the pure light of the mind. The body, he says, which requires food and warm, and is subject to all sorts of diseases, obstructs us in our search for truth, and, by filling our heads with loves, fears, and other fancies, prevents us from having so much as a thought. Only philosophy, says Plato/ Socrates, can free us from bodily passion. For it is only through reflection that the soul re-connects with the realm of purity, and eternity, and changelessness. To pass into the Realm of Ideas the soul must be purged completely from the taint of the earth — and the only way the soul fully achieves this is when the body dies. It is for this reason that Socrates has no fear of death.

The central theme of Phaedo is that philosophers should not be concerned with the body, but with the soul, which should be freed from the body as much as possible. (It should be noted that by 'soul' Plato, and the Greeks in general, meant the mind). The body, says Plato, as an obstacle to obtaining knowledge, since we may be led astray by what we see hear, touch or taste. Reality, for Plato, is more accessible to the mind (soul) through reason than through sensory experience, since reason is undisturbed by sense perception or by the sensations of pleasure or pain. The philosopher should turn away from the body towards Realm of Ideal Forms such as Justice, Beauty and Truth which can only be grasped by reason.

In short, Socrates argues that if we are ever to attain pure knowledge or wisdom, we must free ourselves from the body and observe things in themselves with the mind (soul) itself. This is what he means by saying that the unexamined life is not worth living — it is not worth living because it does not prepare us for the next life, and it does not allow us to see things as they really are. It is only when the mind is detached from the body that it can have true knowledge. While it is in the body it must purify itself as much as possible.

Convinced profoundly that knowledge alone is salvation, Socrates saw that the first and most important step toward getting rid of the confused mass of opinions going by the name of knowledge, was to make its inadequacy apparent. Socrates saw himself as the divinely appointed gadfly given to the state. The state, or polis, he said, was 'a great and noble steed who was tardy — lazy — in his motions owing to his size, and requires to be stirred to life'.

In spite of his insistence upon his own ignorance, Socrates was convinced that no one can be more convinced that there exists an absolute truth, and that realisation of this truth is possible to man (see Apology 29b), and it was in his awareness of the existence of this truth and his belief in its availability that Socrates saw himself as superior or wiser than other men: 'Whereas I know but a little of the world below', he said, 'I do not suppose I know. But I do know that injustice and disobedience to a superior, whether God or man, is evil and dishonourable, and I will never fear or avoid a possible good rather than a certain evil' (ibid.). It is here, in this notion of a truth which transcends that which we glean from human experience, that we get a hint of a theme that is central to development of Platonic thought found in Phaedo and, in even greater detail, in Republic. That is, the notion of the Realm of Ideal Forms.

In Phaedo, we see this theme continued, and developed, where Socrates discusses the doctrine of recollection. (72e-78b). In this argument, Plato, through the auspices of Socrates, combines the doctrine of recollection with the doctrine of Forms. Starting with Justice, Beauty, Goodness, he maintains that there are eternally existing entities which are distinct from ordinary things in the perceptible world, and which the mind (of the philosopher, at least) can grasp by a kind of pure thought. When one does grasp one of these forms, says Plato, one attained true knowledge of an absolute value. (Note the shift from the Apology, where Socrates moves from the view that knowledge of truth is possible to the view that knowledge of these truths is actual). The theory of Forms leads Plato deep into metaphysics, and the theory of knowledge (epistemology), and compel him to consider how the human mind (psychology) can have a nature which allows it to know the eternal Forms — he thinks of the human mind as being in some way related to the forms — and how such knowledge can be made to guide the community.

In the Recollection Argument in Phaedo, Socrates uses another 'absolute form' in the examples of Equality or the Equal Self when he argues that absolute Equality (or equality with a 'big E') is distinct from any notion of equality that we derive from worldly experience: like the equality of stones, trees, and so on. When we see two objects which appear to be equal, he says, we are reminded of a distinct ideal form which we do not perceive, but that we recollect. This recollection does not come from prior experience, but from knowledge of Forms which predates our birth.

At this stage it is important to recognise another shift in Platonic thought between Apology and Phaedo. In Apology Socrates says that death is either a dreamless sleep or an opportunity to spend eternity fulfilling his philosophic mission in Hades. Implicit in this statement is the view that Socrates does not know what awaits the soul after the death of the body. In Phaedo, however, Plato introduces the argument that the soul is distinct from the body: that it exists separately from the body, and that, after death, it awaits rebirth. In this theory of opposites he claims that all things arise from their opposite. For example, good is the opposite of evil and arises out of evil, and vice versa. Life and death are also opposites — to die you must first be born, and conversely, he argues, to be born, first you must be dead. And it is in the realm before the life of the body that we acquire knowledge of true Forms.

The Development of Platonic thought in Apology/ Phaedo: Convinced that knowledge alone is salvation, Socrates saw that the first and, most important step towards getting rid of the confused mass of opinions going by the name of knowledge, was to make its inadequacy apparent. Socrates saw himself as being the divinely appointed gadfly given to the state. The state, or polis, he said, was 'a great and noble steed which was tardy — lazy -in its motions owing to its size, and required to be stirred to life'.

Socrates' mission, his 'raison d'etre', is to expose false knowledge. Hence, we can conclude that he believes that true knowledge is attainable. So, in spite of his insistence of his own ignorance, Socrates was convinced that there exits an absolute truth, and that this truth is possible to man (see Apology 29b). It was his awareness of the existence of this truth and his belief in its attainability that Socrates acknowledged that he may be wiser than other men: 'Whereas I know but little of the world below', he said, 'I do not suppose I know. But I do know that injustice and disobedience to a superior, whether God. or man, is evil and dishonourable, and I will never fear or avoid a possible good rather than a certain evil' . This statement by Socrates is central to his concept of truth: while he says that injustice or disobedience to one's superior, God or man, is evil, it should be noted that he has already determined that there is no-one wiser than himself, therefore, in the world of men, he has no superior. However, we have also seen that the god that directs him is his own personal daimon which is not of this world; thus, the truth to which he aspires must be of that same world. It is here, then, in this notion of a truth that transcends that which we glean from human experience, that we get a hint of a theme that is central to the development of Platonic thought which is continued in Phaedo, and developed in greater detail, in Republic. That is, the notion of the Realm of Ideal Forms.

In Phaedo, we see this theme continued, and developed, where Socrates discusses the Doctrine of Recollection (72e-78b). In this argument, Plato, through the auspices of Socrates, combines the doctrine of recollection with the doctrine of forms. Starting with Justice, Beauty, and Goodness, Socrates maintains that there are eternally existing entities which are distinct from ordinary things in the perceptible world, and which the mind (of the philosopher at least) can grasp by a kind of pure thought. When one succeeds in grasping one of these forms, says Socrates, one has attained true knowledge of an absolute value. This is a significant shift where Socrates moves from the view advanced in, that knowledge of truth is possible, to the view that it is actual. It also marks a shift from the view that only God is wise, and that the wisdom of men is worth little or nothing.

Another significant development or shift in Platonic thought is found where, in the Doctrine of Recollection, Socrates uses another absolute form in the example of Equality or the Equal Self when he argues that absolute Equality (equality with a Big E) is distinct from any notion of equality that we derive from human experience. When we see two objects that appear to be equal, he says, we are reminded of a distinct ideal form which we do not perceive, but that we recollect. This recollection does not come from prior experience, as Hume would later argue, but from knowledge of true forms which predates our birth. If you recall, in Apology, Socrates says that death is either a dreamless sleep or an opportunity to spend eternity fulfilling his philosophic mission in Hades. Implicit in this statement is the view that Socrates does not know what awaits the soul after the death of the body. In Phaedo, however, Plato asserts positively that the soul is distinct from the body: that it exists separately from the body and that, after death, it resides in the world of forms, where, in accordance with his 'theory of opposites', it awaits rebirth. In the theory of opposites Plato maintains that all things arise from their opposite. For example, good is the opposite of evil and arises out of evil, and vice versa: to become good, first you must be evil, and to become evil you must first have been good. Life and death are also opposites — to die you must first be alive, and to be born you must first have been dead. It is in the realm before life of the body that we acquire knowledge of true forms. Concepts like Justice, Beauty, Goodness, Equality, and Truth, says Plato, are already in the mind when we are born. All it takes to remind us of these absolute forms is to experience their imperfect representations in the physical world.

Tony Fahey

back

(58) Len asked:

My question has to do with language and in that sense it could be a linguistics or a philosophy of language question. Of the two, I'm not really sure into which category it falls.

If you agree or disagree with a statement, it seems to me this is an absolute. However, on many psych tests employers use these days for candidates seeking to fill the open position, they give choices of 'agree,' 'strongly agree,' 'disagree' or 'strongly disagree. For example; if the the statement is 'The sky is blue,' I can either agree or disagree with the statement. How could I further agree or disagree about the state of the color of the sky or any other statement for that matter. If you and I both disagree, how could either of us disagree 'more' than the other? Herein lies my question: How can you assign an adverbial quantifier to something that I believe is an absolute? I'm pretty sure I'm not the only person who thinks this way so could tell me the difference between agree and strongly agree?

---

This is a fascinating question in the philosophy of language. Somewhere (I can't remember where) Michael Dummett raises the possibility of a speech act similar to assertion, where the speaker is less than fully confident about what they are saying. I think the term he used was 'probabilistic assertion', an idea he associated with Michael Polanyi. I remember long ago discussing this with my thesis supervisor John McDowell, who was roundly dismissive of Dummett's proposal.

Consider weather forecasts. People complain when the weather girl says, 'It will be fine tomorrow,' when she knows damn well that there is only a 70-80 per cent probability that it will be fine tomorrow. (I'm talking about BBC weather girls who've studied meteorology and actually know what they're talking about, on other TV stations they just read a script.) In the discussion I made the point that the context (a TV weather report) makes it clear that when the weather girl makes an assertion about tomorrow's weather, she isn't doing what we normally do when we make assertions. It isn't necessary for her to quote the probability figure, or express some degree of doubt about what she is telling us. It's understood.

But that's just the thin end of the wedge. (I think that this was McDowell's objection.) We would have to admit a whole family of speech acts, speculative assertion, tentative assertion, cautious assertion, confident assertion, emphatic assertion. And that just seems wrong. To make an assertion is to aim at truth. There are only two possibilities, you aim at truth or you aim to miss (i.e. you tell your audience a deliberate lie). It's understood that failure is a possibility. But you can't include a rider to that effect without destroying the whole point of this language game. Or, if not, then the rider adds nothing to what you've already said, the force and semantic content of your speech act.

However, my intuitions tell me that there is a point in the way these questionnaires are constructed, and the options they give. To extract this point, we need to do quite a bit of of work in a number of related areas: game theory, probability theory, the analysis of knowledge from testimony, as well as philosophy of language. Just to give a sense of the complexity involved, here's a short parable:

I am having a pleasant stroll in the hills around Athens with my three companions, Parmenides, Zeno and the young Socrates. Somehow, we've managed to get lost. I'm sure we passed that broken tree half an hour ago. We reach a point where the path forks three ways. 'Which we should we go?' I ask. Zeno scratches his chin. After what seems like an eternity he says, 'It's not right and it's not straight ahead, so I think it must be left.' 'No, no!' shouts the young Socrates waving his pointing finger enthusiastically, 'We have to go right, I'm sure of it!' Parmenides scowls. He stares straight ahead and nods. 'That is the way,' he says in a quiet tone.

Which way do you go?

I don't think that there's any doubt. I would follow Parmenides, I'd go straight ahead. Zeno isn't completely sure, so we can discount him. Socrates' wild gesticulations aren't convincing. Whereas Parmenides impresses us with his authority. He doesn't need to make a fuss about it. He knows.

There's a discussion of the connection between knowledge and authority in my Answer to Demetreus. If you think about it, there could not be a linguistic device which qualified a statement in a way which reliably gave the hearer information about the speaker's authority to make that statement, the credence one should place on it. And yet, we make these kinds of judgements all the time. The reason why we couldn't have such a device is that people aren't always the best authority on how credible an authority they are.

However, there is no objection in principle to introducing new devices into the language game, provided they have a use. Indeed, arguably, we already have such a device in the various ways and means available for conveying the strength with which you hold a belief or opinion. The finesse here is that the 'measure of strength' isn't like assertion, it doesn't function in the same way as a speech act, nor does it function as a qualifier of the speech act. It's information which you give out, more or less voluntary, of the same order (or at least closer to) the information you give out when your face blushes, or you tremble, or your features contort in anger. It is almost impossible to imagine what human life would be like if these features were absent.

When you tick the boxes (and I fully accept, sometimes it doesn't seem to make a lot of sense when you are asked whether you 'agree' or 'strongly agree' to a particular statement which is just plain true so far as you are concerned) you are giving out information which will be processed to yield a result. A numerical scheme is applied, somewhat like the various proposed preferential voting schemes for proportional representation. In a similar way to preferential voting, knowing this gives you some additional measure of control over how your application will be assessed. And the people who designed the form, know that you know this. In other words, you are being invited to participate in a game.

Here's just one example: Good psych tests (I mean, ones that are actually researched empirically, and constructed so you can't just 'cheat' your way to a better result) give you plenty of opportunity to contradict yourself. If you strongly agree to X and also strongly agree to Y, and the implicit assumptions behind X are inconsistent with the implicit assumptions behind Y, you earn a higher demerit than if one or other or both of your statements was less emphatic.

Your doubts justifiably reflect uncertainty about exactly what game you are being invited to play. Who designed the test and what is its real purpose? You are at a disadvantage because you don't know the rules. You don't know what numerical scheme will be applied. Or maybe — and this is potential source of criticism of this kind of exercise — you don't agree to this game at all. (That's what I feel about the new '0-5 star' system of appraisal introduced by eBay. If you're happy with the transaction, there ought to be only one choice, so far as I can see.)

However, if you are applying for a job, you don't really have the option. Honesty is, or ought to be, the best policy. But if it seems to you as if you are being required to be dishonest, give a false account of yourself, then maybe you should consider how badly you want the job.

Geoffrey Klempner

back

(59) Laurens asked:

I have a question regarding the differences or similarities between Kant's categorical imperative and rule utilitarianism.

I'm in the process of writing an essay about this subject and also about whether there is actually a difference between rule utilitarianism (RU) and the categorical imperative (CI).

One, apparently, big difference is that RU makes rules en acts depending on the consequences of an action or rule. And Kant's CI only cares about whether the action (or maxim? is this the same?) complies with the CI. But isn't the CI just a way of testing the consequences if you were to apply your action (or maxim?) universally? If so it doesn't really make sense to say that Kant an the CI are totally disregarding the consequences, which a lot of people state on the internet. And isn't the starting point of any ethical system such as utilitarianism or Kantianism the consequence? The greater good of everybody?

I got a little bit confused because of all of this and I would be really glad if somebody could help me out!

---

You are correct to say that Rule Utilitarianism decides the rightness or wrongness of an action partly on the consequences and partly on the rules, conventions, established rights involved or having a bearing upon the consequential action[s].

The issue with Kant's Categorical Imperative is nuanced. Kant can give the impression the CI is evaluated upon the universalisable consequences of an action. This is not the case though.

'That an action done from duty derives its moral worth not from the purpose which is to be attained by it but from the maxim by which it is determined; and therefore [the moral worth] does not depend on the realisation of the object of action but merely on the principle of volition by which the action has taken place, without regard to any object of desire.'

Consequences do not enter the equation of deciding whether an action is right or wrong. What determines the value is the obedience to the moral law, or dictates of Reason. Reason determines the rightness or wrongness of an act. In universalising an action, Kant is not, like Utilitarianism, concerned with the consequences. he is concerned with the issue of whether it is rational, whether it is logical, free of inconsistency or contradiction-as befitting a rational creature. Satisfying this internal criteria determines its worth. Kant's CI is therefore not consequentialist but deontological. Hi Geoffrey, here you are (a bit long, sorry!).

Helen Beebee
Director
British Philosophical Association

back

(60) Jenna asked:

In mediation 3 what is the reality of the things that Descartes calls the formal/ inherent and the ideas he calls objective or representative reality? I don't quite understand how these prove the existence of god.

---

OK, this is probably a little crude by Descartes scholars' lights, but it'll do: You can think of 'formal reality' just as reality — except that it comes in degrees. So while, say, you and a snail both exist — that is, are real — you have different degrees of formal reality. Roughly, the bigger and/ or more complex and/ or more powerful etc. something is, the more formal reality is has. So you have a lot more formal reality than a snail. Let's say (Descartes would definitely not put it like this!) that you have 90 units of formal reality, and a snail has 6 units of formal reality.

'Ideas' have 'objective reality'. Objective reality also comes in degrees, and those go by how much formal reality is had by the thing the idea represents. So the idea of you has 90 units of objective reality and the idea of a snail has 6 units of objective reality.

Now, what about ideas of things that don't exist? Say, your idea of a unicorn. Unicorns have no formal reality, because they don't exist. But if they did exist, they'd have, say, 30 units of formal reality (they have more reality — they're bigger and more sophisticated — than a snail, but less reality than you). So the idea of a unicorn has 30 units of objective reality.

Now, here's the crucial principle that Descartes and other rationalists subscribed to: a cause must have at least as much reality as its effect. An everyday analogy: if you heat the oven to 300 degrees, and then put your dinner in it to heat up, your dinner won't get hotter than 300 degrees, no matter how long you leave it in for. The effect' (your dinner) can't have more reality (in this case, heat) than the cause (the oven).

Descartes thinks this principle also applies to the relationship between formal and objective reality. So an idea with, say, 30 units of objective reality must have been caused by something with at least 30 units of formal reality. In the normal case — say you look at a snail and this causes you to form the idea of a snail — this principle is easily satisfied. The snail has 6 units of formal reality, and your idea of the snail has 6 units of objective reality. So the cause (the snail) has at least as much formal reality (6 units) as the effect (your idea of the snail) has objective reality (again, 6 units).

What about your idea of a unicorn (30 units of objective reality)? That wasn't caused by a unicorn — there are no unicorns. But something with at least 30 units of formal reality must have caused your idea, according to Descartes' principle. In fact, you are the cause of your idea of a unicorn (or let's assume this) — you invented that idea all by yourself. And that accords with the principle: you have 90 units of formal reality, so your unicorn idea (30 units of objective reality) was indeed caused by something (you) with at least as much formal reality (90 units).

Still with me? Now for the God part. We have (Descartes claims) an idea of God. How much objective reality does that idea have? Well, God (according to our idea of him) is infinite: there are no limits to his knowledge, power, etc. So your idea of God must have infinite objective reality, since God, if he exists, has infinite formal reality. (We haven't assumed the existence of God here; remember the unicorn.)

So, if your idea of God has infinite objective reality, it must, by Descartes' principle, have been caused by something with at least as much formal reality. In other words, it must have been caused by something with infinite formal reality. What has that much formal reality? Why, only God, of course. In particular, you only have finite formal reality (90 units, in fact), so you cannot be the cause of your idea of God. Nor can any other finite being. So the only possible cause of your idea of God is God himself — nothing else has enough formal reality to have caused the idea. So God exists. Simple!

Helen Beebee
Director
British Philosophical Association

back

(61) Ogundele asked:

What is philosophy? How does Greek philosophy affect the way we live today?

---

Although these are questions that arise quite frequently, the answers to them can be of major concern to one setting out on the philosophical path. Because I have given my view on the first question, 'What is Philosophy?', some weeks ago on Ask a Philosopher, and because this view has not changed in the meantime, I have decided to deal with the issue, 'How does Greek philosophy affect the way we live today?', first.

How does Greek philosophy affect the way we live?

Sometimes it is possible to look at the natural world and become aware of an unseen energy, a dynamic that animates physical phenomena. Some people see this dynamic as evidence of that each phenomenon is created by divine force or god for a particular end or purpose, and that this purpose belongs to a greater harmonious system. This view is described as a teleological approach. Others, whilst they may agree that in the natural world events may appear to occur in a regular, preordained, sequence, are reluctant to ascribe to these events the intervention of a divine province, whilst others argue that there is no evidence of a teleological dimension to natural events.

The early Greeks looked at how this energy or force manifested itself in various natural phenomena and attributed to them anthropomorphic or human characteristics. Thus Zeus, or Jupiter, was seen as the supreme god whose anger, at what was perceived as wrongful behavior, was expressed by the roar of thunder, whilst Poseidon was seen as the god of earthquakes and sea, and Bacchus as the god of wine and vegetation. In other words, these gods were seen as whimsical or capricious entities that possessed all the frailties of mortal beings. The myths that evolved from the belief in the power of these gods formed the basis of the early Greeks worldview. Thus, we see the early Greeks found answers to what can be called philosophical questions in religious myths which were handed down from generation to generation. Gods were given human attributes, and in order to appease these gods, and to ensure a sense of permanence and stability — that the sun would rise each day, Spring and Summer would return each year, and so on — sacrifices and homage were paid to these divine entities.

Western Philosophy began when three Milesian thinkers, Thales, Anaximander, and Anaximenes (Aristotle's 'natural philosophers') began to question the legitimacy of the worldview that had been handed down to them by tradition, and to seek answers to the questions concerning the nature of things through the use of reason. And it is the insistence on the search for truth through reason set out by these pioneers in philosophy and taken up by subsequent thinkers from Socrates, Plato, Aristotle right through to Descartes, Kant, Foucault, Habermas and beyond that has shaped, and continues to shape, the world in which we live today. In summa, it should be said that even in our more secularized society, the values enshrined in the philosophies of Socrates, Plato and Aristotle continue to hold influence. Indeed, it should not be forgotten that Christianity, through Augustine (Plato) and Aquinas (Aristotle and Plato), drew heavily on the work of these ancient Greek thinkers, and Islam, through Averroes and Avicenna, borrowed much from Aristotle when Syrian versions of the Stagirite's work had been translated into Arabic as early as the 9th century, that is two hundred years before Aristotle's work became available in Europe.

What is Philosophy?

Philosophy, as any student of Philosophy will tell you, means 'love of wisdom'. In its truest sense it is a desire to challenge, to expand and to extend the frontiers of one's own understanding. It is the study of the documented wisdom — the 'big ideas' — of thinkers throughout the history of humankind. However, even in our most respected institutions, Philosophy is often presented as theology, psychology, spirituality or religion. Indeed, many exponents of these respective disciplines seem to have no difficulty in identifying themselves as 'philosophers' when in fact they are 'dogmatists' (sic). What can be said, however, is that Philosophy is all of the above and none. 'All', in the sense that it will certainly engage with the views advanced by the exponents of these disciplines. 'None', in the sense that Philosophy can never be constrained by views that do not allow themselves to be examined, challenged, deconstructed and demystified in the realisation that 'wisdom' or 'truth' is not something that can be caught and grasped as one particular ism.

For those really interested in Philosophy, it is important to draw a distinction between 'a philosophy' and 'Philosophy' itself. There are abroad today many colleges, institutions, societies, schools of philosophy, groups, cults and sects promoting the view that they 'teach' Philosophy, whereas in fact what they are doing is promoting a particular worldview that they claim is superior to other worldviews or 'philosophies'. What has to be said is that when a body claims that its philosophy has the monopoly on other worldviews it cannot be placed under the rubric of Philosophy — it is dogma. It is for this reason that those institutions that promote a particular religious ethos cannot, by their very nature, be said to teach Philosophy in any real sense: they are constrained by their own 'philosophical' prejudices to treat other worldviews impartially — particularly where these other approaches run contrary to their own. Moreover, by indoctrinating their students into a mindset that holds that it is their way or no way, these institutions show that their interest is not primarily in that which is best for the student, but that which is best in ensuring their own perpetuity. This approach (of using others as a means to one's own ends), as Kant reminds us, is repugnant to Philosophy — the search for wisdom.

What this means is that Philosophy cannot condone any body of knowledge that advocates a closed view on wisdom or truth — one cannot take an a la carte approach to Philosophy. As the Dalai Lama, in the prologue to his book The Universe in a Single Atom: The Convergence of Science and Spirituality advises, where scientific discoveries are made that expose weaknesses in long held traditional beliefs, these beliefs should be abandoned, and the new discoveries embraced (would that all spiritual leaders or 'philosophers' were so open minded!). Philosophy, then, must operate on the premise that its conclusions should ever be open to what Karl Popper calls, 'the law of falsification'. That is where its conclusions are found to be questionable, it is imperative that these views are revisited, re-evaluated and, where necessary, either re-formulated or abandoned. Unfortunately, as history shows, many systems of belief either will not entertain such an approach, or, if or when they do, it is often so far in time removed from the initial discovery that much harm has occurred in the interim.

What should be realised is that the wisdom to which Philosophy aspires is not attained by the practice of uttering self-hypnotising mantras or prayers, nor by being initiated into some select group, sect or cult that promises that its 'road less travelled' is the one true road. Philosophy is not love of 'a truth' or 'some particular approach to wisdom', but a love of truth and wisdom. However, this wisdom or truth does not come pre-wrapped and packaged as one ism or another, rather it involves the courage and preparedness to engage with, to challenge and to expand the boundaries of one's own knowledge and experience. — one's own wisdom.

Tony Fahey

back

(62) Jenna asked:

I don't quite understand how these prove the existence of god. In mediation 3 what is the reality of the things that Descartes calls the formal/ inherent and the ideas he calls objective or representative reality?

---

In Meditation II, Descartes has established the indubitable existence of a thinking thing or Mind. It's existence cannot be doubted as thinking about it or even doubting its existence self-evidently proves its existence. To doubt whether I, as a thinking thing exists presupposes thinking, which in turn presupposes the existence of myself as a thinking thing. The existence of the thinking thing cannot be doubted without incurring a contradiction. One would be saying 'I'm thinking but I don't exist' or 'I exist but I don't exist'. Something must be doing the thinking and, must exist to do it. Hence, I exist. Avoiding the contradiction is evidence of clear, distinct logical thinking.

In Meditation III, having already established his necessary existence as a thinking thing, Descartes ponders the existence of other things such as his body, other bodies and objects beyond him-'in the world out there' Nature has taught him that a world of objects exists 'out there' beyond him. Yet Nature is unreliable as a guide, it has deceived him before. Yet there are some images which contain Objective Reality. That is, they contain more reality than other images. This reality received as an effect or representation by Descartes, cannot arise from nothing [for no thing comes from nothing]. So it must arise from a more powerful cause-perhaps real objects existing independently in the world beyond him? [Hence objects exist out there in the world and cause the effects or representations of themselves in Descartes]

Such an Objective Idea is that of Wax. Clear and distinct reasoning demonstrates the wax possesses certain inherent qualities more real than others. It occupies space, it displays motion, it is flexible. There are other qualities which although it displays, are not inherent to it possessing less reality. Namely, its colour, its shape, its taste, texture and smell. These are temporary, accidental or contingent properties. The wax can be thought of without them. The wax cannot be thought about without it occupying space, without its motion, without its flexibility. These are more real than its contingent qualities. Again, clear and distinct reasoning discerns what is essential to a thing — in this case the wax- and what is not. It is, the criteria of truth. In other Aristotelian words- for Descartes is here using Aristotle's concepts of causality and substance-accident- clear and distinct thinking reveals the substance of a thing [its Objective Reality, what it is] and its accidents [less real].

Descartes continues to question and doubt the existence of a world of objects existing independently of him. He has images of them but still cannot prove they actually exist. Yet clear and distinct thinking has proven the existence of himself as a thinking thing and likewise, that there are certain inherent qualities to images such as the wax. They both display an Objective Reality. That is, their real, essential rather than their less-real inessential qualities are more clearly known yielding their true nature. [Similar to John Locke's Primary and Secondary Qualities-Essential and Inessential respectively]. However, unlike the thinking thing, the independent existence of the wax despite its Objective Reality, cannot yet be proven. If however, clear and distinct thinking proves more fruitful than confused thinking, maybe this method, which uncovers the Objective Reality of thoughts and images, is the correct method to use.

Clear and distinct thinking reveals another Objective Idea-that of God. In this idea, God is the creator, He caused the universe to exist. Using the causal argument for the existence of God [that God is the First cause], Descartes reasons that what is in the cause [God] must be in the effects [Descartes, a thinking thing] [Aristotle again]. So God must be a thinking thing but

infinite and all powerful. As a finite thing Descartes cannot fully comprehend what infinite, unlimited power is. So the idea cannot come from him. It must be innate, placed there by God who, like a craftsman, leaves his mark upon his creation, product.

Now if what is in the effect is what is in the cause, the Understanding and Will must have been placed there by God. [Meditation IV]. So God has placed in each thinking mind, the faculty of Understanding and Judging to produce clear and distinct ideas. Correct application of Judgement by the Will on the contents of Understanding produces clear and distinct ideas. These can uncover absolute, indubitable truth. In fact, freely employing this method of Thinking will move the thinker towards the highest knowledge or reality. S/ he will acquire the necessary, Objective Reality of things, God's 'blueprint' or the scaffolding of his creation. Failure to employ this method moves the person towards ignorance and non-reality.

In Meditation V, the idea of God is again analysed clearly and distinctly. Descartes concludes God must exist from out of his own nature [the Ontological Proof for his existence]. Further, as God's nature is Good, he cannot deceive. To deceive is not to be Good. God cannot therefore deceive Descartes. So the images he has before him, images he has doubted the existence and nature of, are not deceptions but images of real objects, really existing in a really existing world. See the clear and distinct thinking?

It is clear and distinct thinking which reveals this truth of God in Meditation V i.e. logical truth. Truth that is self-evident and cannot, without contradiction, be denied. It is what is called-although Descartes never uses the term-a-priori thinking. Thought is concerned to analyse itself. Thus with the thought of a Thinking Thing necessarily belongs the predicate existence. The subject of Wax necessarily has the predicates of space, motion, flexibility. To the subject of God necessarily belongs the predicate of infinity, Goodness, infallibility. For subject read Substance, for Predicate read accident. This type of thinking is characteristic of that philosophy called Rationalism [Leibniz and Spinoza belong here also]. Truth can be discerned by correct use of Judgement to the Understanding yielding clear and distinct ideas. Correct thinking will connect with the essential reality of things as created by God. This stands in opposition to empiricism which, maintains knowledge is derived from sensory experience.

Hope this helps Jenna.

Martin Jenkins

back

(63) Hunter asked:

If the universe is expanding, what is it expanding into?

---

Hunter, according to Stephen Hawking the discovery that the universe is expanding is one of the great intellectual revolutions of the 20th century. (see A Brief History of Time, 1998, p.41) Given that this is the case it seems legitimate to ask, as you do, into what is it expanding into. I must confess, given my interest in the relationship between philosophy and science, this is a question that occupied my mind for a time. My interest concerned the issue of the expansion of time in the immediate aftermath of the singularity which we call the 'big bang'. According to Bill Bryson, most of what we know about the early moments of the universe is thanks to a theory first advanced by Alan Guth, a particle physicist at Stanford University. According to Guth, at one ten millionth of a trillionth of a trillionth of a second, gravity emerged. (see A Short History of Nearly Everything, 2003, p.36) What this means is that in less than the blink of an eye after the 'big bang' the universe was at the very least a hundred billion light years across. For me, the immediate consequence of this revelation is that, if the smallest measurable unit of time is equivalent to the figure stated above, then this figure is representative of the speed of that moment in time that we know as 'the present'.

Moreover, since it is clear that this instance is ungraspable by the mind, it follows that in reality we humans can never have the slightest notion of what it is to live in 'the now'. For us 'the present' is both an illusion and elusive. Rather than experiencing the present the mind is enveloped in a phenomenological membrane, that is always playing catch up with the continuum of phenomena — of things as they appear to the mind — that are being filtered through the intuitions space and time, the characteristic of cause and effect, natural inclinations, inherited dispositions, social conditioning, worldly experience, personal prejudices, memories, expectations, hopes, and ambitions. Given that we can know so little of the true nature of that which is so immediate to us, the question arises as to what can we know about that which is as remote as the outer regions of the universe? The answer to this is simple: that is, we cannot know anything.

Since this is rather a bald statement I believe it should be followed by some explanation. According to Stephen Hawking there are three models of the universe that explain the expansion of the universe. In the first model the universe is expanding sufficiently slowly that the gravitational attraction between the different galaxies causes the expansion to slow down and, in time, stop. The galaxies then begin to move towards one towards one another as the universe contracts. That is, it starts at zero, expands to a maximum, and then contracts to zero again. (op.cit., p.45) Clearly in such a case the universe is not expanding into anything outside itself. In the second model, the universe is expanding so quickly that the gravitational pull, whilst it may slow it down to some degree, it can never stop it. That is, it starts at zero and eventually the galaxies are moving apart at a steady speed. In the third model, the universe is expanding only as fast as it must to avoid collapsing into itself. In this case the expansion begins at zero and continues indefinitely. However, whilst the speed at which the galaxies move apart gets increasingly smaller, it never quite reaches zero. (see ibid.)

The thing that should be noted about the first model is that in it the universe is not infinite in space, but neither does space have any boundary. Gravity is so strong that space is bent round onto itself, rather like the surface of the Earth, where, if one were to keep going in a given direction one would eventually return to the where one had departed. However, as Hawking points out, whilst this first model may make good science fiction, because it can be shown that the universe itself would recollapse to zero size before one would it return to one's point of departure. (see ibid., p.47) Since one would need to travel faster than the speed of light to reach the starting point before the universe came to an end, this model is a non-runner (excuse the pun).

In the first model, which expands and contracts, space is bent in on itself, and therefore finite in extent. In the second model, which expands forever, space is bent the other way, making space infinite. In the third model, space is flat, and therefore also infinite. The question is, of course, which model describes our universe. According to Hawking, present evidence suggests that the universe will more than likely continue to expand forever, but all we can be really certain of is that if it were to collapse, it would not be for at least another ten thousand years, since it has already been expanding for at least that long. (ibid., p.48). Since mankind will have long outlived its usefulness on this Earth before this time, there is little need for this statistic to concern us.

So where does all this leave us in relation to Hunter's question? The answer of course is that for all their ingenuity, the models leave us none the wiser. We simply cannot know into what the universe expanding.

Tony Fahey

back

(64) Dane asked:

Is religious coexistence something that is attainable, and sustainable, by mankind?

---

Well no it isn't. It is possible for reasonable people to coexist with other reasonable people. However many religious people are completely unreasonable. For the religious and political Fascists the only option is agree with us or we will kill you. If we could get rid of all the Fascists then coexistence might be possible. However Fascism and therefore religious Fascism is a recurring part of human nature, so we can never get get rid of it.

Shaun Williamson

back

(65) Kym asked:

Hi, my questions are:

1) why philosophy is not sophistry?

2) why philosophy is not wisdom?

3) why philosophy begins with wonder?

Thanks...

---

Having once described myself as an 'internet sophist' (see My Philosophical Life) you could say that I deserve this question. I am proud to belong to the tradition of Sophists, which includes the great figures of Thrasymachus, Protagoras, Prodicus and Gorgias. These were thinkers of stature who ventured out into the market place, as I have done, not to talk to anyone willing to listen like Socrates — a most unsuccessful Sophist if there ever was one — but rather on the understanding that their time was worth something, that they deserved recompense for their work. These contemporaries of Socrates and Plato were highly respected. The term 'sophist' had no negative connotations at that time. The closest translation would be 'professor'.

However, I accept the assumption of Kym's question: that there is an accepted sense of 'sophistry' (indeed, no-one these days would use the term any other way) which implies strong criticism and rebuke. To engage in sophistry is to use bad arguments deliberately to confuse your audience, in order to manipulate their beliefs. I hope that I have never done that, deliberately, or even as a result of carelessness or inattention. I share Socrates' passionate concern for the truth. Nor will I criticize his life style. There is nothing commendable about being wealthy. I make a living at what I do — working as an independent philosopher outside the Academy — but no more than I need for a very modest subsistence.

But pity poor Xanthippe. Pilloried by historians for being a fish wife, she had to live with the consequences of Socrates' decision to give up his well paid profession as a stone mason, choosing poverty and despising all comforts in order to follow his muse.

It was, above all, the founding of Plato's Academy and Aristotle's Lyceum which put the final nail in the coffin of the honorable profession of Sophists. If you didn't belong to a school, then you didn't belong, period. To be a genuine 'philosopher' was to be recognized as such by other 'philosophers'. If you were not a member of the Philosophers' Party then by definition you were no lover of wisdom. That still holds true today, although universities are now under increasing pressure from the marketplace, as the recent scandal over the massive hike in UK university tuition fees has demonstrated. It is high time the university professors recognized that they no longer have the monopoly on excellence.

There are indeed signs that the prediction I made back in 1999 when I wrote my piece for The Glass House Philosopher was not so wide of the mark: 'The university departments have had their day. Time has come for a more democratic arrangement.' If I may venture a plug for my philosophy school, Pathways to Philosophy, you can do a highly acclaimed BA (Hons) degree in Philosophy from the University of London, with a higher standard of tutorial support from Pathways than any of the universities is able to provide (including Oxford and Cambridge with their long-established tutorial systems) for less than £5000 all in, for a complete four-year course, a fraction of what it would cost you if you applied as an internal student to the least 'expensive' university today. — And you don't have to give up your day job!

(I think I have earned the right to blow my trumpet now and then. After all, no-one pays me to do this blog.)

Well, what about wisdom. There are examples of great philosophers you could point to who were not very wise. Possibly the most catastrophic example from the 20th century would be Heidegger, whose flirtation with the Nazi regime (whatever gloss you place on it) cannot be justified or explained by any amount of sophistical reasoning. Bertrand Russell, rightly regarded as one of the most important figures in English-speaking philosophy and one of the founders of the tradition of philosophical analysis, was a serial womanizer, who alongside his brilliant views on logic and epistemology was prepared to entertain ideas on social reform which many today would consider opiniated and uninformed. Finally, there is Gottlob Frege, possibly the most important of all the founders of the analytic movement, about whom Michael Dummett in the Preface to his monumental first book Frege Philosophy of Language (1973) laments,

There is some irony for me in the fact that the man about whose philosophical views I have devoted, over years, a great deal of time to thinking, was, at least at the end of his life, a virulent racist, specifically an anti-semite. This fact is revealed by a fragment of a diary which survives among Frege's Nachlass, but which was not published with the rest by Professor Hans Hermes in Frege's nachgelassene Schriften. The diary shows Frege to have been a man of extreme right-wing political opinions, bitterly opposed to the parliamentary system, democrats, liberals, Catholics, the French and, above all, Jews, who he thought ought to be deprived of political rights and, preferably, expelled from Germany. When I first read that diary, many years ago, I was deeply shocked, because I had revered Frege as an absolutely rational man...

Which just goes to show that the capacity for logical reasoning doesn't always go together with wisdom. That's not to say that we should take a sanguine view of philosophers who do not aspire to wisdom. There is a point in speaking of the 'love of wisdom', it's not just hot air or a mere political slogan. I view my own lapses from wisdom with regret, but it doesn't seem to me that my failings in that respect make me any less of a philosopher. One could have also pointed out that there are many persons whom one would consider wise, who have never ventured into philosophy. My old grandmother Rose was wise, though to my knowledge she had never read a word of philosophy. To put the point in terms of the language of logical analysis, being a philosopher is neither a sufficient condition for being wise, nor is being a philosopher a necessary condition for wisdom.

Finally, wonder. The motto on the web site for the International Society for Philosophers is 'Philosophy begins with wonder'. When I came to write this answer, I couldn't remember whether it was Plato or Aristotle who said this. Then I found this answer from Hawkinsian on Yahoo Answers:

Plato puts the following words in the mouth of Socrates at Theaetetus 155 d (tr. Benjamin Jowett): 'I see, my dear Theaetetus, that Theodorus had a true insight into your nature when he said that you were a philosopher, for wonder is the feeling of a philosopher, and philosophy begins in wonder.'

Aristotle echoes the Theaetetus passage at 982b12 of his Metaphysics: 'It was their wonder, astonishment, that first led men to philosophize and still leads them.'

What many miss, however, is that Plato and Aristotle are both talking about the search for theoria, for a knowledge and understanding of the nature of the cosmos and our place in it, in a sense which today would include the great figures of science as well as those of philosophy. (I guess that Hawkinsian is a fan of Stephen Hawking.)

Another motto — which I penned for the PhiloSophos web site — is, 'Philosophy is for everyone and not just philosophers. Philosophers should know lots of things besides philosophy.' Philosophers should know about Hawking — and Dawkins and all the rest. Nor do you need any specialist academic training in philosophy to be a philosopher, to feel that special sense of wonder. That's why I said that philosophy is for all, and I meant it. But not everyone does feel that sense of wonder. My explanation would be, not that the non-philosophical multitude are are not clever enough, or sufficiently well informed. Rather, as the great Presocratic philosopher Heraclitus said, they are like people asleep. They sleepwalk through life, never realizing the truth about the Logos, the ultimate principle of all existence. The question never occurs to them! But it can occur to anybody.

I date the beginning of my interest in philosophy with that question. I have grappled with the question all my career, although, if the truth be told, there has been a long gap where my attempts to make progress with it had to go on the back burner (roughly, from the date when my book Naive Metaphysics appeared). Now I'm back on the case. If I may end this post with another plug, I've started another blog, Hedgehog Philosopher where my daily attempts to put the the jig-saw pieces together is recorded. I may never succeed. In fact, given the ambitiousness of the project, it is almost guaranteed in advance that I will not succeed. But while I remain engaged, I am filled with wonder, I am doing the thing that I do best, I can without blushing call myself a 'philosopher'.

Geoffrey Klempner

back

(66) Gonzalo asked:

I have recently become interested in reading the works of L. Wittgenstein and I would like to know if you could be so kind as to recommend to me a book of introductory nature that would perhaps allow me to understand what he is trying to get across in the Tractatus and Philosophical Investigations.

---

No unfortunately, I can't. There are a few good books about Wittgenstein but they are as difficult to understand as Wittgenstein's works. Wittgenstein is a philosopher's philosopher and by that I mean you can only hope to understand Tractatus and Philosophical Investigations if you have an expert knowledge of Western philosophy and philosophical problems.

Some things are really difficult to understand and there are no simple introductions. For example there are no beginners books on to how to design a nuclear power station.

There are many works about Wittgenstein that are just completely wrong. You could try 'Wittgenstein's Place in Twentieth Century Philosophy' by P.M.S.Hacker. However this book is as difficult to understand as Wittgenstein's own works. You need to accept that there are many difficult things in the world that will make your head hurt when you try to understand them. Not everything can be understood from a simple introduction or even explained in a simple introduction. A degree in philosophy would be a good place to start but failing that just read Wittgenstein's books and ask questions about them.

Shaun Williamson

back

(67) Laura asked:

Explain and critically evaluate the ways in which it might be said that Plato has an otherworldly or nonmaterialist view of reality. How is this otherworldly view reflected in the treatments of the soul, of learning and knowledge, and of ethics in the Apology, Meno, and Phaedo.

---

As essay questions go, this is an interesting example.

1. It is actually two questions.

2. While the first question appears to give the student room to formulate a view of her own, the second question in fact assumes an answer to the first. Consequently, the student is warned off imagining that Plato does not have an otherworldly view, on pains of being unable to address fully half of the question. This would seem to prejudice issues controversial in the literature. For example, to M.M.McCabe Plato is 'anti-empirical'. To Iris Murdoch 'there is no Platonic elsewhere'. If you want any view of the variety of Plato exegesis a helpful survey is Melissa Lane's 'Plato's Progeny'.

3. Also, the question as a whole controversially assumes (in contradiction of a very great deal of literature) that 'otherworldly' is a synonym for 'nonmaterialist'. A bright student could be tempted to base their answer entirely around the large difference between 'otherworldly' and 'non-materialist'. But in that case they would be failing to answer both halves of the question, and instead rejecting it. Rejecting the question is often a good move in philosophy, but not in getting good marks. Question setters should be mindful of this, and, so far as possible, not set questions that ought to be rejected.

4. Thus, while awareness of the dangers of the leading question is gestured towards in the use of the expression 'could be said', the question could hardly be said to have successfully avoided them.

5. A slightly more open question might be:

'Consider the treatments of the soul, of learning and knowledge, and of ethics in the Apology, Meno, and Phaedo. It might be said that Plato has an otherworldly view of reality. Does he?'

6. Yet it seems to me that even that question is pretty large for undergraduate essay — a question to attract the ambitious.

David Robjant

back

(68) Keshov asked:

If first event caused the universe to happen (ie the trigger of the big bang) what caused the first event to happen ? How do you defeat the infinite regress related to this question ?

---

The traditional theological view was that God is the first cause (God being self-caused). This avoids the regress if you can make sense of a self-caused being.

There are two ways of avoiding the regress without appeal to the supernatural:

1. No first event. The universe has an infinite past. Kant famously held that endless past time is contradictory. But there was no rigorous account of mathematical infinity in his day. There is no logical or conceptual barrier to the notion of infinite past time. It's a popular idea with physicists. The earlier version was of an endless succession of Big Bangs/ Big Crunches so that one universe succeeds another indefinitely. More popular nowadays is the Multiverse idea of a boundless foam of universes endlessly budding off new ones which go on to do the same, with variations in the constants of nature and in natural laws from one to the next (perhaps even random variation with natural selection of universes best fitted for budding). It's speculation but solves the 'fine tuning problem' (naturally we must find ourselves in one of the universes in which constants/ laws allow emergence of life), and may be testable (among range of universes fit for life, ours is statistically likely to be an average one so that we expect constants to be a random sample within the ranges allowing life, not absolutely spot-on optimal for all of them which would be suspiciously like design).

2. First event uncaused ie no cause for (single) Big Bang. Why accept 'every event has a cause'. It's easy to conceive of uncaused events. Also, science (quantum mechanics in its currently most widely accepted interpretation) tells us that uncaused events (involving fundamental particles) happen all around us all the time. So why couldn't the universe be an uncaused event ? Indeed the Big Bang view suggests just that possibility (well, there are issues about whether some sort of quantum laws must first exist for a Big Bang to occur in this way, but that's another story)

Craig Skinner

back

(69) Hunter asked:

If the universe is expanding, what is it expanding into ?

---

It is not expanding into anything. There is no pre-existing void. Rather, it is space itself which is expanding. All of the space is within the universe (if ours is the only universe) or within our bubble universe (if there is a multiverse). Don't think of the Big Bang as an explosion starting at a central point. In the early universe, expansion occurred at every point, and indeed every point still seems central because the galaxy clusters are all receding from whatever point you choose in the universe. Note that gravity holds things together on scales less than clusters of galaxies so that there has been no expansion in the space between me and the door since I started typing this, no expansion of the solar system or, indeed, of our galaxy since its formation. It's difficult to visualize expansion of the universe, just as it's difficult to visualize a particle going through two holes at the same time, our brains being adapted for thriving and surviving in the biosphere,but the mathematics of relativity and of quantum mechanics handle such matters nicely(and we invented the maths so we're not doing too badly)

Craig Skinner

back

(70) Jenna asked:

In Meditation 3 what is the reality of the things that Descartes calls the formal/ inherent and the ideas he calls objective or representative reality ? I don't quite understand how these prove the existence of God.

---

We tend to forget Descartes wrote nearly 500 years ago because we (or at least I) read him in modern English translations,so that his prose seems modern, whereas we read, say, Hume, as he wrote it (in 18th C English). Descartes was no fan of scholastic philosophy (he aimed to replace it) but inevitably he sometimes uses scholastic terms and notions that were current in his day.

His argument for God's existence in Meditation 3 is replete with such terms. Two types of reality are distinguished regarding ideas. The mere existence of an idea (as with the existence of anything else) is its formal reality. The content of the idea is its objective reality. Here 'objective' refers to the object contained in the idea (eg a tree in the case of an idea of a tree). The word is used rather like our modern use of 'subjective' — it refers to the tree (say) in the mind, not the tree in the garden. Descartes then introduces the notion of degrees of reality. Ideas all have the same degree of formal reality (all existing as states of mind) but they differ in degree of objective reality — lowest in a 'mode' (a modification of a substance eg shape), intermediate in a finite substance, highest in an infinite substance.

So far, only unfamiliar terms explained, nothing asserted, nothing proved.

Then comes the key assertion (the Causal Principle) on which the proof of God's existence rests.: the degree of formal reality of the cause must be at least as great as the objective reality of the effect. This leads to the conclusion that an idea whose content (objective reality) is infinite (such as my idea of God)cant have its cause in a finite being (with less than infinite formal reality) such as me, only in God, so that God exists.

To me, the Causal Principle is simply an assertion. No evidence is given for it. To assume that an idea in a finite mind needs an infinite cause begs the question as to God's existence. Some claim that the Principle is little more than the commonsense 'something cant come from nothing'. I don't feel this is an adequate statement of the notion, and even if it were, it is questionable. I agree with the point made in the 2nd Objection that animals/ plants (greater) derive from inanimate causes (lesser), contrary to the Principle. Descartes replies, unconvincingly to me, that animals lack reason and so have no perfection not found in inanimate matter, or if they do, it comes from some other source. I would generalize this objection to say that simple things plus simple rules can lead to complex tings eg simple initial conditions in the early universe plus simple laws of nature has caused atoms, compounds, life and minds, so that the Causal Principle is false.

So, whether couched in scholastic or modern terms, Descartes argument fails to prove the existence of God.

That's how it seems to me, although I am no Descartes scholar.

Craig Skinner

back

(71) Cari asked:

Philosopher Peter Kreeft presents us with an interesting thought experiment. He discusses what a huge impact the idea of God has had on the world, and then he ask you to imagine what the world would be like if no one had ever conceived of the idea of God. Would humanity as a whole be better or worse off? What do you think?

---

By way of responding to this question let us begin by asking what concept should we take as the definitive understanding of God? Is it that of Vishnu or Hari, the Superior Reality, the all-pervasive Lord who expands into everything? Is it the Greek God Zeus, God of gods, who thought nothing of copulating with mortal women and who, through the midwife Maia, produced Hermes, messenger of the gods, and from whose head emerged Athena? Is it Aristotle's 'primum mobile immotum' ,the 'first unmoved mover': the roi faineant, the nothing king who reigns but does not rule: a god whose only occupation is to contemplate the essence of things? Is it the God of Islam: a god who rewards those warriors who die so that others might know the true way with maidens who save their honour for this worthy cause? Is it the pantheistic God of Spinoza, or the God of Moses and the Old Testament: the all powerful jealous and angry God who demanded complete allegiance to his whims and commandments? Or is it the God of the New Testament, the God of Jesus who, whilst seemingly benign, still insists that the only way to salvation was through him?

Whilst all these divinities are revered in their own way by their own adherents, for Peter Kreeft it is the God of Catholicism — the God of the New Testament — for whom he is an apologist. Now while Kreeft's position may be admirable to those who share or are persuaded by his views, to the philosophy his 20 ways of proving the existence of God are little more than a rehashing of the same arguments extended by Anselm, Aquinas, Descartes and others -all arguments, that is, that have been found to be rationally or philosophically unsustainable. Although Kreeft accepts that science cannot prove the existence of God, he argues that it is possible to just know something without necessarily being able to prove it. This, of course, places him in the company of flat-earthists, and those who insisted that the earth is the centre of the universe.

Let us pause for a moment to consider what recent history tells us of this world of Kreeft's God. By coincidence I have just returned from visiting the graves of those slaughtered on the battle fields of Flanders during the 1st World War, in this God's world. In my lifetime I can recall the horrors that humans perpetrated against other humans during the 2nd World War, in Kreeft's God's world. I have lived at the time of the Vietnam War, of the genocide of the 'Killing Fields' of Cambodia, of Chechnya, Rwanda, Bosnia, Serbia, Kosovo, East Timor, and many many more troubled lands, all which have taken place in Kreeft's God's world. I live in a country that, over many centuries, has been constantly torn apart by religious differences of people who, paradoxically, all believe in the same God; I have grown up in a country whose indigenous people still carry the emotional scars of a famine that reduced its population from 8 million to little more than 3 million in less than 3 years; I have learned of the misery visited on this God's creatures by earthquakes, tsunamis, volcanoes, and other 'natural disasters'; I have visited 'Ground Zero', site of the place where the planes destroyed the iconic Twin Towers in New York; against this act of terror, and I have witnessed counter acts which, rather than punishing the perpetrators, have terrorized and destroyed the lives of innocent men, women, and children — all in Kreeft's God's world. I have watched those close to me struggle and win their battle against cancer and other terminal diseases, and I have seen others close to me fight the same fight and lose — all in Kreeft's God's world. And I have seen how leaders of the Catholic Church have for too long not only ignored the cries for justice of victims of child physical and sexual abuse by its own clergy, but also how they actively moved to protect those guilty of these crimes by moving them to parishes and/ or institutions where they were free to continue their heinous practices — in Kreeft's God's world. And you ask if the world would be better or worse without such a God.

So let us return to the question at hand. It should not come as a surprise to learn that there are systems of beliefs that do not look to other worldly entities for social values and moral guidance. Buddhism, for example, whilst its adherents follow strict ethical guidelines, is known as the religion without a God. Secular humanists, who espouse reason, ethics and human fulfillment, make the case that humans can be ethical and moral without religion — or God; and believe it or not it is actually possible for atheists and agnostics to live perfectly moral lives without seeking direction from religion and its gods.

So what do I think of a world in which values are grounded in common sense rather than superstition? What do I think of a world built on reason rather than the imagined whims of an imagined transcendent deity? What do I think of a world in which humans, rather than attempting to conceive that which is unconceivable, would be better occupied addressing themselves to the study of the human world: the laws, institutions, customs and practices that are germane to this world rather than the next? And what do I think of a world built on pragmatism, tolerance, pluralism, and mutual respect? You know what, not only do I think that it couldn't be worse, I actually think that it might even be better.

Tony Fahey

back

(72) Sydney asked:

I had my first class in critical thinking earlier today and my professor was unable to tell me if instinct was epistemic luck. I was wondering if you might be able to help answer this question?

---

Reading Sydney's question, my first, somewhat unkind thought was, 'No-one likes a smartass.' But then the question immediately came to mind, Why wouldn't someone qualified to teach critical thinking be able to answer Sydney's question? There is an answer: The term 'epistemic luck' is a piece of technical jargon, coined in the debate over epistemological theories following Edmund Gettier's landmark 1963 article, 'Is Knowledge Justified True Belief?' If you aren't trained as an academic philosopher (or studying for a degree in philosophy) it is fairly unlikely that you would have come across that term. Sydney has obviously been doing a lot of extra-curricular reading.

The curious fact is, you don't need to be a trained philosopher in order to teach critical thinking, at least the way this subject is often taught at colleges and universities. I'm not offering comment on whether that is a good or bad thing.

(Ignorance cuts both ways. My lack of knowledge of the current state of debate in critical thinking leaves me totally unable to answer the question what view critical thinking takes about instinctive knowledge generally, knowing but not being able to explain how you know, following hunches etc. I can live with that.)

If you want to get up to speed with the debate over epistemic luck, you could start by Googling "Rocking Horse Winner" or "chicken sexer", plus Epistemology. These are standard examples of cases where we might be inclined to say that someone 'knows' even though they are unable to explain how they know (the little lad who mysteriously predicts the winners of tomorrow's horse races, workers trained to sort newborn chicks into male and female by subtle differences in their look or feel — or is it?).

I am somewhat bemused by these debates, even though I regularly mark essays sent to me by my students taking the University of London BA module in Epistemology. Epistemology is one of those areas of philosophy that has increasingly acquired the aspect of chess opening theory, with every possible avenue of inquiry, every argument and counterargument explored and elaborated on ad nauseam. No better evidence could be put forward that current academic philosophy has drifted into a new age of scholasticism, driven in part by the incessant need to publish or lose tenure.

However, whenever I begin to feel sick, or bored, I remind myself of things that matter to me in relation to the question of knowledge. Then it all gets real again. Knowledge matters, no more so than to the philosopher pursuing knowledge.

I rely a lot on my instincts. I have hunches. I will pursue an investigation, expending many days, weeks or even months on a question because I have a feeling that it might lead somewhere. What wasted effort, if that feeling could not be relied upon, or did not at least promise some probability of success! Then there are issues in philosophy which I take a strong position on, where I am sure that I am right, even though I know that are those who take the completely opposite view who are just as sure that they are right and I am wrong. How is that possible?

Human beings, like other members of the animal kingdom, have instincts which we have acquired through the process of Darwinian natural selection, although because we are language users and reasoners, the instinctive side of human knowledge has been pushed very much to the sidelines. Instincts are much less useful to us than they are, say, to a pair of nesting birds or a pride of lions. I guess my direct answer to Sydney's question would be that if you believe something 'on instinct', say, that beneath the false smile of the person extending their arm and hand in greeting there lurk aggressive intentions, and that instinct is a genuine biological instinct, with an aetiology, an explanation of its 'reliability' then that isn't a case of 'luck' epistemologically speaking. It is not an accident when the person who roused your suspicions turns out to be a thief or confidence trickster.

The problem is, there are many, perhaps many more examples where one 'feels something on instinct' where there is no valid explanation that a more knowledgeable observer could provide. Then is it just mere guesswork? If you turn out to be right, was that just luck? I'm not sure that it is, always. Maybe I've watched too many American TV detective programs, but it seems to me that hunches can be valid, even if there is no explanation of how you could possibly know, or what it was that gave you the hunch. There is an art to judgement, which no amount of methodological analysis will ever unravel. This applies, in different though related ways, to police work, sports like golfing and archery, or the judgement of a scientific researcher or philosopher.

Of course, one has to exercise caution here. It's so easy to persuade oneself that one's hunch is valid (it wouldn't be a hunch if it didn't feel that special way). But how can you possibly know? More to the point, why should anyone else, who doesn't feel the hunch that you feel, believe you? (How many TV detective plots have followed that theme!)

I've alluded to the question of reliability in Epistemology. One of the main contrasts in current debates is between Epistemologists who consider 'acquiring a belief through a reliable means' as sufficient for knowledge, provided that the belief is true, and those who require something stronger, say, the ability to defend your belief with persuasive reasons when challenged. The problem, is, it's too easy to defeat a knowledge claim just by asking an innocent question (see my Answer to Demetreus). My own tentative view would be that we need to shift the focus away from the question of defining knowledge and onto the question why we are interested in identifying the 'one who knows' the answer to a particular question.

To make a factual statement, any statement, implies that one has the authority to speak. At any time, you can be legitimately challenged. But the inability to meet that challenge doesn't necessarily undermine your right to state your view. 'I just know,' can be a sufficient answer — say, when one is a very experienced golf caddy who just 'sees' that the number 5 iron would be too heavy for that shot, even though according to the book that's the correct iron to use. Trust your caddy, he knows.

At Oxford, I was lucky to have a term of supervision by P.F. Strawson for my B.Phil paper on Kant. In our conversations, it very quickly became clear that when Strawson told me, 'no, you are wrong', it was no use arguing. I was wrong. It wasn't arrogance on Strawson's part, just the voice of experience.

Strawson wasn't claiming to be right about everything. Just about some things. I don't do this too often (my students wouldn't let me). But the philosophical point is about authority. Authority is established, granted, defended, or challenged and defeated. Our interest in knowledge, as a concept, hinges on the question of authority: Whose authority do you trust on a particular topic? when do you accept a piece of advice or testimony and when do you reject it? This isn't about a definition of knowledge in terms of necessary and sufficient conditions, but rather about the place of the concept of knowledge in the social matrix. The simplest example: 'How do you know?' 'I saw it with my own eyes.' End of discussion. This is how language (to use Michael Dummett's happy phrase) 'extends the range of human perception'. Your eyes become my eyes, through the authority which being a witness of the event in question grants you.

I'm coming up to my 60th birthday (next Monday, as it happens). Having been in philosophy for the best part of four decades there are one or two things that I know. In saying this, I hope you will believe me but the decision is yours. Judge me on my work. Right now, I am pursuing a line of investigation (in my other blog Hedgehog Philosopher) where feelings and hunches are playing a somewhat larger role than I would like. I know there's 'something there' which I can't articulate. In the past when I've had that feeling, it turned out to have substance, but not always. I've been down many blind alleys, took many wrong turns and there's no saying for sure that I haven't taken a wrong turn this time. But, in the end, it is a matter of judgement and one has to trust one's judgement.

Good luck with your course, Sydney. Don't blindly accept authority, but don't become a boring sceptic either. Strive to find a balance, that way you will grow.

Geoffrey Klempner

back

(73) Keshov asked:

If first event caused the universe to happen (i.e. the trigger of the big bang) what caused the first event to happen? how to you defeat the infinite regress related to this question?

---

That's easy! You deny one of the premises to the argument.

(a) Deny the premise that the Universe had a start. Perhaps the Universe is infinitely old and the Big Bang might be simply a highly improbable quantum fluctuation that was inevitable given the infinite extent of time which was available.

(b) Deny the premise that the Universe was caused. Perhaps the universe was uncaused (in the same manner as the decay of a radioactive nucleus is uncaused — statistically predictable, but individually uncaused.)

(c) Deny that there was a 'first event'. Perhaps our Universe was caused by events in a prior universe that spawned our own. Some physicists speculate that black holes might be the spawning of new universes. And that chain of parent universes might be infinite, so that there was never a 'first' event.

(d) Deny that there was any spacetime before the Big Bang. If there was no spacetime, then there was no place and no time for there to have been a first event. If the Big Bang created spacetime, then there would be no sense of a 'cause' for the Big Bang. One could say neither that the Universe was caused, or that it was uncaused. Both concepts require spacetime.

There are likely other alternatives that I have missed. These are merely the options I have seen most frequently explored. Each has had its defenders in both philosophy and in physics.

Stuart Burns

back

(74) Dan asked:

I have a question pertaining to why someone would think 'free will' would be incompatible with determinism. (according to W.T.Stace) After reading and thinking about the only answer I can think of is that those that believe they are incompatible have the wrong definition of 'free Will'. Any thoughts?

---

My suggestion is not so much that they have a 'wrong' definition of free will, as that they have a 'wrong' conception of what it is to be a person with free will. Most incompatibilists maintain a notion of 'free will' that demands that a choice, decision or 'willing' is free if and only if it is not caused by the totality of preceding circumstances. But in order to do so, in my opinion, they have glossed over the problem of what constitutes a person under such conditions. As a person, am I not the totality of my character, experiences, beliefs, hopes, and dreams (etc.)?? And are not those things the product of the totality of my past experiences and my genetic constitution? Hence am I not the product of the totality of preceding circumstances?? Whatever conception of 'free will' is brought to this understanding of what it means to be a person, then that conception must be compatible with determinism.

The only alternative that I can envision, involves the denial that as a person I am the product of the totality of my past experiences and my genetic constitution. Which to me, doesn't make any sense. Some free will libertarians draw upon some sort of randomness to escape the deterministic horn of their dilemma. But it seems to me that inserting some sort of 'randomness' into 'self-forming actions' does not make them my actions. So how can it be that random events form part of me in the sense required by their notions of uncaused 'free will'.

It does not matter, of course, on which side of the argument you position yourself — it is self-evidently obvious that those on the other side have the 'wrong' definition of both 'free will' and 'personhood'. And therein lies the basis of the argument. Each side is not so much as arguing their position against the opposition as arguing past the opposition by using terminology and concepts that the opposition would not accept. The only way to resolve this sort of conflict, is to retreat to more fundamental concepts and establish a common foundation there before attempting to progress to free will. Unfortunately, my own reading of the proponents of each side suggests to me that so far neither side seems interested in attempting this sort of approach.

Stuart Burns

back

(75) George asked:

In Ayn Rand's egocentric philosophy we have every right to pursue our happiness but do we have an obligation to concede something for someone else's happiness if it does no harm to you?

---

The brief answer is No!

The highest good within Rands system of ethics, is living the life of Man qua Man. Rand is less than clear as to what this means. But she is explicit that it involves the use of reason in the pursuit of ones own best interests.

Rand does not mean this in the narrow minded, short-sighted, ego-centric sense usually associated with the pejorative adjective of egotist ie. a selfish self-centered person with an exaggerated sense of his or her own importance. Rand views a persons rational self-interest in the same vein as Aristotles eudaimonia an inclusive, and enlightened, and fore-sightful appreciation of ones longer term and wider scope best interests.

Randian ethics is therefore the diametric opposite of Kantian ethics. Kant argues that one should never treat other people as means to your own ends. Rand argues instead that one always must treat other people as a means to your end. But Rand also argues that voluntary cooperation with other people in the achievement of mutually desirable ends is the best means (by far) of achieving your own best interests. Hence a fundamental principle of Randian ethics is that initiating the use (or threat) of force (or fraud) is never the best means of efficiently pursuing a life of Man qua Man. In other words, dealing fairly with other people is the best means of achieving your own interests, and a respect for the interests of others is a necessary requirement for dealing fairly with other people.

One has only the obligation to oneself to pursue ones rational self-interests to the best of ones capabilities. But this obligation may necessitate paying proper attention to the welfare of ones family and friends, ones social, cultural, and economic environment. Hence, the welfare of ones family and friends, etc., are key considerations for a proper view of ones rational self-interests. And this may entail secondary obligations as a means to the ultimate end of ones own rational self-interests.

Stuart Burns

back

(76) Brittney asked:

Why is it reasonable to think that the future will resemble the past?

---

Can you think of a better idea. When I go into a dark room I reach for the light switch. Of course the light switch won't always work but going into a room, clapping my hands and shouting 'Abracadabra' doesn't seem like a better idea.

We assume that in some ways the future will be like the past. If it isn't then we wouldn't know what to think or do.

Shaun Williamson

back

(77) Kunz asked:

Can philosophy liberate Africa from underdevelopment?

---

No of course it can't. Only the African people themselves, by their actions, can liberate Africa from underdevelopment. Philosophy can make people think about the world and about truth so it might inspire people but liberation is about political action, not just about thought.

Shaun Williamson

back

(78) Michelle asked:

This is not a school question I would really like an answer me and a few of my friends are studying this and that's why im asking... looking at the philosophical disciplines of Epistemology and Metaphysics. Aristotle is our strongest example of a pure metaphysical philosopher, although there are many others in the history of philosophy. Philosophers such as Descartes and Hume were radically skeptical of any attempt to discuss the reality of things in itself. Where do you come down on this issue? Do you think that the nature of reality can be known in some fashion, or do you think that human knowledge is fundamentally incapable of knowing in this way? What are your reasons for holding this position?

---

Well it is possible to reject the whole question. Wittgenstein wrote 'Philosophy is a battle against the bewitchment of our intelligence by means of our own language'. For him Western philosophy is a 2500 year old mistake. Getting to see clearly that it is a mistake is the only thing worth doing.

Wittgenstein rejects realism, scepticism, metaphysics, idealism empiricism, logical positivism and any other philosophical ism you care to mention. So it is possible not to come down on either side of the dispute. I discuss things but I never discuss things as they are in themselves and I never fail to discuss things as they are in themselves.

In philosophy you must always question your assumptions and the most persuasive assumption is that the question makes sense.

Shaun Williamson

back

(79) Louella asked:

Kindly explain the saying,

'NOTHING IS WHAT IT SEEMS.'

or does REALITY exist?

thanks...

---

Louella has struck a nerve with her question. On the face of it, it looks like a beginner's question, the sort of thing that someone who hasn't had much exposure to philosophy would think about. 'Nothing is what it seems.' We know that isn't true, don't we? Some things are what they seem (e.g. the half-drunk cup of luke warm coffee on my desk is a half-drunk cup of luke-warm coffee), and some things aren't what they seem. We sometimes get the wrong impression of things. We correct that wrong impression, and then we see things aright.

But, actually — at least in certain moods — I am more inclined to think that all that's just superficial. What we term 'reality' is just a more or less coherent story, not the real truth about things whatever that may be. — I'm just describing a feeling, you don't need to think particularly deeply just to feel this, say, to feel the way Neo felt in The Matrix.

But note what I just said: 'not the real truth about things'. Louella goes on to ask, 'or does reality exist?' Either nothing is what it seems, or reality exists, but not both. That's the implication of her question. But I'm suggesting the opposite: In stating that 'nothing is what it seems', we have in mind, or imply, that there is something real, a real truth about things, which we can never know, or at least which is very difficult to know, or maybe only a few people know.

What if reality didn't exist? How would you describe that situation? Then everything is what it seems. A thing cannot fail to be what it seems unless reality exists, unless there is a way that thing 'really' is, which is different from the way it seems. If reality doesn't exist then everything is the way it seems to me, and everything is also the way it seems to you. If things seem different to you than they do to me, neither of us can be wrong. We are both right. My world-of-seeming is mine, and your world-of-seeming is yours.

But surely that's just... nuts? How could absolutely everything just be exactly as it seems? That would mean that I never a mistake or error about anything, that it is never necessary to correct my first impressions, that, basically, my beliefs are always true (and so are yours). One only needs to consider that a person's beliefs are are not always consistent with one another to realize the impossibility of what I've just stated.

Plato in his dialogue Theaetetus considered these questions. Interestingly, he didn't think that the idea was so 'nuts' that it was OK to ignore it. He puts the thesis, 'Everything is what it seems' in to the mouth of the great sophist Protagoras. (Some commentators would argue that this is a somewhat unfair gloss on Protagoras' famous statement, 'Man is the measure of all things.') Plato doesn't rest content with saying the obvious: that the very attempt to state the thesis leads to absurdity. He considers how one would have to think of knowledge if that hypothesis were accepted.

If there is no real distinction between 'seeming' and 'reality', then we can no longer think of statements as 'aiming at the truth', that is to say, aiming to correspond with the way things 'really are'. Instead, a statement becomes a tool which one uses to affect someone's behaviour. That's what a sophist aims to do in Plato's picture. As a result of listening to the sophist's discourse you are not 'informed' about 'reality' (because there is no reality). Rather, as the sophist would claim, you are made 'better' in some way. The athletics trainer helps you run faster. The rhetoric coach helps you to impress people with your speaking ability, that is to say, your ability to use words to influence or manipulate them.

In a Protagorean universe, according to Plato, everything as we 'know' it is turned upside down. Nothing is 'rational' or 'irrational', 'valid' or 'invalid', 'true' or 'false'. All one is permitted to say is that the verbal statements we make are either 'effective' or 'not effective'. Nor can one even speak of there being a 'truth' as to whether or not a statement is 'really' effective. All speech is propaganda, all thinking is reacting. In some ways, it is a perfect depiction of the world George Orwell horrifyingly portrays in his novel 1984. That's surely not what Protagoras or the other Greek sophists had in mind, but according to Plato it is the inevitable consequence of the relativist view of knowledge.

So what about that feeling I had, that maybe Louella is right and nothing is what it seems? All this, all of you, these... things around me are just shadows, as indeed am I myself. Plato talks eloquently about this too, in his dialogue Republic, in the allegory of the Cave. But, then, according to Plato, something is real, because you can get out of the cave — if you're clever enough, if you know how to work the dialectic. And then you will 'see', not with your eyes (which can never yield true knowledge) but with your mind. The perfect world of Forms.

But if Plato is right then something is what it seems, after all. The eternal Forms are what they seem (to the mind's eye). You cannot gaze upon the highest Form, the Form of the Good, and not know it for what it is, in its very being and essence.

If like me you think that this is all fairy tales — or 'the last fumes of evaporating reality' as Nietzsche describes it in Twilight of the Idols — then maybe you will begin to feel an unnerving sense of the threat that the Protagorean way of seeing things poses. I can't quite wholly believe in this familiar world, I can't fully accept it's 'reality'. But I can't see anything else either, no alternative, certainly no 'purer' or 'higher' world behind these deceptive appearances. Then, maybe, we really can't say for sure whether the Protagorean view, as Plato describes it in Theaetetus might not after all be the only possibility left standing, after all the alternatives have fallen away.

If this is a Protagorean universe, then I am not arguing with you now. I am not making a case. This isn't logic and my words are not governed by any notion of validity. I am behaving — linguistically — a trick invented by a certain species of ape around 50 thousand years ago in order to improve their success rate in hunting non-speaking animals for food. Or whatever is the current explanation. Except of course that what I'm telling you now isn't 'knowledge', or even a 'probable theory'. Just words intended to produce an effect.

One of the more interesting developments in English-speaking analytic philosophy in the last century, was the idea of a clash between 'realist' and 'anti-realist' approaches to the nature of meaning and truth. A foremost figure in the debate is Michael Dummett who argued for an 'anti-realist' theory of meaning, along the lines of the later Wittgenstein's notion of 'language games'. (I first came across Dummett's views in his celebrated book Frege Philosophy of Language London, Duckworth 1973.) To know the meaning of a word or a statement is no more, or less, than to be competent in following the rules for using that word or statement, as accepted in one's local linguistic community.

Subsequently, in an interview (around 1980 in a religious program on TV exploring Dummett's Catholic faith) Dummett confided that his real desire — though he could not yet see a way to do this — was to argue for the necessary existence of God, in a manner similar to the idealist philosopher Berkeley. Rebounding from the Protagorean universe described in the anti-realist theory, there is no alternative to believing in God, if you want to defend your belief in knowledge, truth and rationality. When I saw the program, I was shocked by Dummett's frankness. My D.Phil thesis which I was working on at the time defended a stark version of anti-realism, without the God option.

I don't know exactly where Dummett stands on the God issue today. It is true that he has modified his views on the theory of meaning somewhat. But the stark challenge posed by the philosophy of anti-realism remains: believe in God — or something — or resign yourself to living in the world of 1984. I am aware that there are many analytic philosophers today who broadly follow Dummett's line who would dispute this claim. Wittgenstein believed he was merely combating illusions about our inner life and the 'grammar' of our language. Quietism does not necessarily lead to totalitarianism. However, I don't think that things are that easy or simple. I don't think we really know where we are. My impression is that the way things are going now, it would only take a couple of small steps to find ourselves living in an Orwellian universe.

Meanwhile, academic philosophers debate minutiae, not realizing the ground is being cut from under them.

Geoffrey Klempner

back

(80) Stephen asked:

Is mathematics discovered (synthetic) or invented (analytic)?

---

If you believe in the reality of eternally existing abstract entities such as numbers and sets (Platonism), then we discover these (and their properties/ inter-relations). No problem as to why maths is such a good way of describing and understanding the world. We have a handle on real entities.

If you don't believe in such real entities, then we invent them.We invent systems comprising entities and rules (axioms)and then discover the logical consequences within the system (theorems). Example is Euclidean geometry with its points and lines, and rules (postulates, axioms) from which we can deduce (discover) many interesting and previously unknown theorems. It turns out that actual spacetime is non-Euclidean and better described by Reimanian geometry. That's a different sort of discovery, not a mathematical one but an empirical one.

If we invent maths, how come it's so 'unreasonably effective' in understanding the world ?. Well, our brains are part of the natural world so we would expect evolution to have achieved a mesh between how they work and how the world works. Ah yes, say the Platonists, then how come the best fit between maths and the world is with quantum mechanics, relativity and astronomy, matters far removed from the selection pressures that made our brains what they are. I suppose we can say that these esoteric areas are governed by the same natural laws as everyday phenomena, indeed are simpler (the movements of the planets are predictable for thousands of years, but we cant predict the weather for more than a week or two).

Also, we retain what were once thought to be ridiculous inventions if they turn out to be useful. Irrational numbers were incomprehensible to the Pythagoreans, negative numbers later thought nonsensical, zero took centuries to be accepted in the West (how could nothing be a number ?). Imaginary numbers (square root of minus one etc) the last straw, till we found them essential for the maths of quantum mechanics.We sometimes discard inventions which prove useless. Infinitesimals (much ridiculed by Berkeley) have long been given up.

I am not a Platonist, and so think mathematical entities and systems are invented. We then go on to discovery in two senses. First, discovering the theorems provable within the system. Secondly, whether the real world instantiates the mathematical model (an empirical discovery about the maths, not a mathematical discovery).

Your equating 'discovered' with 'synthetic' is problematic. Your question is clearer without the words in brackets. There is of course much to say on the analytic/ synthetic, a priori/ a posteriori and necessary/ contingent distinctions and their relationship/ overlap, but I don't want to start on that.

Crag Skinner

back

(81) Hugo asked:

Is Schopenhauer a realist or idealist?

---

Given that in his famous essay, The Emptiness of Existence, Schopenhauer refers to 'the flitting present' as the only manner of real existence' one can understand how Schopenhauer might be seen as a realist. Indeed his preparedness to dismiss 'the fleeting present' as 'unimportant 'cannot itself be dismissed so offhandedly to a philosophical position which he insists falls under the rubric of transcendental idealism. Perhaps his unique concept of transcendental idealism is made a little clearer towards the end of the same essay where he states that 'it is only Ideas which exist, and the shadow-like nature of the things corresponding to them' — by which he means noumena. Notwithstanding what might be deemed as the ambiguity of his position vis a vis idealism and realism in this instance, there is no doubt that Schopenhauer saw himself as a transcendental idealist: a position he set out as early as 1813 when, in his doctrinal dissertation, The Fourfold Root of the Principles of Sufficient Reason, he argued that all representations for a knowing consciousness or subject must stand to each other in a lawlike way according to four principles of connection which can be identified a priori.

These are the four forms of the principle of sufficient reason: those of becoming, being, knowing, and acting. [N.B. the principle of sufficient reason states that anything that happens does so for a reason: no state of affairs can obtain, and no statement can be true unless there is sufficient reason why it should be otherwise]. For Schopenhauer, the principle of being had to do with the arrangement of representations in Space and Time, which, following Kant, he saw as a priori intuitions. The principle of becoming had to do with causality, one of Kant's a priori categories. The principle of knowing has to do with reason and truth, because Schopenhauer believed that truth was the reference of as judgment to its ground. Judgment involves relations between concepts (which are in turn abstracted from perceptual representations by reason, not the understanding); but no judgment can express knowledge unless related to a ground or reason a priori. The principle of acting claims that its one object — the subject who wills — is always determined by motives, which are causes seen from within, and therefore also a priori.

Schopenhauer, then, identified himself as a transcendental idealist. He described transcendental idealism as a distinction between phenomena and the thing-in-itself, and the recognition that only phenomena are accessible to the human mind because we only know both ourselves and things in themselves as they are perceived by the mind. Describing his position in his Parerga and Paralipomena (1851), he says, Transcendental is the philosophy that makes us aware of the fact that the first and essential laws of this world that are presented to us are rooted in our brain and are therefore a priori'. Thus, Schopenhauer believed that the world, considered as representations, was transcendentally ideal whilst empirically real. What he meant by this, following Kant, is that while he accepts that there are 'things-in-themselves', we can only know them as phenomena: as things as they are perceived by the mind.

Whist there is still some debate on Schopenhauer's philosophical position as a realist or idealist, let me finish with the opening section of Schopenhauer's The World as Will and Idea (1844), in which he clearly sets out his position in regards to this matter:

THE world is my idea':-- this is a truth which holds good for everything that lives and knows, though man alone can bring it into reflective and abstract consciousness. If he really does this, he has attained to philosophical wisdom. It then becomes clear and certain to him that what he knows is not a sun and an earth, but only an eye that sees a sun, a hand that feels an earth; that the world which surrounds him is there only as idea, i.e., only in relation to something else, the consciousness, which is himself. If any truth can be asserted a priori, it is this: for it is the expression of the most general form of all possible and thinkable experience: a form which is more general than time, or space, or causality, for they all presuppose it; and each of these, which we have seen to be just so many modes of the principle of sufficient reason, is valid only for a particular class of ideas; whereas the antithesis of object and subject is the common form of all these classes, is that form under which alone any idea of whatever kind it may be, abstract or intuitive, pure or empirical, is possible and thinkable. No truth therefore is more certain, more independent of all others, and less in need of proof than this, that all that exists for knowledge, and therefore this whole world, is only object in relation to subject/ perception of a perceiver, in a word, idea. This is obviously true of the past and the future, as well as of the present, of what is farthest off, as of what is near; for it is true of time and space themselves, in which alone these distinctions arise. All that in any way belongs or can belong to the world is inevitably thus conditioned through the subject, and exists only for the subject. The world is idea.

Tony Fahey

back

(82) Ana asked:

The verificationist principle says, in effect, that a statement is meaningful only if you know what would verify it and what would falsify it. If correct, this principle would classify as meaningless many statements in metaphysics. Give some examples of statements that would have trouble passing the verificationist criterion. Explain why they would not pass, and defend them if possible.

---

One example is the verification principle itself, which can neither be verified nor falsified. Two other examples are the existence of minds other than one's own, and the continued existence of empirical objects when no one is perceiving them; these also can neither be verified nor falsified. The principle was first enunciated by the logical positivists, who wanted science to be meaningful but not metaphysics and theology. Verification was later abandoned by Popper on the grounds that a theory could be falsified by empirical evidence (modus tollens) but not verified by empirical evidence, since that would always be the fallacy of affirmation of the consequent. So he argued that statements can be meaningful only if falsifiable in principle. But he was wrong, as my second and third examples show. Philosophers have been trying to verify these throughout the history of philosophy, without success. I'll leave their defense to you.

Helier Robinson

back

(83) Michelle asked:

looking at the philosophical disciplines of Epistemology and Metaphysics. Aristotle is our strongest example of a pure metaphysical philosopher, although there are many others in the history of philosophy. Philosophers such as Descartes and Hume were radically skeptical of any attempt to discuss the reality of things in itself. Where do you come down on this issue? Do you think that the nature of reality can be known in some fashion, or do you think that human knowledge is fundamentally incapable of knowing in this way? What are your reasons for holding this position?

---

First of all Descartes was not a radical sceptic like Hume. Descartes' hyperbolical doubt was a method of searching for indubitable truth, and it led him to his famous cogito ergo sum. I would claim that the nature of reality can be known to some extent, in the form of theoretical science. Theoretical entities cannot be perceived ('theoretical' means 'non-empirical') but evidence for them can be perceived: for example, you cannot perceive electricity but every modern kitchen provides plenty of empirical evidence of its existence. Theoretical science is never certain, but most of it is highly probable. Note that philosophers who talk about thing in themselves do so because they are taking the view that every empirical object is composed of sensations, which are manufactured by the sense organs, and as such these objects are only images of reality, not reality itself. And, also, every empirical object is somewhat illusory and therefore an image of reality, not reality itself. (If you doubt this see if you can point to an empirical object which is wholly free from illusion; if you can, how do you know it to be so?) If these point are correct then we only ever know images, not the real objects that give rise to these images. These real objects are what is meant by 'things in themselves', as well as by 'theoretical objects'.

Helier Robinson

back

(84) Maria asked:

If X might exist but we have no reason to suppose that it actually does exist, then as metaphysicians we should not concern ourselves with X. Is this true?

---

No, it is not true. We might suppose it to exist if only to discuss what the consequences of its existence might be.

Helier Robinson

back

(85) Drew asked:

I have heard it said that 'Science is not the only way of knowing'. Typically the speaker is arguing that rational thought can be a path to knowledge.

If we exclude learning from others, under what circumstances can we reliably know about things without using some variant of scientific method?

---

Saying that science is not the only way of knowing makes it seem as though science has failed to meet all our expectations of it and as though there are alternative ways of knowing things that can be used in place of science. This is just a confusion.

Scientists study the external material world. They try to find the laws that describe the behaviour of the material world. Science is something that humans naturally do. Cavemen wanted to find the material that would make the best point for their spears, that is what led us from the stone age to the bronze age and the iron age.

It led us to discover electricity, television and the computer. It led to medicine, antibiotics and the discovery of DNA.

However there are other things in the world that are not part of science. The study of poetry and music are not scientific studies although science can help even here.

If you want to know about the material world then you study the material world (that is what science is) and there can never be any alternative way of studying the material world, fortune telling and religion won't help and neither will philosophy. The study of the material world (science) is the only thing that can give us knowledge of the material world.

If you want to know about other things then study them. The study of poetry is the only thing that can give you knowledge of poetry. If you want to study mathematics then get a maths textbook, looking at the external world won't help. Counting sheep is a practical application of counting but it won't teach you how to count or what counting is.

Shaun Williamson

back

(86) Dumebi asked:

THOSE WHO SEE WITH THEIR EYES ARE BLIND!

---

Well that is obviously not true as any optician will tell you. I suppose it is an example of hyperbole. There are higher truths that cannot be seen with the eyes. However no one ever imagined that they could. There are also optical illusions.

Suppose someone says I only believe what I can see with my eyes then we could ask how this helps with the study of mathematics. However we should beware of those who seek to persuade us that there are mystical roads to true knowledge, they are often confidence tricksters trying to sell mumbo jumbo or snake oil. If you want to know if its raining then looking out of the window is the best way to tell.

Shaun Williamson

back

(87) Heather asked:

Using Aristotle, Kant, and Mill's philosophies, apply their theories to the following case:

Six months after San Diego native Natalie Dylan graduated from Sacramento State University, she received a letter from the Direct Loan Servicing Center notifying her that the deferment on her student loans had expired and she would soon be required to begin repaying the loans. Never having taken out a student loan, Dylan was stunned to learn that she was nearly 100,000 in debt. Apparently, Ms Dylan's stepfather used her personal information to obtain fraudulent student loans and then absconded with the money. While it remained unclear whether Natalie Dylan will be held responsible for repaying the loans obtained in her name, the government subsidized financing she was counting on to fund her graduate degree was no longer available to her.

To pay for graduate school, Natalie Dylan made the decision to auction off her virginity to the highest bidder through the online auction site eBay. Not amused, eBay removed the offering soon after learning about it. Undeterred, Dylan turned to Dennis Hof, owner of the Moonlite Bunny Ranch, a legal brothel near Carson City, Nevada where her sister already worked. Hof thought the idea was tremendous and agreed to conduct the auction on the brothels website, offering his establishment as the site for the deals consummation. He asserted that it was better for women to use this particular asset to pay for education rather than to give it away in the back seat of a car.

Dylan, who holds a degree in womens studies and plans to begin work soon on a masters degree in marriage and family therapy, doesn't believe what she is doing is degrading. Rather, she described her unusual auction as empowering in that she is using what she has to better herself. Dylan stated she would undergo to a gynecological exam and a polygraph test to verify the authenticity of her offering.

Dylan explained that the highest bidder might not necessarily win the auction, as she wanted to find someone with whom she had some chemistry. Hof said that bidding had topped 3 million and that Natalie was talking by telephone to some of those hoping to win the auction.

Afterword:

It now appears that Ms Dylan will have to relist her virginity for sale. The winning bidder, an Australian businessman, backed out and requested that Dylan refund his 250,000 deposit because his wife did not want him to go through with the purchase. Ms Dylan returned the money and suggested to the man that he return to marriage counseling.

Case 3 Question

Was it morally justifiable for Natalie Dylan to put her virginity up for auction on E bay?

---

No, it wasn't morally justifiable. Ebay's terms and conditions specifically forbid people from offering sexual services for sale. Natalie had already signed up to these conditions so it was dishonest of her to try to circumvent them.

Her suggestion that the businessman return to marriage counselling is ridiculous. This man certainly needs his head examining but I don't think marriage counselling will be enough for him.

Natalie, her father and her sister are shallow, unintelligent people with no moral sense so I don't think Aristotle, Kant or Mill could help them nor would they want such help, its best to leave them alone to mess up their own lives. Also I don't believe that Natalie is a virgin, no matter what your teacher says about her.

Shaun Williamson

back

(88) Kerry asked:

What is a critique on Mill's idea of higher and lower order pleasures?

---

One possible critique associated with Mill's idea of higher and lower pleasures has to do with their method of judgement or evaluation. According to Mill, a specific kind of pleasure was intrinsically better than another if it was preferred over the other by at least a majority of what we might call 'competent rational judges'. By a competent rational judge Mill appears to have meant a person who satisfies three conditions: that of having experienced both sorts of pleasure in question, being talented in introspection and self-knowledge, and being capable of cool, reflective and unprejudiced or disinterested comparison and judgment.

Mill's suggestion here is probably, in part, derived from Book IX of Plato's Republic, after all Mill was familiar with the Greek philosophers from his earliest days (it is reported that he had read at least six of Plato's dialogues by the age of 7, and had read them all by the age of 12).

In this section of Book IX, Socrates tries to prove that the just man is happiest, and the unjust least happy, and one of the ways he tries to do this is by showing that the philosophic life is pleasanter. This attempt takes the form of arguing that each part of the soul has its appropriate pleasure; that each type of person would exalt as pleasantest the pleasures appropriate to his dominant desires. And so we get the lover of the pleasure of philosophy, the lover of the pleasure of honor and the lover of the pleasure of money. The proper tools, Socrates goes on, for deciding which will be pleasantest are experience, understanding and reason. Since these are the privileges of the philosophers, the latter will be the best judges, and their judgment will be authoritative; not only are philosophers governed by understanding, they also have experienced the two other kinds of pleasures from childhood. The other two, meanwhile, might know each other's pleasures, but they never attain, as Socrates says, the 'delight that the contemplation of reality brings'.

In any case, what are we to make of Mill's competent judges? At least one worry that many of us may have about these judges is whether or not they can be sufficiently impartial. To be a competent judge of the relative values of, say, reading Plato, a so-called higher pleasure, and drinking beer or getting a massage, or whatever, -lower pleasures, it is not enough that these judges have simply experienced both -they must have enjoyed them, and, this is the key, enjoyed them properly or in the right way. But, we might think, that the characteristics of these judges needed to evaluate the value of higher pleasures are in some sense fundamentally opposed to those required in the case of the lower pleasures. That is to say, can, in this case, a philosopher who is at base an uncommitted sensualist actually properly evaluate the enjoyment of certain sensualist activities, and, on the other hand, can the sensualist who takes some time to flip through Plato's dialogues actually properly evaluate the pleasures of philosophy? The person, in other words, of an intellectual bent or leaning may find it difficult to properly appreciate the kind of enjoyment to which he or she is not naturally inclined; and the same goes for the person of sensualist leanings.

Kristian Urstad

back

(89) Rebecca asked:

I am writing a dissertation entitled, 'Freewill, autonomy, and mental illness', any suggestions? I have yet to begin it!

---

Before you even start flicking through publication titles, I recommend that you put your dissertation either in the form of a question that you want to answer or a thesis that you want to argue. I lost many hours writing my MA Thesis because I didn't have a specific question or thesis and only wanted write something about 'Self and Others.' Having an angle will make it easier for you to choose between sources and to assess those sources to see if they offer anything that can help with the dissertation. Paul Robert Wolff has produced some excellent guidance about writing a thesis or dissertation which you can view at the following link: http://robertpaulwolff.blogspot.com/2010/07/how-to-write-doctoral-dissertation-in.html.

These things aside, the themes you have chosen appeal to me, because I think there is an interesting question about the extent to which mental illness affects our autonomy (if at all). I would leave aside the concept of free will as I don't think it has much meaning, but I recognize that you would not be the only student of philosophy to think otherwise. I guess I should back up my point of view here. Free will, as a concept, seems to be about choosing what we would like to do, about having a spark of freedom that comes before all thoughts, emotions and reasons that enables us to make a 'free' choice. However, when I make choices I recognize that I do so because I have something in mind, in the most ordinary sense, that I use to make one choice above another. This is not a failing, but a great strength, since that spark of freedom offers us nothing meaningful, whereas the opportunity to choose for our own reasons is fundamentally important and meaningful. If I choose wrongly, for example, I can always look at the reasons which were working at the heart of my choice and reconsider them. If I choose without reasons, my mistakes are nothing but mistakes. For the record, my favorite writers on free will are probably Gilbert Ryle and Schopenhauer.

Regarding mental illness, there are many schools of thought and also many different forms of mental illness. Perhaps it would be best to consider only a particular type of mental illness. I would personally choose to focus on depression since it is something that I have experienced myself and I think that a lot of people experience, but you may want to choose a different mental illness to concentrate on. The important question to ask is whether you think that there are common enough features between different types of mental illness that you can put them all in a single camp and speak about them generally. I suspect this might be hard. You might also want to think deeply about what you think a mental illness is. Is it a diagnosable medical condition or a personal way of being that makes the individual struggle? Again I suspect that it is possible to come up with different answers for different conditions — my inclination is that depression is more about state of mind, whereas schizophrenia is more about neuron activities — but I may well be wrong. There is plenty of literature out there on this stuff, as long as you know what to look for.

When I considered the concept of mental illness for a paper I started by looking at the philosophy of medicine and health more generally and found that even an illness such as cancer is something that practitioners approach with a value system in place. Even normal, ordinary physical medicine isn't merely a technical practice. In terms of bringing the two together I would look at stuff about the phenomenology of mental illness. If autonomy exists, it exists as part of the individual's life-world. I would also want to consider the role that emotions play in decision making and to think about emotions in great depth, because mental illnesses are generally affective in nature, or at least contain affective elements — especially mental illnesses such as depression or bipolar disorder. I hope these brief musing are of some assistance and good luck with the dissertation.

Thomas Rickarby

back

(89) Eli asked:

Are scientific laws really absolute?

---

I'm not sure what you mean by 'really absolute' (yawn, why do philosophical discussions always start with 'it depends what you mean'.Because it is vital to stop us talking at cross purposes).

Maybe you mean one of the following (if not, we'll be at cross purposes)

1. Can we tell the difference between a scientific law and an accidentally true generalization ? 2. Are accepted scientific laws accurate, or might they just be useful approximations ? 3. Do laws change over time ? 4. Could the laws of nature have been different ?

To deal briefly with each:

1. There are many approaches to demarcation, eg

(i) laws are more general (ii) laws don't mention specifics (iii) laws support counterfactuals (iv) laws involve projectible predicates (v) laws are relations between universals

But all approaches are open to dispute and counterexamples.There is no infallible distinction and ultimately the judgment of the scientific community decides. Sometimes new evidence exposes an accidental generalization wrongly held to be a law eg Bode's planetary 'law'

2. Probably the most famous laws are Newton's laws of motion and gravitation, revered for centuries. But now known to be useful approximations to Einstein's more accurate theory of general relativity. Newton is good enough to get men to the moon and back, but a car satnav would be hopeless in town without an Einsteinian correction programmed into its software. We cant be certain our accepted laws are accurate (although we hope they are, and I imagine some truly are)

3. No evidence for this. But some scientists think it possible that, say, the speed of light, or the value of the fine structure constant, has changed slightly over the last 13 billion years, and are looking for past evidence in the form of info reaching our telescopes from galaxies ten billion or more light years away.

4. Fifty years ago most scientists would have said the 'package' of laws seen in our universe is the only logically possible one (God had no choice as to which laws to instantiate), although we don't yet understand enough about the laws to see why that is so. Nowadays, many scientists think the laws and constants of nature are contingent, indeed maybe all possible packages are tried out in myriad different universes within the multiverse. Naturally, we find ourselves in one of the (few ?) universes with laws and constants allowing emergence of intelligent life

I hope this goes some way to answering your question

Craig Skinner

back

(91) Ergyr asked:

I am seeing that people like Stephen Hawking, famous mathematician and scientist, write with massive quantity of words in order to engage in sleight of hand to convince himself or at least his readers that he has proven something, but he has proven nothing.

The same also with Nietzsche and also today's Richard Dawkins.

Is this suspicion warranted?

Now, since Hawking is a mathematician, can he and he should redact all his arguments in his recent book, The Grand Design, into mathematical symbols, namely, in the language of mathematics, and see whether he can fool mathematicians to agree with him that God is not needed for the universe to begin to exist, that it can and does come from nothing.

---

Dawkins gives us atheists a bad name. As far as I am aware, he has no intellectual training in the field in which he is most popularly received (ie, philosophy of religion), but, like the proverbial man in the pub, sets all learning at nought. If you have ever seen him in debate with a trained theologian or philosopher, the impression is priceless — but that is just why he sticks to abusing fundamentalists. Best ignore him if you plan on doing philosophy.

Hawking knows important stuff that many philosophers do treat as relevant to philosophy. See e.g. the work of James Ladyman, who argues that philosophy must be shaped to fit the findings of fundamental physics. But the man to put that argument is Ladyman. Hawking in turning from his own field of understanding is I think out of his depth. How reassuring this is for those of us who haven't done any original work in fundamental physics. For a response to Hawking's latest foray into philosophy that is usefully informed on both physics and philosophy see: http://www.rotman.uwo.ca/who/whatsNew.shtml#philosophyDead

Nietzsche, ho hum, I do know what you mean, and if you want quiet rigor and plain speaking uncontaminated by irony or other rhetorical complications, FN is not your man. Still, not that I agree with FN's strictures against the slave morality (or his misunderstandings of Plato), still you might be being a tad dismissive. There is interesting stuff to be got from him, of which there is ample evidence in the literature.

David Robjant

back

(92) Derry asked:

Why Wittgenstein in Private language says 'Speech with and without thought is to be compared with the playing of a piece of music with and without thought.'

---

The problem with your question is that individual remarks in Wittgenstein's later work cannot be interpreted on their own but only as a part of a group of remarks on a particular topic. Also as Wittgenstein pointed out in the introduction to Philosophical Investigations, it is a work that can only really be understood as a whole.

Wittgenstein's later work is not just something you can pick up, read and expect to understand. In particular Philosophical Investigations is not a work of philosophy, it doesn't contain any philosophical truths or any philosophical arguments. To understand Investigations you will need to have a graduate level of knowledge of the problems of Western philosophy. You should also be acutely aware of the fact that after 2500 years of Western philosophy there is not even a single agreed solution to any one philosophical problem.

If you think you have the answer to any philosophical problem or if you think there are any metaphysical truths then you are unlikely to find anything of value in Wittgenstein and you will probably never be able to really understand what he is saying. Wittgenstein's later work is only for people who know Western philosophy well and still find it deeply puzzling.

There are two key remarks you need to think about. 'Philosophy is a battle against the bewitchment of our intelligence by means of our own language' and 'We are like savages who take the expressions of civilised men and give them a strange interpretation'.

For Wittgenstein the surface features of our language implant myths about the nature of reality in our minds, these myths are what tempt us to create metaphysics.

One of these myths is about the relationship between language and thought. The thought is a ghostly thing that exists in the mind. We express the thought in words. Sometimes we express the thought badly or we don't see the thought clearly enough with the 'inner eye' and then we say 'What I really meant to say was..' This is the myth of the connection between speech and thought, speech is the expression of a ghostly thought.

Wittgenstein wants us to free ourselves from these myths so he compares speaking without thinking to playing a piece of music without concentrating. We are not tempted to think that a piece of music exists as a ghostly think in the mind and that a performance is a reproduction of that ghostly thing on a musical instrument.

Speaking without thinking is sometimes just speaking carelessly. It doesn't mean that the ghostly thought doesn't exist. Speech and thought aren't the same thing but neither are they two different things.

If after all this you are still tempted to ask 'So what is the relationship between speech and thought?' then you have still not understood what Wittgenstein is trying to show you.

Suppose I say..

1. The government's economic policy will increase unemployment

2. I think that the government's economic policy will increase unemployment.

What is the difference between these two things except a difference in the form of words and that 2 can sound more hesitant than 1. If people judge that I say either of them sincerely then they could be said to express my view of the government's policy. Neither of them imply that I have necessarily had any ghostly things existing in my mind or that I have been thinking about the governments policy. However they could also be an expression of the result of many months deliberation on the subject. It all depends upon the context in which my words are uttered. However in philosophy words have no context, they invite us to invent the most likely context.

Shaun Williamson

back

(93) Marion asked:

I've just read an article titled 'Row Over Crematorium Heating For Swimmers' which details plans of a local council, in an attempt to save energy, 'recycling' the heat generated from an adjacent crematorium to heat a swimming pool. Some have slammed the idea calling it, 'Sick and an insult to local residents', and I agree with them... Is this not ethically unacceptable?

---

Here's the link to the BBC TV News story which begins, in typical laconic BBC style, 'The heavy curtains hide a world that few of us want to think about. But the people of Redditch are being forced to do just that...'. The local Council, in a bid to save around £15,000 a year, want to pipe some of the heat from a crematorium to a nearby leisure centre. There have been protests. Supporters point out that the practice is fairly commonplace in Sweden. Another British council is already using the heat from a crematorium to heat administration buildings.

I am not sure that this is a question of ethics rather than taste. Imagine a glass bottomed swimming pool where you could view the human corpses in the process of being burned to supply heat to the pool. That would be rather ghoulish. Although it would certainly make an original tourist attraction, better even than the London Dungeon. However, I can fully understand that there are those who would not want to swim in a pool knowing where the heat came from, even if they couldn't actually see the process going on while they were practising their strokes.

When philosophers think about this kind of issue, there is a regrettable tendency towards excessive rationalism, which includes appeals to utilitarian thinking ('the greatest happiness for the greatest number'). I have no truck with this. My response to Marion's question hinges on questions of ethical rights and wrongs. I am not assuming any particular view of of the nature of ethics. I shall also leave aside the question of good or bad taste, as this is something philosophers have a tendency to get rather pompous about.

Philosophers argue a lot. Sometimes, the arguments get so tangled that ordinary folk are left far behind. However, the true nature of philosophical thinking, its inner essence, isn't found in complex logical argumentation but in the determined attempt to see things differently. This is as much about feeling as it is about logic. To feel appropriately rather than inappropriately is part of what we mean by 'rational behaviour'. Those who put excessive faith in rationalism — the Mr Spocks of this world — lack to some extent the capacity for appropriate feeling.

So how should we feel about the swimming pool news story? Here is one possible take. I'm not saying that this is the only way.

I would like to take a step back and consider the question of how we dispose of the dead, in relation to what it means to live out the span of a human life, not for ourselves, or even those who know us or are affected by our actions, but in terms of ecology. There are many ways in which one can look at human life and its value, but one issue which is becoming increasingly urgent concerns the question of how each of us, during our lifetime, contributes towards consuming the resources of this planet, and the detritus that we leave in our wake.

Imagine on your death bed being shown the vast mountain of refuse that you created during your lifetime. Not something to be proud of. At a time when the very life of the planet is at stake, human achievements seem rather puny in comparison with the consequences of generation upon generation of reckless rubbish generators fouling up the environment, then as a final insult, leaving their own mortal remains behind for others to dispose of.

Burial is an ancient practice. Some would place the beginning of civilization at the point where human beings conducted ceremonies for the burial of the dead. But at a certain point in human history — which may or may not yet have come, but is certainly very close — increasing population and dwindling space must inevitably reach the stage where the dead no longer have the right to take away space from the living. A person who lives out their natural life should depart this earth, not lay a permanent claim to some portion of it.

However, cremation is not the ideal answer either. Let's deal with this question of heat which seems to have so upset the Redditch residents. The amount of energy, in calories or BTUs required to convert a human corpse into ash is far greater than the calorific value of the corpse itself (notwithstanding very rare cases of so-called 'spontaneous combustion' — more accurately called auto-combustion — where the corpse supplies its own energy). All that excess energy goes up the chimney of the crematorium. It's not just a waste, in the economic sense (I fully accept that there is a limit to economic thinking, money is not the only consideration). It's adding insult to injury, after all the resources that we consumed during our lifetime.

I would like to see the dead disposed of in a way which gives something back. The swimming pool crematorium idea is not to everyone's taste. Well, you don't have to swim there, and you don't have to be cremated there. It's your choice. But I believe people should be permitted to choose.

Selling your body for medical use is another possibility which many find distasteful. However, in this case sheer economics make this a serious proposition for some people. A corpse in fair condition, fully exploited and mined for organs and tissue, is worth many thousands of pounds. It could be your most precious gift to your loved ones, as well as helping in a small way to make up for the resources you consumed while you were alive.

Geoffrey Klempner

back

(94) Ergyr asked:

I am seeing that people like Stephen Hawking, famous mathematician and scientist, write with massive quantity of words in order to engage in sleight of hand to convince himself or at least his readers that he has proven something, but he has proven nothing.

The same also with Nietzsche and also today's Richard Dawkins.

Is this suspicion warranted ?

Now, since Hawking is a mathematician, can he and he should redact all his arguments in his recent book, the Grand Design, into mathematical symbols, namely, in the language of mathematics, and see whether he can fool mathematicians to agree with him that God is not needed for the universe to begin to exist, that it can and does come from nothing,

---

Hawking's 'The Grand Design' is no masterpiece. He argues for the truth of M-theory, an extension of string theory. Whether this speculative view, or something like it, becomes an accepted cosmological theory, nobody knows, and I wouldn't bet on it. In addition, he presents his familiar atheistic views. But I don't think he claims to have proven anything. There simply are no proofs that gods (or demons, devils, angels or other supernatural beings) exist or that they don't. There are arguments for and against, but none is conclusive. This doesn't necessarily mean that we should say it's a 50-50 shot. Dawkins argues that the evidence very much favours the 'no gods' view. But of course those who believe in a God or gods don't agree.

Dawkins militant atheism can be a bit wearing. He comes across to me as being angry with God for not existing, and wanting God to come out from under that cloak of nonexistence and take the rap for all the natural disasters, evils and suffering in the world.

As for prospects of a 'no-gods'scientific account of how the universe might have come to exist,there is no knockdown argument, but I feel the eternal inflation/ multiverse view is promising. But I don't think putting this view in full mathematical dress makes it any more or less plausible.

No chance, anyway,of Hawking putting his arguments into mathematical symbols in a popular book (he has already done this in the journals for fellow mathematicians/ physicists). In his first book, 'A Brief History of Time', he says that he was advised that each equation he included would halve the book's sales, and included only 'e=mc squared', hoping not to scare off half his potential readers. A wise move it seems. His book was a blockbusting best seller for over a year. Sixteen years later, his friend and colleague, Roger Penrose, wrote 'the Road to Reality', a better book in my view, but he said some math was essential, readers would rise to the challenge, and,optimistically,included plenty of it. He didn't sell many copies.

As scientists, Dawkins and Hawking are perfectly entitled to point to cosmological and biological evidence suggesting no gods needed, just as those in earlier days (and a few today) argued that the wonders of the natural world showed the glory of God's creation

I'm not Nietzsche-savvy enough to comment on his alleged shortcomings

Craig Skinner

back

(95) Chuck asked:

Why are people homeless?

---

Chuck this is a very complex question so I can't give you any simple answers. Some people are homeless for personal reasons i.e addiction to drink or drugs or mental illness make it impossible for them to exist in a situation where they have to pay rent regularly.

I have also know tramps (hobos) who suffered from a form of claustrophobia which made it impossible for them to live indoors. However most homelessness is caused by the policies of governments and mortgage lenders and a failure to ensure that enough homes are built which can be rented at a reasonable rent. The present recession has been caused by mortgage lenders pushing a enormous amount of money into fraudulent mortgages and these fraudulent mortgages were then turned into fraudulent securities which were traded by financial institutions to the point were they caused a collapse of the banking system. Yet very few people have been prosecuted over this fraudulent activity. Both in Europe and the U.S. many people are becoming homeless because they cannot pay a mortgage which they should never have been given in the first place. At the same time there are growing numbers of vacant properties and a growing number of homeless people.

So some homelessness is caused by the government failing to keep fraud out of the banks and out of the housing market. All of this stems from the deregulation of the banks by President Regan and Margaret Thatcher. Banks which previously had only been allowed to take deposits from individual customers ( a safeguard introduced after the 1927 Wall St. crash which was also caused by the banks reckless lending) were now allowed to gamble on the stock exchange and to invent new types of securities which were not what they pretended to be.

Shaun Williamson

back

(96) Eli asked:

Are scientific laws really absolute?

---

I don't know what you mean by really absolute. Scientific laws represent the best explanation we have for the facts about the material world that we know at present. As we get more information about the material world we may modify our scientific laws but for real scientific laws such as quantum mechanics or the theory of evolution, it is not likely that that they will be overturned but they may be modified or become part of a larger theory in the future.

However not all so called scientific laws have clear and overwhelming evidence in their favour.

The important fundamental theories won't be overturned but scientists and scientific researchers cheat, lie and publish false results. This is especially true in the fields of medicine and medical research. The newspapers also distort and lie about scientific research in order to make a good story.

Shaun Williamson

back

(97) Derry asked:

Why does Nietzsche speak of the 'timeless knowing subject' that is based on the fiction of 'an eye turned in no particular direction'?

---

He was pointing out that an eye is always turned in a particular direction and so there are things that the eye can see and things that the eye cannot see, because for example they are behind its field of view.

In the same way the things that we think we know are always relative to our time and place in the world. However this has been interpreted by some people to mean that truth is always relative to a time and place and the things that we think are true could always be seen as false at some time in the future. Truth and knowledge are just a historical interpretation of the world.

The truths of logic and mathematics will always be true. The major scientific theories may be replaced by more comprehensive theories in the future but that doesn't mean that our present theories are false. What Newton and Rutherford discovered about physics is still true even if we now have a fuller understanding of their ideas through the theories of Relativity and Quantum mechanics.

It is more likely that our ideas about politics and society and morality will change radically over time although even there some things remain constant.

And of course some things seem to remain outside our field of view at all times. Will we ever get politicians who understand economics and who know that money doesn't grow on trees and that all bubbles eventually collapse? I think not.

Shaun Williamson

back

(98) Aviral asked:

When we talk about changing past then to change it where can we find it ? Is it anywhere or any place we can specify ?

The past mostly exist in our memory and nowhere else. How can we change something which is merely our memory or impression in mind. So till we are not able to get inside our own mind where can we find past to change it ? If past could be changed then the whole concept of destiny would have been gone.

Can we deny destiny completely despite of so many things in this world which are not under our control and everyone at some point in time realizes destiny if he is honest to admit it?

---

If you think of Einstein's four-dimensional space-time, then time is a dimension. What we call 'now' is a point on this dimension, moving steadily along it into the future. So all of time exists, timelessly, as it were. We are stuck in the now, so cannot travel to the past, but it exists. (One reason we cannot travel there is the grandfather paradox: suppose you could travel to the past, and went back in time to when your paternal grandfather was a boy, and killed him. Then your father would never have been born, so nor would you have been born, in which case your grandfather would not have been killed as a boy.)

Destiny, on the other hand has to do with the future: it is the supposition that the future is like the past in that it cannot be changed (or some of it at least cannot be changed). This is tied up with the question of free will. Although we have a strong feeling of being free in making choices, it can be argued that this feeling is an illusion, and we are not free. The argument is: every event is either caused, or not; if it is not caused then it is a chance event; making a choice (that is, deciding an action) is an event, which is either caused or chance; if caused it is not a free choice, and if chance it is not a free choice; therefore there is no freedom of the will, in which case everything that happens is destined.

I might add that neither of the conclusions of these two paragraphs is final: these matters have been debated for centuries, inconclusively. So if you can refute them, preferably validly, then you will be doing philosophy on your own. Good luck.

Helier Robinson

back

(99) Albert asked:

Speculations, procedures, and conclusions are not scientific unless they are made public.' Is this true?

---

No. First of all, publicity is very important in science. For example, the three main criteria in empirical science are: experimentalists should be objective, quantitative data are preferable to qualitative data, and experiments must be repeatable. Objectivity is attention to the public (as opposed to subjectivity, which is attention to the private), quantitative data are more public than qualitative data, and repeatability of experiments makes their results more public. Again, reasoning, in the form of logic and mathematics, is the most public part of mental activity. However, speculations, procedures, and conclusions are more or less scientific regardless of when or if they are made public; making them public is not a magical act of reconstitution.

Helier Robinson

back

(100) Kaitlyn asked:

Is it possible for one to start life after they have previously gained age? Can one experience two or more separate lives before they die? What is life?

---

I will answer only your third question. The best definition of life that I know of is that offered by Erwin Schroedinger, the discoverer of Schroedinger's Equation in quantum mechanics. He defined life as a system of very high negative entropy in dynamic equilibrium. Entropy in physics is a measure of disorder, so that negative entropy is a measure of order. Dynamic equilibrium, as opposed to static equilibrium, is equilibrium during change. According to the second law of thermodynamics the entropy of a closed energy system can increase to a maximum (called the (static) equilibrium state), but is exceedingly unlikely to decrease. In a living system the entropy is very low (the negative entropy is very high) and this keeps on trying to increase in accordance with the second law; but the increase is mostly cancelled by bringing in negative entropy from the environment, by feeding on other life or on sunlight. Ill health is a matter of increasing entropy, and death is the final collapse of the dynamic equilibrium.

Helier Robinson

back

(101) Zzy asked:

Hi, I would like to ask a couple of questions.

It would be great if you guys can suggest a few solutions, thanks!!!

To put it simply, my questions are about the rules of the universe (and whether the universe is logical or not).

I think that things in the universe may or may not obey unchanging rules. There should be more than a handful of unchanging rules up to now at least. I believe that many things in our daily life are based on assumptions similar to those rules.

So why are those rules obeyed by the universe? I believe an obvious answer is that they are obeyed for no reason. It is just the way it is. If this is indeed the case, then wouldn't it be reasonable to not expect the rules to stay the same? If those rules just happened to exist, then they could also just happened to change, right? I probably have committed a bunch of fallacies in the above words, but I hope you guys can see what I'm trying to say here.

Does the universe have to be logical/ rational/ sensible/ reasonable? Because when I think, I need to know where my starting points are. If I don't know anything for sure, how am I suppose to think? One problem I used to think about is:

Why do things exist?

If the universe is not logical/ rational/ sensible/ reasonable, then there could be no reason at all for existence. If there is no reason for existence and there is no reason for the rules of the universe, can't we just say that there is no reason why the universe's current conditions will not change? Since the universe is already not logical/ rational/ sensible/ reasonable, then illogical things can happen right?

Please help me out!

---

'Rational' is the best word to use here, since it includes the logical and the mathematical. The most important thing about the rational is that it requires that a contradiction cannot be true, which in turn means that contradictions are impossible in reality. (In fact the only realm in which contradictions can occur is language: we can say them and write them, but cannot make them exist otherwise.) The second important thing about rationality is that it contains necessities: logical necessities. For example, if it is true that you have two apples and three oranges then necessarily you have five pieces of fruit.

Philosophically, reality (the Universe) is rational because it contains no contradictions and it does contain necessities. Real necessities are causal necessities, and they correspond to the logical necessities of rationality. The importance of this is our desire for explanations: an explanation of something being a description of its cause. To describe cause is to explain their effects. We have several kinds of explanation: superstition, common sense, theological, metaphysical, and scientific. The last is the most reliable, partly because of its rationality. In science theoretical science explains what empirical science describes, and it does so by describing the underlying causes of empirical phenomena. 'Underlying' here is a metaphor for imperceptible, and 'theoretical' means imperceptible, or non-empirical.

The question now is: can everything in the Universe be explained? If it can then everything is causally necessitated, or determined. If it cannot then some things are uncaused, or a product of chance. (Note that chance is not necessarily wholly so; if you toss an ordinary die and the result is a chance number, then that number necessarily is one of 1, 2, 3, 4, 5, and 6.)

So your questions all revolve around the question of whether determinism (the doctrine that everything in the Universe is fully caused) is true or not. Philosophers are not agreed on this question. My own view, for what it is worth, is that determinism is true.

Helier Robinson

back

(102) Ross asked:

Hi, I once remember an old philosophy tutor I had using the quote 'there has been no new philosophy, just two thousand years of commentary on plato and aristotle' I seem to remember him saying that the quote was from Bertrand Russell, but I am putting it in an essay and need to make sure that it is referenced correctly and cant find the quote anywhere. does anyone know who said this and/ or which work it is from? Regards.

---

See Alfred North Whitehead 'Process and Reality', p. 39 of the Free Press edition, 1979, ISBN 0-02-934570-7:

'The safest general characterization of the European philosophical tradition is that it consists of a series of footnotes to Plato'

Various web references:

http://www.lehman.edu/deanhum/philosophy/platofootnote/PlatoFootnote.org/Plato.html
http://www.archive.org/details/processrealityes00whit
(USA public libraries only)
http://www.amazon.com/Process-Lectures-Delivered-University-Edinburgh/dp/0029345707
http://www.free-ebook-download.net/science-book/4443-process-reality-corrected-edition-alfred-north-whitehead.html

I hasten to add that I have sadly yet to read a book that still has its fans today, among those who think Whitehead was here anticipating strands of thought in Heidegger.

David Robjant

back

(103) Deborah asked:

What came first the chicken or the egg?

---

The egg came first. Before modern chickens evolved there were many other creatures who laid eggs, for example insects, reptiles and dinosaurs all existed before modern chickens and they all laid eggs. Fish also lay eggs and they also existed before chickens did.

You should study evolution, then you will not be tempted to ask this sort of question.

Shaun Williamson

back

(104) Roy asked:

could the angels and aliens be hermaphrodites

---

Angels are supposed to be pure spirits. If they exist they do not have bodies or sexual organs, so they cannot be hermaphrodites.

I don't know anything about aliens, I have never met one and neither has anyone else.

Shaun Williamson

back

(105) Payal asked:

Numbers are a man made creation, so how we know 1 + 1 = 2?

---

It is precisely because numbers are a man made creation that we can be sure that 1+1=2. In the same way man invented the game of chess and we can be absolutely sure that a game of chess ends if one player achieves checkmate.

However our number system also has useful application to the real world and that is why it is important to us. Man invented the ruler and we can be absolutely sure that 1 metre = 1000 millimetres. But rulers are also useful for measuring things. If measuring things didn't work, i.e. if the things in our world changed their size randomly, we would have no interest in rulers or in measuring things.

Shaun Williamson

back

(106) Ruth asked:

(I apologise for asking such a basic question, but I have googled and googled and.... nothing).

I was reading a discussion online the other day and one of the participants posited that 'all experience essentially takes place in the mind'. My question is, if there is no such thing as 'objective' reality, are people altered by the things they experience and change because of outside stimulus, or do they 'change' the things they experience to suit their own framework? Which choice is preferable?

---

Neither, things are much more complicated than simplistic philosophical talk might lead you to believe. If all experience essentially takes place in the mind and if there is no objective reality, why do I have to pay taxes? Why can't I fly?

Wittgenstein wrote 'Philosophy is a battle against the bewitchment of our intelligence by means of our own language'.

Shaun Williamson

back

(107) Julie asked:

I was reading a fantasy novel recently, and the theology in this book was very interesting. In the novel, the idea is put forth that WE are God. That man, and his sentience, are a vessel with which the universe is to becoming selfaware.

Is this a common belief? Is there a name or an 'ism' for this particular line of thought?

---

Julie, it seems to me that the author of the novel to which you refer has borrowed a small part of his theme from the 'ism' known as pantheism and a larger part from Hegel's 'dialectic process'. Pantheism, from the Greek pan (all) and theos (god), is the belief that God and nature are identical where nature is taken to mean the universe and the totality of all that is; alternatively that God is all and that all is God, that God is identical with the universe. This doctrine holds that all beings are modes, attributes or appearances of a single, unified reality or being. Baruch Spinoza's philosophical views, as set out in his Ethics is considered to reflect this approach. However, since you suggest that the novel in question concentrates rather on 'man, and his sentience' rather than everything in the universe, it seems to me that your writer has taken more from Hegel than Spinoza. According to Hegel everything in the universe can be understood only in terms of the Absolute mind which has been evolving throughout the world`s history into a transcendent, self-contained being. Each stage in the world`s history is the expression of the inner struggle of the Absolute to achieve self-realisation. The term Hegel used to describe this evolutionary process is `dialectic`. All change, says Hegel, is subject to this law. By this he means that first a thesis is produced, it then develops an opposition (an antithesis), from the ensuing conflict between the thesis and antithesis a synthesis arises. This synthesis then becomes the thesis, and the unfolding process continues throughout history until the Absolute achieves self-realisation.

Hegel believed that in discovering the dialectic that he had discovered a necessary historical law. That is, for Hegel, the dialectic is an actual process which worlds events necessarily follow. All change, he insists, occurs by virtue of this law. According to Hegel, history is not a series of discontinuous contiguous events, rather it is structured by the triadic process of thesis, antithesis and synthesis. All forms of life, says Hegel, whether artistic, philosophical or religious, display this organising principle. The dialectic process shows how, in human history, one form turns into another as the contradictions are revealed in previous forms. However, the preceding form is not discarded, rather that which is valuable in each is carried into a richer and deeper form of life which incorporates and transcends the previous forms.

For Hegel history can best be understood by observing the development of nations in the light of the dialectic process. For example, a nation develops; it produces an opposition to itself, and from the resulting conflict between the two opposing forces a new order emerges: a civilisation that is of a higher order than either of its predecessors. For a working model of Hegel's argument we need no further than the northern counties of this island: the Nationalist opposition (antithesis) arises in conflict to Unionism (thesis). Following a long struggle a synthesis (which has its genesis in the Good Friday Agreement) emerges. If Hegel's thesis is correct, this new situation should lead to a more pluralist, equitable, and stable society: a society that will be most value to each of its predecessors. In time this synthesis will become the thesis, out of which will emerge an antithesis, and so the process continues.

According to Hegel the dialectic process must take account of all major forms of human experience and activity and with the basic psychological structures of selves who engage in such activities. That is, the nature of the self and its capacities has to be understood in relation to basic forms of activity in which human beings engage: familial, economic, political, artistic, philosophical, and religious. For Hegel, human beings only develop a rich sense of self in relation to common activities. That is, they achieve recognition from others, and affirmation of their 'selves' by engagement and interaction with the world in which they live. For Hegel there is unity in diversity. That is, beneath the surface differences between various activities in which human beings engage there is an underlying connectedness which can be brought to the surface by virtue of the dialectic process. Immanuel Kant had argued that all experience is subject to certain conditions which exist a priori in the mind. The human mind, he held, is destined never to know reality. Kant's philosophy had established a chasm between philosophy and science. It maintained that scientific enquiry and discovery is constrained by certain a priori criteria. Hegel rejects this approach. According to Hegel, a complete understanding of reality is possible: there is nothing real which cannot be grasped by reason, nor is there anything which is a concept of reason that is not real.

Hegel's dialectic process, then, stands in direct opposition to Kant's thesis. What Kant holds to be transcendental a priori conditions peculiar to all rational human beings at all times, Hegel holds to be historical. That is, conditions that Kant maintains has always existed in the human mind, Hegel says have been developed over time. Hegel's dialectic process contends that the subject is primarily in a state of primitive consciousness. That is, the individual is in a state of consciousness that does nothing but grasp that which confronts him or her at any given moment. Hegel calls this most primitive form of consciousness 'certainty at a certain level of sense experience', or simply 'sense certainty'. From this naive form of consciousness the mind, by virtue of the dialectic process, evolves to a state of self-consciousness. However, human beings only develop through interaction with the world in which they live. Self-consciousness cannot develop in isolation. If consciousness is to develop it requires an object outside itself as opposition to itself. This relationship manifests itself in the form of desire. And to desire is to want something for oneself: to obviate that which is different in 'the other' and to possess that which is different in oneself.

To understand the place of the triadic dialectic process in Hegel we need to look at the most well-known section of his Phenomenology: the Master/ Slave relationship. In this relationship the master holds his position precisely because he keeps a slave. The slave is subservient to the master who enjoys the fruits of the servant's labours. At first examination the master seems to be superior to the slave in that the latter's identity depends on his relationship with the master. However, when we consider that the master also needs affirmation of his identity we understand that, while he has acknowledgement of his slave, in the mind of the master, the slave is just a 'thing' not an independent consciousness. Thus, since to the master the slave is a non-person, his desire or need of affirmation of his identity by this 'other' is not satisfied. In a sense, the master is slave to the slave. When the master reflects on this situation, as Hegel believes he must do, he cannot fail to acknowledge the arbitrariness and injustice of his relationship with his slave, and upon whom he depends, and out of this understanding arises a synthesis: a more rational and humane social arrangement. For Hegel, human consciousness, history, philosophy, science, religion, art, and everything else operates in accordance with this triadic law.

Hegel's dialectic mode, then, contends that the most primitive form of consciousness is 'sense-certainty'. By this he means that primitive beings react spontaneously, or without reflection, to the world around them. The inner conflict generated by others leads to a state of self-consciousness. That is, in virtue of the triadic process, one form of consciousness emerges from others. However, rather than replacing one of the preceding states in favour of another, the new state retains all that is good from both states. Thus, sense-certainty evolves to self-consciousness, and self-consciousness become objective consciousness: the ego-centricity of self-consciousness gives way to the experience of oneself as a meaningful part of a greater, universal consciousness. By letting go of the concept of being — of 'I am' — as a separate or partial entity, the individual becomes aware that the experience of being, of 'I am', is not of oneself as an isolated subject in an alien universe, rather it is the experience, the knowledge and awareness of oneself as an integral part of the whole: the absolute, which is one's true self — Geist.

For Hegel, the term `Geist` means `objective` or `cosmic` spirit. Every condition of thought, or of things — every idea and every situation in the world -generates within itself a contrary thesis — an antithesis. From this conflicting thesis arises a higher, more complex whole. This evolutionary movement is a continuous spiralling development of oppositions, and their merging and reconciliation constitutes the formula of all development and all reality. The evolution of consciousness is the same as the movement of all things: in each there a dialectic progression from unity through diversity and to diversity- in-unity. In the individual, the Absolute rises to self-consciousness and becomes the Absolute Idea. That is, thought realising itself as part of the Absolute, transcending limitations, and drawing together beneath the surface chaos, an underlying universal harmony. By the Absolute Idea,, Hegel means that the Absolute, reflecting on Its own Being becomes, in the virtue of the dialectic process, conscious of Its true self. It is through the dialectic process that the Absolute gains a clear understanding- a clear idea — of its true Self. The task of philosophy is to discover the unity that lies potential in diversity. The task of religion is to reach and feel the Absolute in which all opposites are resolved into unity, that great sum of being in which mind and matter, subject and object, good and evil, are one. God is the system of relationships in which things move and have their being and their significance. As mentioned above, the individual rises to self-consciousness, and becomes the Absolute Idea. What this means is that the dialectic process is not simply a process through which human beings become conscious of the Absolute, but that it is a process through which the Absolute comes to consciousness of Itself. Human beings, for Hegel, are not contingent to the Absolute, not things which can be done without, rather it is that they play a vital and necessary role in the Absolute's coming to realisation of Itself.

Geist, or cosmic spirit, then, is the Absolute striving to each self-awareness, and the locus, the channel, through which this comes to realisation is the human being. It is in virtue of the dialectic process that, through human beings, the Absolute reaches Self-realisation, and that human beings come to a new understanding of their own lives. That is, they see themselves, not as individual fragments of the universe, but as indispensable vehicles of the cosmic spirit. This spirit is not reducible to man; it is not identical with the human spirit, for it is the spiritual reality underlying the entire universe. For Hegel, Art, Religion and Philosophy are humanity's aspects of the self-revelation of the Absolute. The medium of Art is sensation, the medium of Religion is mental imagery, and the medium of Philosophy is imageless conceptions. Each represents different areas of the Absolute Spirit -`Geist` — revealing itself.

Hegel conceived Art to be the absolute form of spirit, a universal phenomenon and an epochal force: a mode of being and a thinking that characterises particular stages in the history of mankind. Thus understood, as the constitutive force and the substantial expression of an epoch, art is no longer. From a purely critical-historical point of view, Hegel concluded that for the modern world art is something of the past. The history of Art evolved through the dialectic process. The spiritual contents of art are `moments` or stages in which the mind reveals itself and becomes conscious of itself through history. Symbolic Art, that is pre-Greek and especially Egyptian and Oriental, represents the seminal stage of the unfolding process, and is represented in the architecture found in objects like the obelisks and pyramids which represent the Absolute trying to realise itself. Out of this emerges Classical Art, that is Greek and Renaissance art, which represents the unfolding of the Absolute as it emerges in harmony with itself. By the time the dialectic process reaches the Romantic period the Absolute has evolved to the stage where it dominates the medium: thinking and refection have superseded art. There is a deeper comprehension of Truth which can no longer be appropriately expressed in Art. Art is no longer the highest and most privileged way in which we articulate and express our understanding of reality, of ourselves or the Absolute. Art has become absorbed into philosophy. It is no longer the primary and spontaneous means in which the comprehensive image of our world is brought to consciousness.

Through the individual the Absolute achieves self-realisation. The dialectic from which Art arises is the dialectic between spirit and matter. For Hegel, Art is conceived as a manifestation of the Absolute and the supreme form of Truth. The function of Art, for Hegel, is to reveal Truth. In Art the individual grasps Truth in its immediacy. The aesthetic experience, for the individual, is the Absolute realising itself through that individual.

Tony Fahey

back

(108) Erin asked:

I think that utilitarianism is superior to ethical egoism. I am trying to explain why. I want to show how a problem is entailed by egoism and how utilitarianism offers a better account. I have found that there is a lack of trust found in ethical egoism, but that utilitarianism definitely includes trust. Are there any other problems with ethical egoism that show why utilitarianism is better?

---

I think that you are going to have a problem with your attempt to show that ethical egoism is superior to utilitarianism. (Hmm. Superior on what scale of reference??) In fact, I think that the only way you are going to be able to succeed in this endeavour is to build a straw-man version of ethical egoism a version that no ethical egoist will accept.

Utilitarianism suffers from three serious flaws what are called the open question argument, the commensurability argument, and the separateness of persons argument.

Utilitarianism maintains that the measure of moral goodness is net aggregate happiness. But unlike some moral theories, Utilitarians do not just define moral goodness in terms of happiness. Instead, Utilitarians must offer an argument to the effect that moral goodness is (or ought to be) equivalent with the net aggregate of happiness. The closest that JS Mill came to doing so was to claim as an empirical (and hence debatable) fact that the majority of those generally recognized as moral experts would agree that happiness is equivalent to the Moral Good. The open question argument says that even if you can say this makes me happy, one can still ask whether this is morally good. Since it is not a tautology that happiness is equivalent to good, it remains an open question that can (and is) debated.

Utilitarianism maintains that happiness is both additive and commensurable happinesses can be ordered (which implies an ordinal scale if not a cardinal scale), and can be summed (which implies a cardinal scale). Mill (and most Utilitarians since) recognized that some happinesses are so much better than others that there are discontinuities in this scale. So it is possible that a small amount of a higher happiness might out weigh any amount of a lower happiness. Even with this refinement, however, it is not obvious that my happiness with some personal achievement is at all commensurable with my happiness at doing the (morally) right thing. Even if one adopts as a guide, at Mills recommendation, the consensus of opinion of those who have experienced both forms of happiness, it is still very debatable that my own happinesses are either comparable or summable let alone comparable and summable with yours.

And Utilitarianism maintains that whose happiness is involved is irrelevant. Utilitarianism takes no account of the separateness of persons that it is fundamentally more important to me that the happiness in question is mine instead of yours, or vice versa. Utilitarianism demands a completely disinterested attitude towards the calculus of happiness. Hence it is quite possible to find a Utilitarian argument that would recommend torturing my daughter if the result was sufficient happiness for a sufficient number of others not a generally acceptable moral recommendation. Now I might find this distasteful result acceptable if it was your child that was to be tortured in return for my happiness. But it is completely unacceptable to me for it to be my child that is to suffer, regardless of how much happiness is the result. This disregard for the separateness of persons means that Utilitarianism can be used to support almost any insult that the majority might wish to inflict upon a minority.

Given these deep flaws of Utilitarianism, I think that the only way you can position Utilitarianism as better (on what scale?) than Ethical Egoism is to caricature Ethical Egoism as a moral philosophy that recommends as morally good an excessive narrow-minded and short-sighted preoccupation with one's own well-being and interests, accompanied by an inflated sense of self-importance resulting in a tendency to be self-centered to the point of considering only oneself and one's own interests. But this, of course, is a straw-man caricature that no advocate of Ethical Egoism would support.

If you are seriously going to compare Utilitarianism with Ethical Egoism, then you really ought to select as a target a version of Ethical Egoism that people actually advocate. Consider, for example, a version of Ethical Egoism called Evolutionary Ethics. This version maintains that the correct measure of moral goodness is the probability that ones genes will continue to flourish when evaluated with a wide-scope and long-term view of the consequences. Since this is the definition of moral goodness, it avoids the open question challenge. Since the focus of the evaluation is on the genes of the acting agent, it avoids the separateness of persons argument. Since we are dealing with probabilities, they are naturally commensurate. And since, in any given circumstance, the probabilities in question are objectively examinable (in principle if not usually in practice), the subjectivity of an evaluation of happiness is avoided. Evolutionary Ethics is not by any description an advocate of an excessive narrow-minded and short-sighted preoccupation with one's own well-being and interests. Its focus is on ones own interests, to be sure. But it advocates a very wide-scope and long-range view of the matter. Hence, it is relatively simple to show that cooperation in the pursuit of mutually desirable goals is a superior means of pursuing ones own best interests. And trust is a natural part of that cooperative spirit. In other words, I disagree with your evaluation with regards to trust.

If you are determined to continue your effort to show that Utilitarianism is superior to Ethical Egoism, I would be very interested to see the result. I would be happy to contribute further comments to your efforts.

Stuart Burns

back

(109) Roy asked:

Could the angels and aliens be hermaphrodites?

---

Given the complete absence of any evidence what so ever that might be used to constrain our flights of fancy, angels and aliens might be anything at all including non-existent.

Stuart Burns

back

(110) Deborah asked:

What came first the chicken or the egg?

---

Evolutionary genetics has finally laid this old saw to rest.

Since mutations take place in the germ-line at the point of mitosis, it is obvious that the egg came before the chicken. Identifying where one species diverges from another is a matter of drawing an arbitrary line across a continuous line of parent-child descent. So no matter where you choose to draw the line in the evolutionary history of the modern chicken, there will be a point where a proto-chicken hen lays a chicken egg.

Stuart Burns

back

(111) Andre asked:

Is philosophical discourse being discouraged in american culture today, especially among the youth?

---

In my non-expert opinion, the answer is clearly Yes!

I base this opinion on two separate factors. First, American culture is, regrettably, a very religious culture. And religious belief does not well tolerate philosophical discussion. After all, the entire basis of philosophical discussion is the investigation and questioning of fundamental beliefs. And religious belief does not permit being questioned or challenged. In America this intolerance of philosophical discourse shows up as much between Republicans and Democrats as it does between Christians and heathens. Both sides maintain that their truth is The Truth and as such is not open to challenge or debate.

Secondly (and largely perhaps because of the first), philosophical education in American schools is locked into a stultifyingly historical approach to teaching philosophical thinking. Nothing will turn off the budding philosophical thinker faster than a demand that they try to get into the mind of some long dead (and largely wrong) ancient philosopher so they can rebuild a philosophical line of reasoning that was erroneous in the first place. What Aristotle or Plato or Descartes or Hume thought about anything should be the last concern of a student being introduced to philosophical discussion.

Stuart Burns

back

(112) Ross asked:

Hi, I once remember an old philosophy tutor I had using the quote 'there has been no new philosophy, just two thousand years of commentary on plato and aristotle'. I seem to remember him saying that the quote was from Bertrand Russell but I am putting it in an essay and need to make sure that it is referenced correctly and cant find the quote anywhere. Does anyone know who said this and/ or which work it is from.

---

Well done for making sure refs/ quotes are accurate (they often aren't). I'm not sure whether your old tutor got it wrong, or whether you weren't listening or have forgotten. The quotation is from Russell's collaborator, the philosopher and mathematician Alfred North Whitehead, in his book 'Process and Reality' (1929). The correct version is:

' The safest general characterization of the European philosophical tradition is that it consists of a series of footnotes to Plato'

Google in 'Footnotes to Plato' and all is revealed

Good luck with essay

Craig Skinner

back

(113) Julie asked:

I was reading a fantasy novel recently, and the theology in this book was very interesting. In the novel, the idea is put forth that WE are God. That man, and his sentience, is a vessel with which the universe is to becoming selfaware. Is this a common belief? Is there a name or an 'ism' for this particular line of thought?

---

You and I, and my cat and dog, are all bits of the universe. We are all aware (sentient). So all of us can be said to be vessels with which the universe has become self aware. It doesn't seem sensible to call any of us God, so I wont pursue that, and it is scarcely a fantastic thought (other than in the general sense that the mere existence of the world and ourselves is a source of wonder). And, obviously, it is a common belief.

Less mundane is the thought that we bring the universe into existence by observing it. Yes, this line of thought has a name: the Participatory Anthropic Principle.

Let me lead up to it.

The Weak Anthropic Principle (WAP) is uncontroversial. It simply points out that, because we exist, the observed values of physical/ cosmological quantities are restricted by the requirement that there exist sites where carbon-based life can evolve, and by the requirement that the universe be old enough for such life to have already evolved.

The Strong Anthropic Principle (SAP) is speculative, saying that the universe must have those properties which allow life to develop within it.

The Participatory Anthropic Principle (PAP) is much more speculative, saying that observers are necessary for the existence of the universe. The idea here derives from some interpretations of quantum mechanics. In the microworld, things exist in a superposition of states, described by a wave function, until collapsed into one definite state by an observer. So the universe is one giant wave function which only collapses into something definite when observed (by us). It's unclear to me why humans get the credit: after all, giraffes, say, observed it before us, so these principles could as well be called Giraffic Principles. But I suppose it is we, not the giraffes, who dreamt up the idea.

The WAP is a truism. The SAP and PAP are speculative.

The FAP (Final Anthropic Principle) states that once intelligence enters the universe, it can never be destroyed. Enthusiasts speculate that, in one form or another, intelligence increases indefinitely, leaving the likes of humans far behind, and ultimately becomes God (and is then able,retrospectively, as it were, to create the universe, say the superenthusiasts).

Please note the suggestion by the late Martin Gardner that SAP/ PAP/ FAP might usefully be referred to collectively as CRAP (Completely Ridiculous Anthropic Principle).

If you can stand any more, try 'The Anthropic Cosmological Principle' by Barrow and Tipler (OUP 1988). It's exhaustive, informative and entertaining, and a nice counterpoint to fantasy novels.

Craig Skinner

back

(114) Gabriella asked:

According to Marx and Engels, how does the proletariat differ from the bourgeoisie?

---

The proletariat is that social class or category of people who have no private property or capital to provide for their means of subsistence. So, they have to sell their labour power to whoever needs it-usually the bourgeoisie or capitalists — in order to provide the means of subsistence.

The bourgeoisie have private property earned from capital. They do not have to sell their labour power. Profit made from the surplus labour expropriated from the proletariat provides their income. This is either reinvested into the capitalist business or used for personal, private use.

As such, each class has antagonistic interests. These interests provide the dynamism for class struggle. Class struggle will entail the triumph of either class; the bourgeoisie will either perpetuate capitalism whilst the proletariat will overthrow and end capitalism to install Communism.

Martin Jenkins

back

(115) Marcus asked:

'Who was the first to use the phrase 'necessary truth'? I've heard that Leibniz introduced the concept: is this true was the concept known under a different name before his description of it, or was he not the first to use the phrase? Thank you very much

---

The concept of necessity and necessary truth was important in ancient Greek philosophy from Parmenides onward. Aristotle uses it in his definition of 'syllogism' as 'an argument in which, some things being supposed, something else follows of necessity because of the things supposed'. Here he defines a valid argument. He also sets forth the fatalist argument allegedly showing that, since true propositions about the past are (now) necessarily true, so must true propositions about the future be (now) necessarily true.

Leibniz distinguished 'truths of reason' and 'truths of fact' . These correspond, respectively, to necessary truths and contingent truths, or maybe more accurately, to analytic truths and synthetic truths), similar to Hume's later distinction of 'relations of ideas' and 'matters of fact'.

What Leibniz did introduce into philosophy was the notion of possible worlds (although this idea can probably be discerned in John Duns Scotus' 13th Century logic). Scholars argue as to what extent Leibniz explicitly connected this notion with his accounts of 'truths of reason' and 'truths of fact'. But clearly, truths of reason are true in all possible worlds, and truths of fact are true only in some possible worlds — allowing a nice account of how 'Caesar crossed the Rubicon' is a contingent truth although it is necessarily true to say (now, of the actual world) that 'Caesar crossed the Rubicon'

Craig Skinner

back

(116) Romer asked:

Does satan really exist?

---

Satan is as real as Santa Claus, the bogeyman, the tooth fairy, the Loch Ness Monster, or any anthropomorphic entity, benign or malign. Moreover, it can be said that the belief that Satan 'really exists' will persist as long as educators continue to employ these myths to indoctrinate the undeveloped minds of innocents. Whilst it can be argued that some of these beliefs, such as Santa Claus and the tooth fairy, are innocuous enough and can symbolize the altruistic nature or natural inclination to sociability of human beings, it is when they are used by those who hold the reins of power to subdue, control and oppress others that they become instruments of malevolence. It should also be said that whilst it is accepted that the developing mind, through the use of reason, has the wherewithal to reach a stage where it can deconstruct many of these myths, it must be argued that there are those attached to certain institutions (particularly religious bodies) who not only fervently believe that the more pernicious of these myths are in fact real, but who also accuse those who refuse to subscribe to the perpetuation of such beliefs of being a threat to social stability (whilst in fact they are only a threat to the stability of the particular institution responsible for perpetuating such myths), or even heresy.

Tony Fahey

back

(117) Rebecca asked:

I'm writing an essay about Aristotle's categories in particular the differentiation of 'substance' from the other categories. I am struggling to write a lot on it and wondered if you had any words of wisdom? I also need some well recommended books as all the ones I have are irrelevant!

---

Substance can be problematic, and I suspect that you have difficulties with the other categories because they are so simple and obvious. One good way would be to write a story of an extended family: there you have quantity (subdivide them: so many males/ females; adults/ children); where/ when each member lives gives you time and position as well as their relations (uncle, brother-in-law etc). Then what they own and their occupations (possession, activity), if they are sick (passivity) and some of their attributes (good, kind, handsome) for quality. Now write another story and replace the people with things in a supermarket. All those items also have qualities, attributes that govern your choice when you buy them. That leaves you with substance. A substance is a thing that can move and work by itself: living things, people. They are indivisible (all have something like souls). Computers and cars need to be switched on; balls have to be thrown, so they are not substances. Finally living things have a will; objects do not. Living things especially strive to survive and procreate, objects do not. The difficulty with substance is that in religious philosophies the problem crops up that God is the only substance, we as creatures live by the 'concession' of God to let us have a limited sort of autonomy (substantiality). But you don't have to get involved in these issues. You only need to be aware that today we tend to misuse the word 'substance' (e.g. drugs are referred to as substances). But this is an instance of changes in common language use. Although it's got nothing to do with Aristotle, you could mention it (i.e. drugs are not substances in the real meaning of the word). Hope this helps.

As for books, there is Guthrie, History of Greek Philosophy, Vol. 6, 'Aristotle: An Encounter', and 'The Cambridge Companion to Aristotle', ed. by Jonathan Barnes. They will make a good Aristotelian of you!

Jurgen Lawrenz

back

(118) Kim asked:

Explain Aristotle's Golden Mean. Do you think it has any value as a guide to ethical behaviour?

---

For Aristotle, the good for humans is rational activity of the soul in conformity with excellence (virtue)

To act in this way is a matter of observing the principle of the mean in relation to us (finding the appropriate response between excess and deficiency in a particular situation).

The idea of the mean features in Aristotle's views on medicine, perception and politics, and in the views of the Pythagoreans. As a good work of art is spoiled by adding to it or taking from it, so virtuous activity is spoiled by excess or deficiency, and secured by observing the mean.

Aristotle distinguishes between the mean in the object (the arithmetic mean) and the mean relative to us (neither too much nor too little for the particular circumstances).

Recommendation in he first sense, the midpoint between two extremes, would simply be a counsel of moderation in all things, perpetual sober caution, a rather dreary doctrine.

But this is not his position. The mark of virtue is observing the mean 'in relation to us'. By this is meant 'to have the right feelings at the right times on the right grounds towards the right persons for the right motive and in the right way is to feel them to an intermediate....to the best degree', Strong anger, say, may be appropriate in some circumstances, moderate anger may be excessive in others. I must also take account of my natural inclinations. If I am inclined, say, to be overcautious, a response a little more toward the rash end of the spectrum than my first inclination will be appropriate.

Nothing mediocre about the mean.It is the peak of suitability. Aristotle likens it to an archer hitting the target, taking account of the wind, angle of the flight path, bow, and degree of steadiness of his hand. Many ways to get it wrong (missing the target), one way to get it right.

Aristotle declares that each of virtues is somewhere between two opposing vices. Courage for example is a mean on two scales — first, as a response showing degree of fear, between cowardice and recklessness; secondly, as a response showing degree of confidence, between timidity and rashness. He cites five scales for even temperedness, relating to frequency, degree, duration,people and provoking circumstances. Things start to get a bit complicated and forced. Honesty, for most of us, is not a mean between two vices. It is just a virtue, opposite to the vice of dishonesty. But Aristotle tries to shoehorn it in to his scheme by implausibly restricting it to talk of one's own achievements, saying it lies between boastfulness and self-deprecation.

In summary, virtue is a state of character, concerned with choice, lying in a mean relative to us determined by reason as the practically wise person would determine it.

The doctrine might be of practical value to us if it included a useful way of determining where the mean is ie what is appropriate to a particular situation. But there are no algorithms or formulae from Aristotle. The mean is determined by reason as the practically wise person would determine it. In short, if one wants to be a virtuous person, then one must act as a virtuous person should act — not much help. It's not as if there is any shortcut to acquiring practical wisdom. Aristotle doesn't make it completely clear (to me) how it is acquired, but I think he sees its acquisition by receiving a good upbringing and education, by reflection, by experience, and by taking part in a society with shared values. A fair description of how most people try to bring up their children these days. A bit deflationary if you want to get practical wisdom by study, or develop a rigorous theory of ethics, but he may be right. The general advice not to be overenthusiastic or too diffident, to be emotional to an appropriate degree, to take account of our own weaknesses, and to have role models (people with practical wisdom) is sensible, but might be given by any pub pundit or self-help guru.

The doctrine of the mean, for me, is not one of the enduring highlights of Aristotle's ethics. Opinions about it among experts on Aristotle vary from most-valuable-if-interpreted-rightly to best- forgotten

Craig Skinner

back

(119) Paul asked:

I have a friend who says he is one third atheist, one third theist, and one third agnostic. Is that legitimate?

---

Firstly Paul, I'm a little suspicious that this question may be a wind up by you on the Ask a Philosopher panel, or a wind up by your friend on you (can one really not know the correct way to spell 'atheist' and 'theist?). Secondly, since his trisecting of his beliefs is so precise, and given that atheism is the direct opposite of theism, and agnosticism neither one nor the other, my initial reaction to this question is that your friend is either: a joker, a procrastinator, or a budding philosopher. However, whilst any one of the above may be the case, and starting with the premise that one cannot be an atheist, a theist, and an agnostic at the same time, I am prepared to accept that there is something to be gained by putting my skepticism aside and treating your question with a degree of seriousness.

Allowing that your friend's declaration, whilst somewhat ambiguous in its formulation, is genuine, it seems to me that the rubric of budding philosopher, may be that under which he best may fall. Thus, giving him the benefit of doubt, and given that one cannot be all three at the same time, I will take it that what he means is that he is struggling to decide which of the three isms best describes his philosophical, religious, or spiritual position, and that at different times he finds drawn to one over the other two. If this understanding is correct, I would say that whilst this position may be legitimate, it is not one that is easily resolved and that many thinkers, philosophical, spiritual, and religious, find, during the course of their lives, that this is a dilemma which they will confront many times — I would add that being open to the view that re-examining one's beliefs is essentially what philosophy is all about. That is, in the pursuit of wisdom, knowledge, and truth, one comes to accept that there is always the possibility that one may be wrong in one's beliefs. In my own case, having given much time, thought, study, and energy to such issues, if I were pressed to state my own position on the subject under discussion, I would have to come down on the side of agnosticism. That is, in regard to the issue of the existence of God, I am forced to conclude that we simply cannot know, with the degree of certainty claimed by theism or atheism, that such an entity exists or not. Paraphrasing Wittgenstein somewhat, I would say that 'that of which we cannot know, we should stand in silence'. It seems to me too that too much has been attributed by too many to that which they cannot possibly know. However, I would add the codicil that whilst this is my current position, it does not mean, given further evidence, that this position may not be revisited, reassessed, agonized over again, and even amended the months and years to come.

Tony Fahey

back

(120) Andrew asked:

I'm new to philosophy and believe that God cannot be proven or disproven. I've come up with the following analogy to 'prove' my beliefs.

The idea of God can be understood, but the mind cannot comprehend God, much like the idea of colors never before seen can be understood, but the mind cannot comprehend what such a color would look like.

Basically, a person can understand ideas of God being omnipotent, all powerful, all knowing, etc etc, but when a person actually tries to comprehend God, (s)he cannot.

Does this hold water?

---

Reads to me as similar to Immanuel Kant's examination of the Ontological Proof for the existence of God contained in his Critique of Pure Reason.

Namely, God has the characteristics of omnipotent, all knowing etc which people can understand. However, it is not established that He exists. If he exists, then he would have those qualities. Whether he exists is another matter. So understanding the characteristics of God does not logically entail his actual existence. God's existence is not proven. All that is established is an idea of the concept called God.

Further, Gaunillo's criticism of St Anselm's version of the Ontological argument can be invoked. Namely, if I can conceive of possible colours although no such colours have been experienced then on the same grounds, anything conceivable can be proposed as possibly existing-unicorns, perfect islands and so on. The argument becomes nebulous and unconvincing.

At most, Andrew, you've said that the human mind can understand things it has not yet encountered such as colours and God. Even assuming the assumption that the mind could understand them is correct, this still entails the view that Gods existence is a possibility not an actuality. As a possibility, Gods existence cannot be proven. At most, faith in his existence is appropriate. I don't believe your argument provides a natural proof for the existence of God.

Martin Jenkins

back

(121) Alan asked:

Do we have free will?

Just before I typed this, I toyed with the idea of not submitting the question. Then I decided to submit the question although it seems that I could effortlessly have decided not to submit the question. This seems to be a process of me doing the deciding. Is it not meaningless to say that it is just an illusion that I have a voluntary choice of whether or not to submit this question when it feels so real that I have this choice?

---

Geoffrey Klempner has already answered this in impeccable philosophical style, so there is no need for me to add anything. But at the bottom of his reply I find a little quote from Benjamin Libet that opens up a problematic little window on the issue. It has a context that has been suppressed for many reasons and seasons — namely its biological aspect; and, speaking candidly, I have never understood why this is consistently ignored. Why do people keep arguing as if neurons were little bits of plastic instead of living things?

However, my reply will be couched in this context.

1. "Making a choice". All biological entities make choices. The meaning of the word "life" includes the definition of its bearers as autonomous agents. Their choices change the habitat. Survival entails making the right choices often enough. Creatures evolve and adapt on this basis, or become extinct.

2. "Free will". This compound noun is analogous to another: dead corpse. As it is futile to deny that a corpse is dead, so it is futile to deny that will is free. A will that is not free is not a will.

3. Corollary: No creature HAS a will. They ARE a will.It is a language problem, as you can see. The word "has" insinuates an independently existing (or illusionary) property that you acquire.

4. The only relevant criterion left over after this is that there are constraints on its EXERTION. It is easily understood that a man with a knife at his throat might agree to abandon his will to eat a steak instead of fish. It does not of divest him of his will, only of its object.

5. Finally, if free will is an illusion, yet everyone shares it, it is not an illusion. It is a matter of consensus, like accepting that things we all sense in common actually exist.

6. The upshot is that free will does not have to be proved. There is nothing to argue about, as it is simply a fact of nature. It has, on the contrary, to be proved that it is not a fact of nature.The two issues that can be discussed are (a) the degree of consciousness to which any creature is aware of its choices, and (b) the constraints that work against its exercise.

7. However, the "Libet Factor" invites one further debating point to be aired. There is also a collective will, qualitatively very different from the individual will. Its discussion seems mostly the preserve of psychologists. But social animals also exhibit collective wills, and so do the neurons in your brain. Once that thought seeps in, Libet's problem loses most of its sting. The neuronal collective is constantly evaluating information, and must be assumed to have "contingency plans" at the ready for any sort of sensory impingement. Some of this strategy is likely to be localised at the site where it occurs. Your "decision" to lift a finger would therefore be "in your brain" long before you implement it as an option. It doesn't strike me as paradoxical. It's only our sequential thinking habits that make a mystery of it. But this is too complex an issue to pursue here, and I think I have made the points about "free will" that seem important to me.

Jurgen Lawrenz

back

(122) Adam asked:

I have a question for you regarding an answer you gave to a person struggling with the idea of an infinite past. I've discussed this with friends of mine and your statement 'A universe stretching infinitely back in time is no more difficult to conceive than a universe stretching infinitely into the future.' appears to be the standard response. I find this response problematic because of a puzzle that arises from an infinite past. Suppose that there is an infinite past, and that this current moment (Time X) is temporally connected to another time previous (Time Y) that is infinitely far away from it. If we suppose that time Y happened and then time X happened, how can it be that they were infinitely far away from each other. It seems impossible that two moments can both happen if they are infinitely far away. What is troubling about this example is that every moment in time suffers the same fate, that other moments which are connected to it happen as a part of the same chain, yet on a link that is infinitely far away. If no two moments in time are infinitely far away from each other then I don't see how there is an infinite past, since any two moments can be finitely (even if extremely large) compared with one another.

---

Philosophers have grappled with precisely this issue for as long as there has been philosophy — including (so as not to forget) — theology. However as you mention a 'standard response', I am prepared to give a 'non-standard' response and you can mull over the difference.

Time is an homogeneous field. Accordingly there are no 'moments', no time X or time Y. If you reflect on what is meant by 'now', you will find it to be an arbitrary assignation of an instant of experience. For us, therefore, to project this instant into the past or future is equally arbitrary. None of the Times X or Y you mentioned in your question are denotations. They do not refer to time, but to you, the agent of cognisance.

Now repeat all this for space, replacing instants with points. The conclusion will be the same. You will see in a moment why this is necessary

Homogeneous fields are indiscernible. Whatever time 'is', it must be seamlessly continuous, and so with space. Accordingly time is not a succession of instants, but of events. Similarly space is not a volume, but a collection of objects. These we can perceive and extrapolate them upon a presumptive time and space, but this does not oblige the universe to supply us with an actual time or actual space.

You might see now that your question is incomplete. The increments from Event A to B, and between Object A and B are matters of perception. Perception cannot deal with infinites; and therefore it cannot deal with an infinity of increments. We can only judge about them from observing their temporal relation to us, and their spatial distance from us. Both of these belong together, as you know, because events and objects do not ever sit still.

A full version of your question would therefore have to be stated as an ensemble of three items, viz.:

An event/ object passes over (moves) spatial points P1,2,3 etc in instantaneous Time1,2,3 etc. How can we correlate these?

Answer: only if we can identify the smallest possible instant of time and the smallest possible point in space. I think you will agree that this a self-contradictory proposition. For any motion from P1 to P2 implies the possibility of a P1/ 2,1/ 4,1/ 8 etc to infinity, and likewise with T.

Which leaves us with only the one conclusions mentioned above: The arbitrary assignation of perceivable increments for both. In a word: we cut into any arbitrary point on the number continuum and assign a value to that point (e.g. 0). This gives us the opportunity for calibrating other points at our convenience (check out the 'Dedekind Cut' for further information).

In short, we stop pretending that time and space 'exist' as pointillist continua, independent of ourselves. We have to set up such a points continuum first and then impose it on time and space to render both of them intelligible.

It is what, for example, we do when we posit a 'Big Bang'. It enables our measurements of time and space. (By the way, I've never met a scientist who believed in the actual occurrence of such a bang: I think they are all responsible enough to recognise that it's all in the mathematics).

Let me end by expressing my hope that I've explained something useful to your thinking!

Jurgen Lawrenz

back

(123) Len asked:

My question has to do with language and in that sense it could be a linguistics or a philosophy of language question. Of the two, I'm not really sure into which category it falls.

If you agree or disagree with a statement, it seems to me this is an absolute. However, on many psych tests employers use these days for candidates seeking to fill the open position, they give choices of 'agree,' 'strongly agree,' 'disagree' or 'strongly disagree. For example; if the the statement is 'The sky is blue,' I can either agree or disagree with the statement. How could I further agree or disagree about the state of the color of the sky or any other statement for that matter. If you and I both disagree, how could either of us disagree 'more' than the other? Herein lies my question: How can you assign an adverbial quantifier to something that I believe is an absolute? I'm pretty sure I'm not the only person who thinks this way so could tell me the difference between agree and strongly agree?

---

Simple statements of fact e.g. 'it is raining' do not leave any room for complex judgements, they are merely true or false. However other complex statements of fact such as 'Playing violent video games does not encourage children to behave violently' can call for complex judgements since we cannot know whether they are true or false but we can say how likely we think it is that they are true given our present state of knowledge.

When we say that we strongly disagree with this statement we are not contradicting the idea that it is true or false , however its truth value has not been definitely established and this leaves room for complex judgements about the likelihood of it turning out to be true.

It is easy to be mislead by simple examples. Many statements are so complex e.g. Politicians do not understand the economy' that it would seem to be impossible to establish any definite truth value for them. That does not prevent us from having beliefs about them being true or false.

Shaun Williamson

back

(124) Callum asked:

Recently some thoughts came into my head that worried men greatly. The widely accepted (at least I think it is but am hoping isn't) view of Determinism has worried me purely because it means everything I do was always going to happen (taking away value from my achievements) and therefore makes criminals not bad (not that I'm thinking of being a criminal).

Are there any credible philosophers or experiments that resist determinism or (if my words are ambiguous) believe we have choice to do otherwise e.g. a person walking in a shop has the possibility of stealing or not stealing. As this would put me at ease.

---

I sympathise with anyone who gets caught up in the great nasty determinism debate. I myself spent years arguing and debating it with all sorts of people, trying to find a hole in the arguments of determinism.

Well, I found it. And the most astonishing things is this: it's totally obvious. The answer is: the boot is on the wrong foot.

What I mean is this: Where does determinism come from? (a) Religions, superstitions, fear of ghosts, spirits, fear unexplainable events and things which leads in all cases to a belief in supernatural powers; (b) the expectation of scientists that physics can 'in principle' solve all questions from the big bang to the big crunch, so that meanwhile they feel entitled from the knowledge that has already been acquired to pronounce the universe to be a kind of pre-programmed machine where nothing ever happens that is not already disposed in the elements. You will observe that science cannot explain the nature of free will, therefore they deny free will by ramming the insupportable point of view down your throat that free will 'can be explained chemically'.

But you can see from this that determinism has no leg to stand on. It is in both cases a pretence by the relevant human authorities to a kind of knowledge which in plain fact they do not possess. The classics statement for scientific determinism was issue by Laplace, who knew nothing about thermodynamics, let alone quantum physics. But his belief is an article of faith for science, as God or Allah is article of faith for believers.

You need only look at today's genetic determinism to see how this insalubrious attitude spreads across society as a doctrine. Is genetic determinism proved? By no means! We know almost nothing of their origins and little more about how they actually work. That little serves as 'proof'. Not good enough!

Therefore your proper response to determinisms of every colour would be: The onus of proof is your shoulders. Prove to me, Mr Theologian or Mr Physicist, that determinism meets the facts, and in doing so examine your suppressed prejudgements and leave them out of the argument.

In other words: you, the victim of these pretences, must not allow yourself to be bulldozed into defending your free will. Every creature on this earth has free will: it is an attribute of life. But science does not know what the word 'life' means. So demand proof from them that they know what they are talking about!

Up to this moment, I've been speaking to you casually, as probably an older person to a younger one, with a view to helping you to keep you at arm's length from a trap that many unsuspecting people fall into into.

However, if you wish to get the 'full Monty' of a rigorous philosophical argument, let me recommend my book 'Leibniz' The Nature of Reality and the Reality of Nature' to you. Chapter 7 is a full-fledged philosophical description of how one great thinker, Leibniz, destroyed determinism from the roots. This is not easy reading, and you will need a lot of philosophical nous to get the gist of it. On the other hand, may be you could cajole your professor into putting it on the reading list of the course on determinism!

Jurgen Lawrenz

back

(125) Andrey asked:

What is supernatural?

All right, I know this might sound really broad but I think I can specify and clarify it a little better if I explain what I mean. According to Wikipedia supernatural is anything above or beyond what one holds to be natural or exists outside natural law and the observable universe. If a supernatural being/ thing exists, there could be only two scenarios; one would be a supernatural being/ thing which can interact with everyday material that we can sense (ie. matter, photons, gravity weak force, strong force, electromagnetism essentially anything that exists) and in that case it would have to have its own mechanisms which we can observe test and learn (there is an assumption here that everything that interacts with us has a 'mechanism' or a set of rules by which it behaves), which would mean it is 'scientific' and not supernatural. The other scenario is if it couldn't interact with our physical world, in which case it would be outside the natural law of the observable universe, but even then the question remains of how does it work. So I guess my real question, which I think is the same as above, is 'can something exist without having a mechanism?' and if it does, do we care, or can we just say it doesn't exists because as far as we are concerned it doesn't 'matter'?

PS: there is one other thing I have to clarify, the mechanism isn't necessarily referring to the 'real' world. For example, if we are living in the computer and if a chair came into existed suddenly in my room it still would have to have a mechanism such as the code or a program which was executed by the computer in which we live, and thus the whole process had a 'mechanism'.

---

There are no definite answers to these questions. All I can do is draw your attention to the most sophisticated form of theology (note theology is not philosophy).

God exists outside of time and space so the idea of a mechanism by which God interacts with the world does not make sense. God created the world out of nothing and holds it in existence so God's creation is timeless and outside of time. This has nothing to do with the sort of creation that we are familiar with i.e creating things out of other things. It has nothing to do with cause and effect.

If God exists he doesn't have a mechanism in your sense.

Shaun Williamson

back

(126) Len asked:

My question has to do with language and in that sense it could be a linguistics or a philosophy of language question. Of the two, I'm not really sure into which category it falls.

If you agree or disagree with a statement, it seems to me this is an absolute. However, on many psych tests employers use these days for candidates seeking to fill the open position, they give choices of 'agree,' 'strongly agree,' 'disagree' or 'strongly disagree. For example; if the the statement is 'The sky is blue,' I can either agree or disagree with the statement. How could I further agree or disagree about the state of the color of the sky or any other statement for that matter. If you and I both disagree, how could either of us disagree 'more' than the other? Herein lies my question: How can you assign an adverbial quantifier to something that I believe is an absolute? I'm pretty sure I'm not the only person who thinks this way so could tell me the difference between agree and strongly agree?

---

Len, that's a nice question! I guess — trying to make sense of something that I agree isn't exactly crystal-clear — you could think of those questions as asking about your level of confidence. I believe some things more strongly than others; I am pretty certain that I will die within the next 100 years, but considerably less certain that I'll go to the shops tomorrow. So I might answer 'strongly agree' to the first and only 'agree' to the second. You might not think of this as literally believing some things more strongly than others, in the sense of belief itself being a matter of degree (although plenty of philosophers do think that belief comes in degrees). You might just think of it as a matter of (a) do I believe this? (Yes or no?) and (b) to what extent do I take this belief to be justified?

That said, I think it depends a bit on the question. For example when you're asked to agree/ strongly agree/ etc. with something like 'I enjoyed taking this course', or 'the level of service was acceptable', it's unlikely that your answer could reflect your level of confidence. ('Did I enjoy taking the course? Well I'm not sure — I think I did, but maybe I'm wrong ...') So in those kinds of cases it looks like you're being asked whether you enjoyed taking the course a lot, or quite a lot, or not very much, or not at all (or the level of service was excellent, not bad, pretty mediocre, etc.). I agree that asking whether you agree/ strongly agree/ etc. with the proposition is a pretty inaccurate way of describing what's going on here!

Helen Beebee
Director
British Philosophical Association

back

(127) Adan asked:

Hi. I just had a question in regards to 10 necessary and sufficient clauses. They are as follows:

Are the following sentences true or false ? In each case explain the reasoning behind your answer.

1. Being alive is a necessary condition for having a right to life.

2. Being human is a sufficient condition for being a mammal.

3. Having US citizenship is a necessary condition for becoming president of the US.

4. Having US citizenship is a sufficient condition for becoming president of the US.

5. Being a woman is a sufficient condition for being human.

6. Rain is a sufficient condition for the ground's being wet.

7. A perfect SAT score is a sufficient condition for getting into Notre Dame

8. Being 7 feet tall is a sufficient condition for being at least 5 feet tall.

9. Completing all assignments is a sufficient condition for getting an A in PHIL 20101.

10. Committing acts that scare civilians is a sufficient condition for being a terrorist.

Thanks!

---

What fun. The questions deal with a nice mix of logical, legal, physical and practical necessity/ sufficiency

1. Being alive is a necessary condition for having a right to life. I think True. Being alive is obviously not sufficient for having a right to life (TB germs are alive but nobody thinks they have rights) But is it necessary ? For existing things, the answer seems clear. A thing cant have a right to life unless it already has life. So books, houses and other inanimate things cant have such a right. For non existent things, it's less clear. We sometimes say future generations have a right to, say, a world without devastating, maybe lethal, global warming. So do we have an obligation to any future individual (who is not now alive) so that this individual has a present right to expect us to discharge that obligation thereby recognizing and protecting her right to life ? I'd say no, future individuals, being non existent, cant have rights. Our present policies will determine which individuals get born. The present policy by many Western women to have children when aged 30-plus means that the children who would have been born had those women gone for teenage pregnancy don't get born at all (the sperms and eggs that would have formed them just go to waste). Have we violated the right to life of these never-existing children ? I don't think so.

2. Being human is a sufficient condition for being a mammal. True (taking 'human' to mean 'a human being', rather than including, say, a piece of human DNA, which obviously isn't a mammal). Logical sufficiency. Mammals include the species homo sapiens (human beings) and many other species, so that being a member of any one of these species is sufficient to be a mammal.

3. Having US citizenship is a necessary condition for becoming president of the US. True. Here, the necessity is legal rather than logical. The US constitution requires candidates to be US citizens, US-born and over 35 years old.

4. Having US citizenship is a sufficient condition for becoming president of the US. False. Other things are needed as well. You must be eligible to run for the job, you have to get more votes than any other candidate. Having US citizenship isn't even a sufficient condition for being eligible. Sadly for Austrian-born Arnold Schwarzenegger, who, as Governor of California, tried but failed to get the Constitution amended so that he could run for president, you have to be US-born.

5. Being a woman is a sufficient condition for being human. True. A woman is human by definition being an 'adult female human being'.

6 Rain is a sufficient condition for the ground's being wet. True. Given that the ground is wet, one condition that is enough to explain it, is rain. Of course rain is not always sufficient to make the ground wet. Rain at sea doesn't (no ground) and rain wouldn't if the ground were very hot, say 800 degrees C (rain just vaporizes) or very cold (rain instantly freezes so ground is icy not wet).

7. A perfect SAT score is a sufficient condition for getting into Notre Dame. Don't know. It's a matter of practical sufficiency — true if regulations say so and assumed background conditions met. I've no idea what a SAT is (age-related ignorance I suppose). I assume that the only life forms on earth to get perfect SAT scores are humans, and that by 'getting into Notre Dame' you mean enrolling at the University rather than visiting the cathedral. So, is being a human with a perfect SAT score enough to get you in ? Say the regulations state that a SAT score of 70% or more gets you in, then your perfect score is sufficient. So the answer is yes. The regulations make background assumptions eg that you are human (a perfect SAT score from an ET might or might not suffice to get the alien in) and not dangerous (a perfect SAT score might be insufficient in an applicant recently-released, hopefully 'rehabilitated', after 20 years in jail for compulsive arson targeted exclusively at University buildings in various countries. More general background conditions are assumed to be met eg the Earth continues to exist, Notre Dame isn't closed down because it turned out to be an elaborate money-laundering scam etc.

8. Being 7 feet tall is a sufficient condition for being at least 5 feet tall. True. Logical sufficiency.To be at least 5 feet tall means to be 5 feet tall or to be more. Seven feet tall is more than 5 feet tall and so is sufficient.

9. Completing all assignments is a sufficient condition for getting an A in Phil 20101. Don't know. Do you mean 2010, or are you a visitor from the 201st century, or is 20101 a course number rather than a date? Anyway, answer as for 7. (depends on regulations and assumed background conditions). I doubt if just completing assignments would suffice, they could be done so atrociously as to fail to merit any grade never mind an A. But, for all I know, maybe 'completing'' means 'completing to the satisfaction of those setting them', and different assignments are set depending on the grade you are aiming for, so that completing all assignments is sufficient.

10. Committing acts that scare civilians is a sufficient condition for being a terrorist. False. Hinges on definition of terrorist. Those committing acts that scare civilians would seem to include drunken revellers any night in UK town centres, football hooligans, police officers on duty at violent demonstrations, and I wouldn't call any of these terrorists. To do so would be to define 'terrorist'so broadly that the word becomes useless. At the same time, I recognize that a judgment that somebody is a terrorist depends a lot on your perspective. One person's terrorist is another's freedom fighter.

Craig Skinner

back

(128) Sydney asked:

I had my first class in critical thinking earlier today and my professor was unable to tell me if instinct was epistemic luck. I was wondering if you might be able to help answer this question?

---

Epistemic luck about your instincts is precisely the same thing as epistemic luck in a lottery. The point is that the word epistemic relates to knowledge: so that if your instincts allow you to survive, it means you are the lucky beneficiary of good survival instincts. The same with a lottery: if you picked the right numbers, then you win, so that after the event you are entitled to say, 'I had a good feeling about those numbers'. Most philosophers would of course dispute that this is real knowledge, because instincts are subconscious, and the same again with your 'good feeling'. But this is something that cannot be resolved by argument. Your instincts 'know' something; and since they are your instincts, you 'know' what they know, even if you're not aware of it. The difficulty with numbers, however, is that we would have to invoke some metaphysical connectedness between yourself and a future event, and this is probably aiming a little too high. Finally there is another issue of difficulty. If we are talking about you, personally, and your good instincts, we are taking it out of a realm where instincts are indispensable. You are presumably living in a city where (on the whole) you are well protected, like everyone else, against bad instincts. This is the route humans have taken for the last 6000-odd years. The effect of this is that epistemic luck is virtually nullified. I suppose this is a strong reason why the term has limited applicability — strictly only to animals, but they are not self-reflexively conscious and so can't use it. At any rate, I hope this helps you over the hurdle, and clears up for you why the term is not really an issue on which you should get bogged down.

Jurgen Lawrenz

back

(129) David asked:

I learned about a philosopher in college who spoke about how the categorization of our perceived world was bad because we are placing names and values on things that don't exist.

One example my professor gave me was the question 'what is blue?' because you can not explain what blue is without giving an example it doesn't truly exist except as a perceived notion.

I can not remember the name of this philosopher and I would like to go back and read more on the subject. I believe his first name was George but that could be wrong.

Hopefully you can point me in the right direction, thanks.

---

It seems to me that the person your query relates to is the philosopher/ theologian George Berkeley. Although referred to as one of the 'British Empiricists' Berkeley was actually born in Kilkenny in Ireland in 1685 (he died in 1753). Berkeley's central thesis was that there is no such entity as a physical world, or matter, in the sense of an independently existing object. Rather it is that all that we ordinarily call physical objects are actually collections of ideas in the mind. Berkeley held that all we can ever know about objects is merely the ideas we have of them. The appearances we experience are the very objects and the appearances are sensations or perceptions of a thinking being. His most famous saying is 'esse est percipi' — 'to be is to be perceived'.

According to the 'esse is percipi' thesis, all the things surrounding us are nothing but our ideas. Sensible things have no other existence distinct from their being perceived by us. This also applies to human bodies. When we see our bodies or move our limbs, we perceive only certain sensations in our consciousness. Using a series of arguments, often called by philosophers as the 'veil of perception', Berkeley argued that since we never perceive anything called 'matter', but only ideas, the view that there is a material substance lying behind and supporting these perceptions is untenable. For Berkeley everything was mind-dependent: if one cannot have an image of a something in the mind, then it fails to exist — hence his thesis 'to be is to be perceived'. Berkeley's response to those who argued that if there were no material substrate behind our ideas, how is it that things persist when no one perceives them, was to argue that all our perceptions are ideas produced for us by God.

One of the most ardent of the many critics of Berkeley's philosophy was Dr. Samuel Johnson who is said to have famously refuted the eminent bishop's theory of immaterialism by kicking the bishop's ass and shouting 'Consider yourself refuted!

Tony Fahey

back

(130) Derrick asked:

I recently read a report on a request made by the Russian Minister of Finance who asked that Russians smoke and drink more as the country needed the revenue. Is this not as a result of their adoption of the capitalist system, a system that has been faulty since its exception?

Communism did not work and the West did its utmost to see it failed, the Capitalist system is no better as it benefits only a small segment of the population and the myth of the creation of wealth which is now the holy grail is all smoke and mirrors and has value as long as the paper Dollar retains its value.

When it comes to finance we have people who are awarded the Nobel prize for the creation of systems that are supposed to improve how systems work, I have yet to see this actually effect anything, in fact things keep getting worse.

We are told how well we are doing while pensioners don't know how they are going to survive.

We are also told the markets know best, best for who? A shareholder's interest is never a countries interest, self-interest is the only consideration.

What do we need to break the cycle of greed, a 3rd World war? But then war is profitable.

I understand Greek Philosophers had thoughts on matters of finance, does Philosophy have solutions or is man so flawed that we are too far into the abyss to pull back?

---

Derrick's question is timely. I have been seriously considering whether I want to continue as Editor of Philosophy for Business, the e-journal which I launched in November 2003, in an atmosphere of heady optimism that a 'reformed' version of Capitalism, or 'Capitalism 2.0' was just around the corner. The philosophers would show the way.

While the readership of the e-journal has steadily increased, the flow of articles has significantly declined. There are undoubtedly business ethicists out there, marketing their expertise, but they've gotten smart. They know the things that companies and corporations don't want to hear, so they don't tell them. All the talk is of how, by increasing the company's ethical quotient, or boosting its CSR strategy, or even developing the 'emotional intelligence' of managers and executives, profits will inevitably increase. Cast your bread upon the waters.

Don't mistake these remarks for cynicism. I think that the business ethicists are doing the right thing, the only thing they can do, by working for evolutionary change and not trying to start a revolution. If things seem to be going very slowly one has to remember that the system has massive inertia. Change will come, but it will come slowly. At least, that's the optimistic forecast.

But too slowly for the likes of me. The great slogan of defenders of Capitalism (of which I am one) is 'freedom'. I believe in freedom. You can't have freedom without the marketplace, where goods, commodities and services are freely bartered and exchanged. That's the way it works. This isn't caving in to human 'selfishness' but rather the only way the game can be played. There's a place for ethics, provided you recognize that ethics and CSR are things you have to budget for. In some years you have more to spend and in other years less.

What really hurts me is seeing how unfree this same system has made us. If someone offers you work you don't waste time thinking whether you really need the money (unless you are lucky to have an inheritance or private income). It doesn't matter if you are a senior executive or do the postal round. Now, as a response to the recent downturn, belts are being tightened once more, we are being asked to work harder and longer — while we avert our eyes from those unlucky enough to be cast on the scrapheap.

We are prisoners of our own expectations — for example, that the only healthy state for an economy is growth. You must consume more, so that the money can go round, job opportunities increase etc. This is all economic witchcraft. Why not consume less, work less, have more time to dream, more time to philosophize?

Our wealth is one another, our friendships, our human capacities, the world of culture that human beings have created. When will there be an economics of that? Could there be, or is it more realistic to assume that the very concept of being 'economical' is at fault, that human beings are at their best when they are extravagant, when they don't count the cost? When was the last time you treated yourself — or your partner, or family — to something you couldn't afford? If you ever did, did you feel guilty afterwards? Shouldn't one feel more guilty at allowing such base considerations as money to influence one's decisions? (Actually, I think we do — based on my own experience.)

I sympathize with the Russian Minister of Finance. Alcohol and tobacco are two of the greatest benefits bestowed on humankind and at the same time two of the greatest curses. They are not just 'addictions'. They make you feel good. I can't think of anything more important then feeling good about oneself and about the world. You'll say that the country 'doesn't need' even more resources expended on the illnesses caused by smoking, or the social disorder caused by drinking. But maybe there is a balance that hasn't been reached yet. The economic benefits of a ten percent increase in smoking, say, marginally outweigh the cost of the increased burden on the health services. I can see that.

In his question, Derrick refers to the Greek philosophers. One of the fashionable trends in contemporary business ethics — reflected in the number of articles on this topic published in Philosophy for Business — is the application of Aristotelian virtue theory to the business world. The focus on the virtues needed for the 'good life', and in particular, the virtues needed to be a good business person, is one that I welcome. (See my Ethical Dilemmas, in particular Unit 10.)

The problem is that if you are looking to redress the imbalance between the rich and the poor, Aristotle and Greek philosophy generally is the wrong model. The Greeks had no problem with the idea of social inequality. Slaves were an essential part of the well-ordered polis. Unless you give a totally false, 'Christianized' gloss on the notion of 'virtue', there is no necessary corollary that exercising the virtues, or the business virtues will lead to a 'fairer' world, where we can all be free and equal together.

But I agree with Derrick that the world is in a mess, in so many ways, as it always has been (although that's no comfort).

My response is unoriginal, one that you will have heard many times before. If you can't change the world, if things move too slowly regardless of your best efforts, then at least you can work on yourself. If you are well-off, in a good job, then stop being so complacent. Become aware of your over-dependence on the system, which rewards you now but tomorrow may kick you out through the back door. If you are poor, then stop complaining. Consider all the ways there are of improving yourself without amassing useless material possessions. Ask how you can be helpful to others rather than just looking to others for help.

I am going to publish my answer to Derrick in the next issue of Philosophy for Business, which is due to go out at the beginning of next week, provided I can scratch together another couple of articles to go with it. If you are a philosopher or business ethicist reading this, then the offer of the Editorship is genuine. There's no salary, but then there's not a lot of work to do. Mainly, you will be badgering (or, if necessary, bullying) colleagues or people you know into writing articles. It would look good on anyone's CV.

If you're interested, email me on klempner@fastmail.net. Initially, you will be invited to guest edit one issue. This is an experiment we've successfully tried in the past. If you pass the test, and still have the appetite for more, then the job's yours for as long as you can continue the flow of quality material. Think about it. It could change your life — it certainly changed mine.

Geoffrey Klempner

back

(131) Marion asked:

I've just read an article titled 'Row over Crematorium Heating for Swimmers' which details plans of a local council, in an attempt to save energy, 'recycling' the heat generated from an adjacent crematorium to heat a swimming pool. Some have slammed the idea calling it 'Sick and an insult to local residents', and I agree with them. Is this not ethically unacceptable ?

---

We treat corpses with care, and the plan might seem to be uncaring of them.

First, all cultures honour their dead, although the way they do it varies — some bury them with a speech; some leave them for the birds to eat; some eat them to acquire their wisdom; in this country we mostly send them to an oven to music. And to use the heat from the burn, even as we exit the chapel, strikes some as not fully honouring the recently dead.

Secondly, not unrelated to the first point, we tend to treat a recently dead person as not quite dead until the corpse has been honourably dealt with. Two examples from my own experience.

1. Explaining the extra expense, the undertaker told me my dead sister would be 'more comfortable' in a deeper coffin (she had died in the foetal position and the body couldn't be straightened out). 2. Relatives often refuse to allow a post-mortem examination requested by the doctor on the grounds that their loved one 'has suffered enough', as if she were still capable of suffering.

So I understand people's distaste, and if the locals don't want the plan, fair enough.

I don't think the plan is unethical. For me, ethics is to do with how we should treat the living, not how we deal with corpses, although of course the living have feelings on the matter as I have described.

I would be the first to support such a scheme if my council came up with it. I'd prefer the heat from burning my body, or that of any of my loved ones, to be used rather than to be wasted

Craig Skinner

back

(132) Louella asked:

Kindly explain the saying,

'NOTHING IS WHAT IT SEEMS.'

or does REALITY exist?

thanks...

---

The saying 'Nothing is what it seems' arises from the claim that everything we perceive around us is not reality but just images of reality. There are two arguments for this. One is that everything we perceive is composed of sensations: colours, sounds, tastes, smells, and tactile sensations such as various degrees of hardness, temperature, weight, smoothness, etc.; and sensations are manufactured by our brains out of data from our sense organs; so these structures of sensations are at best images of an external reality, and at worst hallucinations. The other argument is that everything we perceive is at least a little bit illusory, and illusions can only be explained as being somewhat false images of reality. (If you doubt this, try to point to some perceived object which is wholly free from illusion, and explain how you know it to be so.)

The counter argument to this position is that yes, our brains contain images of reality; but what we perceive is outside our heads, material, and public, while brain images are inside our heads, mental and private. So what we perceive, being external, material and public, is reality, not images of reality. This is, of course, the common sense view, and the view held by almost philosophers today (e.g. analytic philosophers and phenomenologists).

The two positions are logically simple to reconcile, but psychologically difficult to do so. Without going into too much detail, the reconciliation, due to Leibniz and, later, Russell, comes from the observation that we each perceive our own body, including our own head. If everything we perceive is an image of reality, then we each have a real body, including a real head, and this real head contains all our images of reality, including our own image body. Our farthest image is the blue sky (or the star-spangled night sky, which, as an image, is equally distant.) This means that beyond the blue sky is the inside surface of your real skull. You see what I mean about psychologically difficult? Your empirical world is inside your real head and outside your empirical head; empirical objects are (somewhat) public by similarity to other people's empirical objects, but not public by identity (as common sense believes); and they are empirically material and at the same time really mental.

This brings us to your second question. There are two useful definitions of reality. One, which we might call empirical reality, is that it is all the we perceive around us that is non-illusory, which in turn is all that is potentially, universally, public; this is the field of study of empirical science. The other, which we might call theoretical reality, is everything that exists independently of being perceived; this is the field of study of theoretical science. So whether reality exists is two questions. Some of empirical reality exists indubitably as long as you are conscious, unless solipsism is true, in which case you are all that exists and everything empirical is unreal. And in so far as theoretical science is true, so does theoretical reality exist.

Helier Robinson

back

(133) Alex asked:

Hey there! My question is: does nothingness exist? If (hypothetically, I am religious) everything came from nothing, didn't SOMETHING cause that? Does profound and absolute nothingness exist?

---

No, nothingness does not exist. If everything came from nothing it was because a first cause was self-caused; or else because there never was a beginning, so that everything was caused by something earlier, during an infinite past.

Helier Robinson

back

(134) Marion asked:

I've just read an article titled 'Row Over Crematorium Heating For Swimmers' which details plans of a local council, in an attempt to save energy, 'recycling' the heat generated from an adjacent crematorium to heat a swimming pool. Some have slammed the idea calling it 'Sick and an insult to local residents', and I agree with them... Is this not ethically unacceptable?

---

A crematorium has two sources of heat. The major one by far is natural gas; the other is the heat given of by the burning body. If they could be separated there would be no problem: the gas heat could go to the pool and the body heat to the atmosphere. But since they cannot be separated, which is better: a lot of gas heat going to the atmosphere, or a little body heat going to the pool?

Helier Robinson

back

(135) Alex asked:

Hey there! My question is: does nothingness exist? If (hypothetically, I am religious) everything came from nothing, didn't SOMETHING cause that? Does profound and absolute nothingness exist?

---

Well, Alex: without mincing words: nothingness is nothingness, and that's all there is to it. You can't use language and then twist it around to make the words mean things they don't mean.

You have to attack this problem in another way. And now there are two ways of doing it, and you may choose whichever you prefer.

The first is: 'what does existence mean when there is no conscious knower around to certify it?' If life was wiped out (and assuming there is no life elsewhere in the universe), can the Andromeda galaxy still be said to exist? Of course it can't, because there no-one around to say it. So this is your 'profound' issue. We humans simply presuppose that existence is existence, as if the human mind could confidently arbitrate that X has always existed because we measure its age and conclude that it was around 15 billions years ago. We also assume that '15 billions years' is a phrase that makes sense. Maybe, maybe not.

However, for you as a religious person that problem can be phrased another way. Namely that God is a name for existence. Then your question is not about existence as such, but about created existence: namely, why did God create the world when this creative act requires work, whereas nothingness requires no work? Did God want something to do? Was he bored? I'm not being silly: there are several great thinkers who have wrestled with just such ideas and (more or less) supposed that a supreme intellect is the principle of activity in itself and must create, because to exist means to be creative.

So this problem boils down to a choice. Your choice, my choice, NOT the universe's choice: Nothingness and existence are words that have a meaning only to a conscious intelligence. You and I are conscious existences; so is God to those who believe in him.

The effect of this is that we humans claim that existence has a specific meaning. But we cannot prove this. A religious person does not have to prove it; but an irreligious person must. So a religious person is entitled to say: existence is God (=conscious intelligence) and everything flows from this. Then you can also say that 'nothingness' refers only to the material universe, not the spiritual universe.

If you are scientist, then you have to explain where existence takes off. But science cannot handle consciousness as a scientific term, and therefore they ignore it. But this leaves them unable to prove that there was existence before there was consciousness. Some people (including me) would say this is a self-contradiction.

Jurgen Lawrenz

back

(136) Wella asked:

Life is really a mystery, because we ourselves is part of God's mystery...

My question is, how can we make sure that we are facing the world of reality, and how can we say that we are not dreaming?

---

This is a question that has been asked of philosophers for at least 2000 years, and many of them have given different answers. But rather than giving you a long-winded explanation, a few simple observations you can make yourself should set your mind to rest.

1. When you face reality in an inappropriate way, it will bite you. So there is one argument that there is a reality of which you are part. What that reality actually is, that's another question. For the moment, though, you can be sure that reality is real, because you and every living creature have to respond to whatever conditions it imposes on life.

2. If we are all dreaming, then we are all dreaming twice: once in life, and then again every night. And those dreams are different from each other. There are two problems with this. First, that dreaming you are dreaming doesn't sound like a probable state of being in the world. Second, if life is just a dream, but everybody (including the plant and animal world) share in that dream and live along with us, then it doesn't matter about mere words. You can call it a dream life, but that's just a phrase. Because what everybody dreams as life, adds up to reality. You and every living being in this world consent that their experience is the same: when you and your dog go to New York you see and experience what every else can see and experience. So at worst you get the equation reality=dream, therefore dream=reality.

There is a third comment to add. We sense the world, and we all sense it in a coherent way, so that all humans can agree on most things as to what they are. But when you look through a microscope, familiar things become unfamiliar, a whole 'different' world reveals itself. Also, our senses sometimes hallucinate. These two features of our life create confusion in people's minds. They jump the gun and suppose that there must be a 'real' world beneath or beyond our world. That's pretty feeble, I'm sorry to be so blunt. All it tells us that the world we experience is tailored to our form of life. The world of fish is necessarily different; and intellectuals among them might well speculate if they are not just dreaming their life... you get the point?

Jurgen Lawrenz

back

(137) Harman asked:

In Descartes' second meditation, he claims there are many more things beyond doubt in the same way his own existence is. He says 'I now hear a noise, I feel heat. These things are false since I am asleep. Yet I do seem to see, hear and feel warmth. This cannot be false. Properly speaking, this is what in me is called 'sensing'. But this precisely so taken, is nothing other than thinking.' Does this mean that Descartes says his senses cannot be doubted?

---

Hi Harman, taken in isolation, I can understand how you might assume that this quotation infers that Descartes held the view that his senses could not be doubted, however, if you read the text immediately following this passage (that relating to the 'ball of wax'), you will find that Descartes quickly moves to question this approach. Thus, rather than saying that the senses cannot be doubted, Descartes held that all empirical evidence — all knowledge deriving from the senses, is unreliable. What he is interested in was a priori clear and distinct ideas: ideas which do not depend on sensory experience, but ideas or knowledge that derives only from reason. If you would like a more extensive explanation of Descartes' philosophy, you may find this piece (put together by me some time ago) of some interest.

Descartes' Discourse on Method

With the Renaissance came the rediscovery of man in nature and a desire to assemble contemporary thought into one coherent philosophical system again presented itself. Rene Descartes (1596-1650) came to believe that the body of knowledge inherited from the Middle Ages was unreliable. His main concern was with what we can know. This form of philosophy is called epistemology. In other words, 'certain knowledge'. Descartes was convinced that there was a division between spirit and matter. In his Discourse on Method he raises the question of the method the philosopher must employ in the search for truth. Descartes argued that we should not accept anything as being true unless we can perceive it clearly and distinctly. To achieve this we must break down a compound problem into as many single factors as possible. Then we must take our departure point the simplest idea of all. Every single thought must be weighed and measured, just as Galileo wanted everything to be measured and everything immeasurable to be made measurable. Descartes believed that philosophy should move from the simple to the complex, because only then would it be possible to construct a new insight. Finally, it would be necessary to ensure, by constant control and enumeration that nothing was left out. Only then could a philosophical conclusion be within reach.

Descartes' aim was to reach certainty about the nature of life. He begins by maintaining that at first one should doubt everything. Following this chain of thought he came to conclude that one thing he could be sure of was that he was doubting. Moreover, because he was doubting, he must be thinking, and because he was thinking he must surely be a thinking being. Or, as Descartes says 'Cogito, ergo sum' ('I think, therefore I am'). Following Parmenides and Plato, Descartes held that that which we grasp with our reason is more real than that which we grasp with our senses. He perceived not only that he was a thinking 'I', but that this thinking 'I' was more real than the material world which we perceive through the senses. This is the kernel of Descartes' theory of knowledge. Bertrand Russell maintains that it contains that which is 'most important in his [Descartes'] philosophy :'I think, therefore I am', makes mind more certain than matter, and my mind (for me) more certain than the mind of others' (History of Western Philosophy, 1991, p. 550. Routledge London). Having established his existence, Descartes proceeded to investigate his nature of essence. What kind of thing am I? he asked. I am not essentially a physical being, for, applying the method of doubt, I can doubt whether I have a body — or even that external objects exist. One thing that I can be sure of is that because I am a thinking being, the essence of my nature is to think. Furthermore, this thinking 'I' is separate from my physical body.

Descartes' Theory of Dualism

Descartes believed that each of us possesses the idea of a perfect entity. Inherent in that idea is the fact that a perfect entity must exist — because, as Anselm had said, a perfect entity can only be perfect if it has existence. Neither could we conceive of a perfect entity if there was no such thing. We are imperfect, said Descartes, so the idea of perfection cannot come from us. Descartes reasons that the idea of a perfect being must have been placed in him by a really existing perfect being — God. That God exists was therefore as self-evident to Descartes as that a thinking being must exist.

Descartes believed that the idea of God was innate; it was something we are born with. The more self-evident a thing is to one's reason, the more certain it is that it exists. From this he concluded that he was a thinking being and that there exists a perfect entity, God. With this as his departure point, Descartes lays out his theory of dualism. With regard to all the ideas we have concerning outer reality, there is possibility that we are deceived. We think we have a body but we may be dreaming; we cannot be certain that we have a body. Descartes believed his body and all the non-conscious natural world was non-essential, that is, contingent. It is important to realise that Descartes is not saying that the material world does not exist, but that its existence is radically unlike that of the mind. His body is not part of his essence, therefore, if his body ceased to exist, his mind would not cease to be all that it is. In other words, Descartes would continue to be Descartes even if he had no body.

There are two kinds of reality — two substances — says Descartes. One is thought, or mind, the other is extension, or matter. The mind is purely conscious and occupies no room in space and therefore cannot be subdivided into smaller and smaller parts. Matter, on the other hand has no consciousness. Descartes maintained that both mind and matter originate from God, because only God exists independently of anything else. Although both substances come from God, they are independent of each other. Thought is independent of matter and conversely, the material processes are independent of thought.

Descartes, then, can be said to have divided God's creation in two. In this way, we understand that he was a dualist: that is, he effects a sharp division between the reality of thought and the reality of extension, or matter. According to Descartes, the human body is a machine. But man also has a mind which operates independently of the body. Descartes' 'mind' can be compared to the Christian notion of 'soul'. Christianity teaches that the soul is independent of the body and therefore not limited to the co-ordinates of time and space; it is the source of consciousness and survives death of the body. The Cartesian mind can be understood in the same way. Descartes believed that the existence of the body was contingent to the mind — that is, that the mind does not depend on the body for its existence. This brings us to, what might be called, the 'Cartesian paradox', for although Descartes argued that the body and mind were separate substances, he did accept that there was constant interaction between them. As long as the mind is in the body, Descartes believed it is connected to the brain by a special 'organ'. This organ he called the 'pineal gland', and it is found at the base of the brain, somewhere in the region of the nose. Here, apparently, some sort of impact occurs between the physical, the extended brain and the unextended, thinking mind. The existence of the pineal gland allows for constant interaction between 'spirit' and 'matter'. In this way the mind can be constantly affected by feelings and passions that are related to bodily needs. But the mind also has the ability to detach itself from such 'base' impulses and operate independently of the body. This aim is to get reason to assume command.

Not surprisingly, the pineal gland theory actually produced more problems than it solved, since one could ask whether this was physical, and if so, how could it be connected to something that did not occupy space? If it were mental, how could it be next to any part of the brain? When this problem was pointed out to Descartes he became increasingly vague about the issue and insisted that the fact of the interaction was known to everybody. 'We experience it all the time', he said, 'but how mind and body were united', he admitted, 'was most difficult to explain' (see Philosophy. Popkin and Stroll. 1996: 125). In parenthesis, it should be said that there actually is such a gland (also called the 'pineal body' or 'epiphysis'). It is a small endocrine system endocrine gland in the brain. It is located near the center of the brain, between the two cerebral hemispheres, tucked in a groove where the two rounded thalamus-thalamic bodies join. However, there is absolutely no evidence that it serves the purpose described by Descartes and it should come as no surprise to learn that the 'curious transactions of the pineal gland' was dropped by followers of Descartes.

Descartes' Metaphysics

In Descartes' metaphysics there are three basic components of the universe, called substances: God, mind, and matter. All other things are modifications of these. God is the creative substance, who created mind and matter. The essential property of the mental substance is that it thinks. The essential property of matter is that it is extended.

By reducing all material entities to extended substances, Descartes produced a philosophical justification for a new picture of the natural world. Like Newton, he saw the physical world as an elaborate mechanism — a grand cosmic clockwork. This vast world machine operates in accordance to God's constant laws, says Descartes. God constantly converses with, and controls, a physical order in which various portions of extension move others by contact, producing the regular world that modern science describes. Descartes believed that everything that is extended is part of this elaborate machine. The only aspect of the created world that is not part of the world machine is mind. Mind, says Descartes, is completely unextended and therefore, not in contact with the material world — which brings us back to the unresolved problem of the pineal gland. The activities of the mind include: thinking, willing, reasoning, and so on. In this way Descartes' world machine, in denying any purposeful capabilities in the physical world, provided a suitable metaphysical basis for the new scientific theories of such scientists as Galileo in that it gave scientific activity intellectual respectability without offending the Church. It should also be said that Descartes was not without his critics. Blaise Pascal (1623-1662), for example, saw Descartes as a man of little understanding of religious faith, and his remark that 'The heart has its reasons which reason knows not' (see Pensees) was directly aimed at Descartes.

One argument against Descartes' concept of the I or ego as an intrinsic, independent entity divorced from the body is that it devalues the existence of other human beings. In fact, it could be said that, were it to be true, it reduces other people to non-beings: simply to figments of the ego's imagination. In such a situation there would be no compunction or obligation for the 'thinking I' to attempt to empathise or sympathise with others. Since all would could be certain of is that one alone exists, there would be no reason for a charter of human rights; no need to think of anyone's happiness but one's own, for to possess such an intrinsic, independent existence is to hold the view that the ego has the capacity to remain untouched by or influenced by the actions of others and of other things.

Tony Fahey

back

(138) Terri asked:

What do you think of Mill's 'pig and Socrates' argument for the difference between sensual and intellectual pleasures?

---

I think the difference between sensual and intellectual pleasures can become blurred. Sensual pleasures can be subject to intellectual reception. Playing pushpin, darts, and so on can be subject to PhD research on culture, sociology, anthropology.

Soap-Operas have been the subject of Dissertations and research. So for me, it is how the subject matter is interpreted. Popular music-much derided-can be as emotive, as meaningful, evocative as 'classical'.

This however, does not negate Mill's contention. For Socrates can appreciate the sensual pleasures intellectually whereas, the 'fool' could not. Is it a matter of happiness though? As someone once said in a Philosophy class, 'I was alright before I knew about the 'Mind-Body' problem'. Once the problem is appreciated, I don't see happiness increasing. Perhaps the opposite. Perhaps it is a matter of having more knowledge at one's or a groups disposal, which is the same thing as being a vehicle of power?

'Man does not strive after happiness, only the Englishman does that'.

Friedrich Nietzsche. #12. Maxims and Arrows, Twilight of the Idols.

Martin Jenkins

back

(139) Daniel asked:

Who 'owns' the space between an upright airplane seat and a reclined airplane seat: the person in the seat, or the person behind the seat? Is one always entitled to put one's seat back in a plane? Only some of the time? And even if one is entitled, are there cases when one ought not exercise one's right?

---

Interesting question. It highlights the difference between 'rules' (laws, morals, etc.) and 'etiquette'.

Obviously, it is the airline that 'owns' the space in question and I am sure that you are aware of that, given the 'scare-quotes' you employed. But the airline has seen fit to rent out that space to the person who has purchased a ticket (usually for that specific seat). Since the ticket holder has the right to use that seat for the duration of the flight, the ticket holder also has the right to make the fullest possible use of that seat consistent with the rules established by the airline (ie. whether a baby can be carried in arms, or must have its own seat; various restrictions on the storage and use of hand luggage; and so forth). In those cases where the airline wants to restrict that right (say, in the emergency exit rows), they fix the seats so that they cannot recline.

I know of no rules established by any airline that constrains the ticket holder from reclining the seat to any extent, at any time (except of course, when the steward(ess) instructs passengers to 'return seat backs and tray tables to their upright position'). Hence, I do not believe there are any rules constraining ones entitlement to recline ones seat back when that is permissible.

So whether, and how far, you recline your seat back is a question of how much consideration you have for the person in the seat behind you. And that, of course, can vary according to circumstances. Thus, behaviour in this regard is a matter of etiquette how you hold your pinkie whilst drinking tea. As to whether there may be cases where one ought not exercise ones right to recline ones seat-back that is a matter of fine judgement on your part. Just how considerate do you wish to be to the person in the seat behind you? There may be circumstances where it is clear to you that you ought to be more (or less) considerate than average (or at all). Unfortunately, in the absence of any rules on the subject, if the person you are sitting behind chooses to be inconsiderate, there is not much you can do about it except fume uselessly, and vow never to be considerate to that person if the opportunity should ever present. That is, regrettably, the only enforcement for standards of etiquette.

I must admit that I have given up flying in the last few years (it's become such a pain in the a**) so I am a little out of date with the etiquette displayed by the average passenger. But when I was flying, it was my experience that many male passengers displayed little consideration for their posterior seat mate and reclined their seat backs to the fullest degree whenever permitted. Whereas few female passengers did. I have no idea whether this difference in behaviour was due to a difference in their respective sensitivities to seat-back etiquette, or to a physiological difference between the sexes as to what seat position was more comfortable.

Stuart Burns